Real Property
Real Property
Real Property
REAL PROPERTY i.
REAL PROPERTY
TABLE OF CONTENTS
I.
ESTATES IN LAND . . . . . . . . . . . . . . . . . . . . . . . . . . . . . . . . . . .
A. IN GENERAL . . . . . . . . . . . . . . . . . . . . . . . . . . . . . . . . . . . .
B. PRESENT POSSESSORY ESTATES . . . . . . . . . . . . . . . . . . . . . . .
1. Fee Simple Absolute . . . . . . . . . . . . . . . . . . . . . . . . . . . . . .
2. Defeasible Fees . . . . . . . . . . . . . . . . . . . . . . . . . . . . . . . . .
a. Fee Simple Determinable (and Possibility of Reverter) . . . . . . . .
1) Correlative Future Interest in GrantorPossibility of Reverter
a) Possibility of Reverter Need Not Be Expressly Retained . .
b) Transferability of Possibility of Reverter . . . . . . . . . . .
2) Correlative Future Interest in Third PartyExecutory Interest
b. Fee Simple Subject to Condition Subsequent (and Right of Entry) . .
1) Correlative Future Interest in GrantorRight of Entry . . . . .
a) Failure to Reserve Right of Entry . . . . . . . . . . . . . .
b) Waiver of Right of Entry . . . . . . . . . . . . . . . . . . .
(1) Inaction by Itself Not a Waiver . . . . . . . . . . . . .
c) Transferability of Right of Entry . . . . . . . . . . . . . . .
2) Correlative Future Interest in Third PartyExecutory Interest
3) CompareFee Simple Determinable . . . . . . . . . . . . . . .
a) Construction of Ambiguous Language . . . . . . . . . . . .
c. Fee Simple Subject to an Executory Interest . . . . . . . . . . . . . .
d. Limitations on Possibilities of Reverter and Rights of Entry . . . . .
e. Conditions and Limitations Violating Public Policy . . . . . . . . . .
1) Restraints on Marriage . . . . . . . . . . . . . . . . . . . . . . .
2) Provisions Involving Separation or Divorce . . . . . . . . . . . .
3. Fee Tail . . . . . . . . . . . . . . . . . . . . . . . . . . . . . . . . . . . . .
4. Life Estate . . . . . . . . . . . . . . . . . . . . . . . . . . . . . . . . . . .
a. Life Estates by Marital Right (Legal Life Estates) . . . . . . . . . . .
b. Conventional Life Estate . . . . . . . . . . . . . . . . . . . . . . . . .
1) For Life of Grantee . . . . . . . . . . . . . . . . . . . . . . . . .
2) Life Estate Pur Autre Vie (Life of Another) . . . . . . . . . . . .
a) Inheritability . . . . . . . . . . . . . . . . . . . . . . . . . .
c. Rights and Duties of Life TenantDoctrine of Waste . . . . . . . . .
1) Affirmative (Voluntary) WasteNatural Resources . . . . . . .
a) Open Mines Doctrine . . . . . . . . . . . . . . . . . . . . .
2) Permissive Waste . . . . . . . . . . . . . . . . . . . . . . . . . .
a) Obligation to Repair . . . . . . . . . . . . . . . . . . . . . .
b) Obligation to Pay Interest on Encumbrances . . . . . . . .
c) Obligation to Pay Taxes . . . . . . . . . . . . . . . . . . . .
d) Special Assessments for Public Improvements . . . . . . . .
(1) Apportionment of Costs . . . . . . . . . . . . . . . . .
e) No Obligation to Insure Premises . . . . . . . . . . . . . .
f) No Liability for Third Partys Torts . . . . . . . . . . . . .
3) Ameliorative Waste . . . . . . . . . . . . . . . . . . . . . . . . .
.
.
.
.
.
.
.
.
.
.
.
.
.
.
.
.
.
.
.
.
.
.
.
.
.
.
.
.
.
.
.
.
.
.
.
.
.
.
.
.
.
.
.
.
.
.
.
.
.
.
.
.
.
.
.
.
.
.
.
.
.
.
.
.
.
.
.
.
.
.
.
.
.
.
.
.
.
.
.
.
.
.
.
.
.
.
.1
.1
.1
.1
.1
.1
.2
.2
.2
.2
.2
.3
.3
.3
.3
.3
.3
.4
.4
.4
.5
.5
.5
.5
.5
.5
.6
.6
.6
.7
.7
.7
.7
.7
.8
.8
.8
.8
.8
.8
.8
.9
.9
C.
a) CompareLeasehold Tenant . . . . . . . . . . . . . . . . . . .
b) CompareWorthless Property . . . . . . . . . . . . . . . . . .
d. Renunciation of Life Estates . . . . . . . . . . . . . . . . . . . . . . . . .
5. Estates for Years, Periodic Estates, Estates at Will, Tenancies at Sufferance .
FUTURE INTERESTS . . . . . . . . . . . . . . . . . . . . . . . . . . . . . . . . .
1. Reversionary InterestsFuture Interests in Transferor . . . . . . . . . . . .
a. Possibilities of Reverter and Rights of Entry . . . . . . . . . . . . . . . .
b. Reversions . . . . . . . . . . . . . . . . . . . . . . . . . . . . . . . . . . .
c. All Reversionary Interests Are Vested . . . . . . . . . . . . . . . . . .
2. Remainders . . . . . . . . . . . . . . . . . . . . . . . . . . . . . . . . . . . .
a. Indefeasibly Vested Remainder . . . . . . . . . . . . . . . . . . . . . . .
b. Vested Remainder Subject to Open . . . . . . . . . . . . . . . . . . . . .
1) Divesting Interests Are Executory Interests . . . . . . . . . . . . . .
2) Effect on Marketability of Title . . . . . . . . . . . . . . . . . . . .
c. Vested Remainder Subject to Total Divestment . . . . . . . . . . . . . .
d. Contingent Remainder . . . . . . . . . . . . . . . . . . . . . . . . . . . .
1) Subject to Condition Precedent . . . . . . . . . . . . . . . . . . . .
2) Unborn or Unascertained Persons . . . . . . . . . . . . . . . . . . .
3) Destructibility of Contingent Remainders . . . . . . . . . . . . . . .
a) Rule Abolished . . . . . . . . . . . . . . . . . . . . . . . . . . .
b) Related Doctrine of Merger . . . . . . . . . . . . . . . . . . . .
(1) CompareInterests Created Simultaneously . . . . . . .
e. Rule in Shelleys Case (Rule Against Remainders in Grantees Heirs) . .
f. Doctrine of Worthier Title (Rule Against Remainders in Grantors
Heirs) . . . . . . . . . . . . . . . . . . . . . . . . . . . . . . . . . . . . .
3. Executory Interests . . . . . . . . . . . . . . . . . . . . . . . . . . . . . . . .
a. Shifting Executory InterestDivests a Transferee . . . . . . . . . . . .
b. Springing Executory InterestFollows a Gap or Divests a
Transferor . . . . . . . . . . . . . . . . . . . . . . . . . . . . . . . . . .
c. Executory Interest Follows a Fee . . . . . . . . . . . . . . . . . . . . . .
d. Differences Between Executory Interests and Remainders . . . . . . . .
4. Importance of Classifying Interests In Order . . . . . . . . . . . . . . . .
5. Transferability of Remainders and Executory Interests . . . . . . . . . . . .
a. Vested Remainders Are Transferable, Devisable, and Descendible . . . .
b. Contingent Remainders and Executory Interests Are Transferable Inter
Vivos . . . . . . . . . . . . . . . . . . . . . . . . . . . . . . . . . . . . . .
c. Contingent Remainders and Executory Interests Are Usually Devisable
and Descendible . . . . . . . . . . . . . . . . . . . . . . . . . . . . . . .
d. Any Transferable Future Interest Is Reachable by Creditors . . . . . . .
e. Practical Ability to Transfer Marketable Title . . . . . . . . . . . . . . .
6. Class Gifts . . . . . . . . . . . . . . . . . . . . . . . . . . . . . . . . . . . . .
a. Definitional Problems . . . . . . . . . . . . . . . . . . . . . . . . . . . .
1) Dispositions to Children . . . . . . . . . . . . . . . . . . . . . . .
2) Dispositions to Heirs . . . . . . . . . . . . . . . . . . . . . . . . .
3) Dispositions to Issue or Descendants . . . . . . . . . . . . . . .
4) Class Members in Gestation . . . . . . . . . . . . . . . . . . . . . .
b. When the Class ClosesThe Rule of Convenience . . . . . . . . . . . . .
1) Outright GiftClass Closes at Time Gift Is Made . . . . . . . . . .
.9
.9
.9
10
10
10
10
10
11
11
12
12
13
13
13
14
14
15
15
16
16
17
17
18
18
18
19
19
19
20
21
21
21
21
21
21
21
22
22
22
22
22
22
22
a)
D.
E.
.
.
.
.
.
.
23
23
23
24
24
25
.
.
.
.
.
.
.
.
.
.
.
.
.
.
.
.
.
.
.
.
.
.
.
.
.
.
.
.
.
.
.
.
.
.
.
.
.
.
.
.
25
25
25
25
25
25
26
26
26
26
26
26
26
26
26
26
27
27
27
28
28
28
28
28
28
28
29
29
30
30
31
31
31
32
32
32
32
33
33
33
a.
b.
c.
d.
e.
f.
.
.
.
.
.
.
.
.
.
.
.
.
.
.
.
.
.
.
.
.
.
.
.
.
.
.
.
.
.
.
.
.
.
.
.
.
.
.
.
.
.
.
.
.
.
.
.
.
.
.
.
.
.
.
.
.
.
.
.
.
.
.
.
.
.
.
.
.
.
.
.
.
.
.
.
.
.
.
.
.
.
.
.
.
.
.
.
.
.
.
.
.
.
.
.
.
.
.
.
.
.
.
.
.
.
.
.
.
.
.
.
.
.
.
.
.
.
.
.
.
.
.
.
.
.
.
.
.
.
.
.
.
.
.
.
.
.
.
.
.
.
.
.
.
.
.
.
.
.
.
.
.
.
.
.
.
.
.
.
.
.
.
.
.
.
.
.
.
.
.
.
.
.
.
.
.
.
.
.
.
.
.
.
.
.
.
.
.
.
.
.
.
.
.
.
.
.
.
.
.
.
.
.
.
.
.
.
.
.
.
.
.
.
.
.
.
.
.
.
.
.
.
.
.
.
.
.
.
.
.
.
.
.
.
.
.
.
.
.
.
.
.
.
.
.
.
.
.
.
.
.
.
.
.
.
.
.
.
.
.
.
.
.
.
.
.
.
.
.
.
.
.
.
.
.
.
.
.
.
.
.
.
.
.
.
.
.
.
33
34
35
35
35
36
36
36
36
37
37
37
38
38
39
39
39
39
39
40
40
40
40
40
40
40
41
41
41
41
41
41
42
42
42
42
42
42
42
43
43
43
43
43
43
43
44
44
REAL PROPERTY v.
a)
b)
2.
3.
4.
II.
.
.
.
.
.
.
.
.
.
.
.
.
.
.
.
.
.
.
.
.
.
.
.
.
.
.
.
.
.
.
.
.
.
.
.
.
.
.
.
.
.
.
.
.
.
.
.
.
.
.
.
.
.
.
.
.
.
.
.
.
.
.
.
.
.
.
.
.
.
.
.
.
.
.
.
.
.
.
.
.
.
.
.
.
.
.
.
.
.
.
.
.
.
.
.
.
.
.
.
.
.
.
.
.
.
.
.
.
.
.
.
.
.
.
.
.
.
.
.
.
.
.
.
.
.
.
.
.
.
.
.
.
.
.
.
.
.
.
.
.
.
.
.
.
.
.
.
.
.
.
.
.
.
.
.
.
.
.
.
.
.
.
.
.
.
.
.
.
.
.
.
.
.
.
.
.
.
.
.
.
.
.
.
.
.
.
.
.
.
.
.
.
.
.
.
.
.
.
.
.
.
.
.
.
.
.
.
.
.
.
.
.
.
.
.
.
.
.
.
.
.
.
.
.
.
.
.
.
.
.
.
.
.
.
.
.
.
.
.
.
.
.
.
.
.
.
.
.
.
.
.
.
.
.
.
.
.
.
.
.
.
.
.
.
.
.
.
.
.
.
.
.
.
.
.
.
.
.
.
.
.
.
.
.
.
.
.
.
.
.
.
.
.
.
.
.
.
.
.
.
.
.
.
.
.
.
.
.
.
.
.
.
.
.
.
.
.
.
.
.
.
.
.
.
.
.
.
.
.
.
.
.
.
.
.
.
.
.
.
.
.
.
.
.
.
.
.
.
.
.
.
.
.
.
.
.
.
.
.
.
.
.
.
.
.
.
.
.
.
.
.
.
.
.
.
.
.
.
.
.
.
.
.
.
.
.
.
.
.
.
.
.
.
.
.
.
.
.
.
.
.
.
.
.
44
44
44
44
45
45
45
45
46
46
46
46
46
46
46
46
47
47
47
47
47
47
48
48
48
48
49
49
49
49
.
.
.
.
.
.
.
.
.
.
.
.
.
.
.
.
.
49
49
50
50
50
50
50
50
50
50
51
51
51
51
51
51
51
3.
B.
C.
D.
Tenancies at Will . . . . . . . . . . . . . . . . . . . . . . . . . . . . . . . .
a. Creation . . . . . . . . . . . . . . . . . . . . . . . . . . . . . . . . . . .
b. Termination . . . . . . . . . . . . . . . . . . . . . . . . . . . . . . . . .
4. Tenancies at Sufferance . . . . . . . . . . . . . . . . . . . . . . . . . . . . .
5. The Hold-Over Doctrine . . . . . . . . . . . . . . . . . . . . . . . . . . . .
a. Eviction . . . . . . . . . . . . . . . . . . . . . . . . . . . . . . . . . . .
b. Creation of Periodic Tenancy . . . . . . . . . . . . . . . . . . . . . . .
1) Terms . . . . . . . . . . . . . . . . . . . . . . . . . . . . . . . . .
2) Altered Terms . . . . . . . . . . . . . . . . . . . . . . . . . . . . .
c. What Does Not Constitute Holding Over . . . . . . . . . . . . . . . . .
d. Double Rent Jeopardy . . . . . . . . . . . . . . . . . . . . . . . . . . .
e. Forcible Entry Statutes . . . . . . . . . . . . . . . . . . . . . . . . . . .
LEASES . . . . . . . . . . . . . . . . . . . . . . . . . . . . . . . . . . . . . . . .
1. Common LawLease Covenants Independent . . . . . . . . . . . . . . . .
2. Modern TrendLease Covenants Dependent . . . . . . . . . . . . . . . .
TENANT DUTIES AND LANDLORD REMEDIES . . . . . . . . . . . . . . . .
1. Tenants Duty to Repair (Doctrine of Waste) . . . . . . . . . . . . . . . . .
a. Types of Waste . . . . . . . . . . . . . . . . . . . . . . . . . . . . . . .
1) Voluntary (Affirmative) Waste . . . . . . . . . . . . . . . . . . . .
2) Permissive Waste . . . . . . . . . . . . . . . . . . . . . . . . . . .
3) Ameliorative Waste . . . . . . . . . . . . . . . . . . . . . . . . . .
a) LiabilityCost of Restoration . . . . . . . . . . . . . . . . .
b) Modern ExceptionValue of Premises Decreasing . . . . . .
b. Destruction of the Premises Without Fault . . . . . . . . . . . . . . . .
1) Majority ViewTenant Can Terminate Lease . . . . . . . . . . .
c. Tenants Liability for Covenants to Repair . . . . . . . . . . . . . . . .
1) Rebuilding After Structural Damage or Casualty Destruction . .
2) Repairing Ordinary Wear and Tear . . . . . . . . . . . . . . . . .
2. Duty to Not Use Premises for Illegal Purpose . . . . . . . . . . . . . . . . .
a. Occasional Unlawful Conduct Does Not Breach Duty . . . . . . . . . .
b. Landlord RemediesTerminate Lease, Recover Damages . . . . . . .
3. Duty to Pay Rent . . . . . . . . . . . . . . . . . . . . . . . . . . . . . . . . .
a. When Rent Accrues . . . . . . . . . . . . . . . . . . . . . . . . . . . .
b. Rent Deposits . . . . . . . . . . . . . . . . . . . . . . . . . . . . . . . .
c. Termination of Rent LiabilitySurrender . . . . . . . . . . . . . . . .
4. Landlord Remedies . . . . . . . . . . . . . . . . . . . . . . . . . . . . . . .
a. Tenant on Premises But Fails to Pay RentEvict or Sue for Rent . . .
1) DistressLandlords Lien . . . . . . . . . . . . . . . . . . . . . .
b. Tenant AbandonsDo Nothing or Repossess . . . . . . . . . . . . . .
1) Landlord Does NothingTenant Remains Liable . . . . . . . . .
2) Landlord RepossessesTenants Liability Depends on Surrender
a) Acts that Constitute Acceptance of Surrender . . . . . . . . .
b) Acts that Do Not Constitute Acceptance of Surrender . . . .
LANDLORD DUTIES AND TENANT REMEDIES . . . . . . . . . . . . . . . .
1. Duty to Deliver Possession of Premises . . . . . . . . . . . . . . . . . . . .
a. Landlord DutyMust Deliver Actual Possession . . . . . . . . . . . .
b. Tenant RemedyDamages . . . . . . . . . . . . . . . . . . . . . . . .
2. Quiet Enjoyment . . . . . . . . . . . . . . . . . . . . . . . . . . . . . . . . .
.
.
.
.
.
.
.
.
.
.
.
.
.
.
.
.
.
.
.
.
.
.
.
.
.
.
.
.
.
.
.
.
.
.
.
.
.
.
.
.
.
.
.
.
.
.
.
.
52
52
52
52
53
53
53
53
53
53
53
54
54
54
54
55
55
55
55
55
55
55
55
56
56
56
56
56
57
57
57
57
57
57
57
57
57
58
58
58
58
58
58
59
59
59
59
59
a.
b.
Actual Eviction . . . . . . . . . . . . . . . . . . . . . . . . . . . . . .
Partial Actual Eviction . . . . . . . . . . . . . . . . . . . . . . . . . .
1) Partial Eviction by LandlordEntire Rent Obligation Relieved
2) Partial Eviction by Third PersonRent Apportioned . . . . . .
c. Constructive Eviction . . . . . . . . . . . . . . . . . . . . . . . . . .
3. Implied Warranty of Habitability . . . . . . . . . . . . . . . . . . . . . .
a. StandardReasonably Suitable for Human Residence . . . . . . . .
b. Remedies . . . . . . . . . . . . . . . . . . . . . . . . . . . . . . . . .
4. Retaliatory Eviction . . . . . . . . . . . . . . . . . . . . . . . . . . . . . .
5. Discrimination . . . . . . . . . . . . . . . . . . . . . . . . . . . . . . . . .
E. ASSIGNMENTS AND SUBLEASES . . . . . . . . . . . . . . . . . . . . . . .
1. Consequences of Assignment . . . . . . . . . . . . . . . . . . . . . . . . .
a. Covenants that Run with the Land . . . . . . . . . . . . . . . . . . .
b. Rent Covenant Runs with the Land . . . . . . . . . . . . . . . . . . .
1) Reassignment by AssigneePrivity of Estate with Landlord
Ends . . . . . . . . . . . . . . . . . . . . . . . . . . . . . . . . .
a) Effect of Assignee Assuming Rent Obligation . . . . . . . .
2) Original Tenant Remains Liable . . . . . . . . . . . . . . . . . .
2. Consequences of Sublease . . . . . . . . . . . . . . . . . . . . . . . . . . .
a. Liability of Sublessee for Rent and Other Covenants . . . . . . . . .
1) Termination for Breach of Covenants . . . . . . . . . . . . . . .
b. Assumption by Sublessee . . . . . . . . . . . . . . . . . . . . . . . . .
c. Rights of Sublessee . . . . . . . . . . . . . . . . . . . . . . . . . . . .
3. Covenants Against Assignment or Sublease . . . . . . . . . . . . . . . . .
a. Strictly Construed Against Landlord . . . . . . . . . . . . . . . . . .
b. Waiver of Covenant . . . . . . . . . . . . . . . . . . . . . . . . . . . .
c. Continuing Waiver . . . . . . . . . . . . . . . . . . . . . . . . . . . .
d. Transfer in Violation of Lease Not Void . . . . . . . . . . . . . . . .
e. Reasonableness . . . . . . . . . . . . . . . . . . . . . . . . . . . . . .
4. Assignments by Landlords . . . . . . . . . . . . . . . . . . . . . . . . . .
a. Right to Assign . . . . . . . . . . . . . . . . . . . . . . . . . . . . . .
b. Rights of Assignee Against Tenants . . . . . . . . . . . . . . . . . . .
c. Liabilities of Assignee to Tenants . . . . . . . . . . . . . . . . . . . .
F. CONDEMNATION OF LEASEHOLDS . . . . . . . . . . . . . . . . . . . . .
1. Entire Leasehold Taken by Eminent DomainRent Liability
Extinguished . . . . . . . . . . . . . . . . . . . . . . . . . . . . . . . . . .
2. Temporary or Partial TakingTenant Entitled to Compensation Only .
G. TORT LIABILITY OF LANDLORD AND TENANT . . . . . . . . . . . . . .
1. Landlords Liability . . . . . . . . . . . . . . . . . . . . . . . . . . . . . .
a. Concealed Dangerous Condition (Latent Defect) . . . . . . . . . . . .
b. Common Areas . . . . . . . . . . . . . . . . . . . . . . . . . . . . . .
c. Public Use . . . . . . . . . . . . . . . . . . . . . . . . . . . . . . . . .
d. Furnished Short-Term Residence . . . . . . . . . . . . . . . . . . . .
e. Negligent Repairs by Landlord . . . . . . . . . . . . . . . . . . . . .
f. Landlord Contracts to Repair . . . . . . . . . . . . . . . . . . . . . .
2. Modern TrendGeneral Duty of Reasonable Care . . . . . . . . . . . .
a. Defects Arising After Tenant Takes Possession . . . . . . . . . . . .
b. Legal Duty to Repair . . . . . . . . . . . . . . . . . . . . . . . . . . .
.
.
.
.
.
.
.
.
.
.
.
.
.
.
.
.
.
.
.
.
.
.
.
.
.
.
.
.
59
59
59
60
60
60
60
60
61
61
61
61
62
62
.
.
.
.
.
.
.
.
.
.
.
.
.
.
.
.
.
.
.
.
.
.
.
.
.
.
.
.
.
.
.
.
.
.
.
.
.
.
62
62
62
63
63
63
63
63
63
63
63
63
64
64
64
64
64
64
64
.
.
.
.
.
.
.
.
.
.
.
.
.
.
.
.
.
.
.
.
.
.
.
.
.
.
64
64
65
65
65
65
65
65
65
66
66
66
66
3.
III.
IV.
c. Security . . . . . . . . . . . . . . . . . . . . . . . . . . . . . . . . . . . . 66
Tenants Liability . . . . . . . . . . . . . . . . . . . . . . . . . . . . . . . . . 66
FIXTURES . . . . . . . . . . . . . . . . . . . . . . . . . . . . . . . . . . . . . . . .
A. IN GENERAL . . . . . . . . . . . . . . . . . . . . . . . . . . . . . . . . . . . .
B. CHATTELS INCORPORATED INTO STRUCTURE ALWAYS BECOME
FIXTURES . . . . . . . . . . . . . . . . . . . . . . . . . . . . . . . . . . . . .
C. COMMON OWNERSHIP CASES . . . . . . . . . . . . . . . . . . . . . . . .
1. Annexors Intent Controls in Common Ownership Cases . . . . . . . . .
a. Constructive Annexation . . . . . . . . . . . . . . . . . . . . . . . . .
b. Vendor-Purchaser Cases . . . . . . . . . . . . . . . . . . . . . . . . .
c. Mortgagor-Mortgagee Cases . . . . . . . . . . . . . . . . . . . . . .
2. Effect of Fixture Classication . . . . . . . . . . . . . . . . . . . . . . . .
a. Conveyance . . . . . . . . . . . . . . . . . . . . . . . . . . . . . . . .
b. Mortgage . . . . . . . . . . . . . . . . . . . . . . . . . . . . . . . . .
c. Agreement to Contrary . . . . . . . . . . . . . . . . . . . . . . . . .
D. DIVIDED OWNERSHIP CASES . . . . . . . . . . . . . . . . . . . . . . . . .
1. Landlord-Tenant . . . . . . . . . . . . . . . . . . . . . . . . . . . . . . .
a. Agreement . . . . . . . . . . . . . . . . . . . . . . . . . . . . . . . .
b. No Intent If Removal Does Not Cause Damage . . . . . . . . . . . . .
c. Removal Must Occur Before End of Lease Term . . . . . . . . . . .
d. Tenant Has Duty to Repair Damages Resulting from Removal . . . .
2. Life Tenant and Remainderman . . . . . . . . . . . . . . . . . . . . . . .
3. Licensee and Landowner . . . . . . . . . . . . . . . . . . . . . . . . . . .
4. Trespasser and Landowner . . . . . . . . . . . . . . . . . . . . . . . . . .
a. Trespassers Recovery Limited to Value Added to Land . . . . . . .
E. THIRD-PARTY CASES . . . . . . . . . . . . . . . . . . . . . . . . . . . . . .
1. Third Person Claims Lien on Chattel Afxed to Land . . . . . . . . . . .
a. UCC Rules . . . . . . . . . . . . . . . . . . . . . . . . . . . . . . . .
b. Liability for Damages Caused by Removal . . . . . . . . . . . . . . .
. . 66
. . 66
.
.
.
.
.
.
.
.
.
.
.
.
.
.
.
.
.
.
.
.
.
.
.
.
.
.
.
.
.
.
.
.
.
.
.
.
.
.
.
.
.
.
.
.
.
.
.
.
67
67
67
67
67
68
68
68
68
68
68
68
69
69
69
69
69
69
69
69
70
70
70
70
70
70
70
70
71
71
71
71
71
72
72
72
73
73
73
73
c.
3.
4.
5.
Implication . . . . . . . . . . . . . . . . . . . . . . . . . . . . .
1) Easement Implied from Existing Use (Quasi-Easement)
a) Existing Use at Time Tract Divided . . . . . . . . . .
b) Reasonable Necessity . . . . . . . . . . . . . . . . . .
c) Grant or Reservation . . . . . . . . . . . . . . . . . .
2) Easements Implied Without Any Existing Use . . . . . . .
a) Subdivision Plat . . . . . . . . . . . . . . . . . . . . .
b) Profit a Prendre . . . . . . . . . . . . . . . . . . . . .
3) Easement by Necessity . . . . . . . . . . . . . . . . . . . .
d. Prescription . . . . . . . . . . . . . . . . . . . . . . . . . . . . .
1) Open and Notorious . . . . . . . . . . . . . . . . . . . . . .
2) Adverse . . . . . . . . . . . . . . . . . . . . . . . . . . . .
3) Continuous Use . . . . . . . . . . . . . . . . . . . . . . . .
4) When Prescriptive Easements Cannot Be Acquired . . . .
Scope . . . . . . . . . . . . . . . . . . . . . . . . . . . . . . . . . . .
a. General Rules of Construction . . . . . . . . . . . . . . . . . .
b. Absence of Location . . . . . . . . . . . . . . . . . . . . . . . .
c. Changes in Use . . . . . . . . . . . . . . . . . . . . . . . . . . .
d. Easements by Necessity or Implication . . . . . . . . . . . . . .
e. Use of Servient Estate . . . . . . . . . . . . . . . . . . . . . . .
1) Duty to Repair . . . . . . . . . . . . . . . . . . . . . . . . .
f. Intended BeneficiariesSubdivision of Dominant Parcel . . . .
g. Effect of Use Outside Scope of Easement . . . . . . . . . . . . .
Termination of Easements . . . . . . . . . . . . . . . . . . . . . . .
a. Stated Conditions . . . . . . . . . . . . . . . . . . . . . . . . . .
b. Unity of Ownership . . . . . . . . . . . . . . . . . . . . . . . . .
1) Complete Unity Required . . . . . . . . . . . . . . . . . . .
2) No Revival . . . . . . . . . . . . . . . . . . . . . . . . . . .
c. Release . . . . . . . . . . . . . . . . . . . . . . . . . . . . . . .
1) Easement Appurtenant . . . . . . . . . . . . . . . . . . . .
2) Easement in Gross . . . . . . . . . . . . . . . . . . . . . .
3) Statute of Frauds . . . . . . . . . . . . . . . . . . . . . . .
d. Abandonment . . . . . . . . . . . . . . . . . . . . . . . . . . . .
1) Physical Act Required . . . . . . . . . . . . . . . . . . . .
2) Mere Words Insufficient . . . . . . . . . . . . . . . . . . .
3) Mere Nonuse Insufficient . . . . . . . . . . . . . . . . . . .
e. Estoppel . . . . . . . . . . . . . . . . . . . . . . . . . . . . . . .
f. Prescription . . . . . . . . . . . . . . . . . . . . . . . . . . . . .
g. Necessity . . . . . . . . . . . . . . . . . . . . . . . . . . . . . .
h. Condemnation . . . . . . . . . . . . . . . . . . . . . . . . . . .
i. Destruction of Servient Estate . . . . . . . . . . . . . . . . . . .
CompareLicenses . . . . . . . . . . . . . . . . . . . . . . . . . . .
a. Assignability . . . . . . . . . . . . . . . . . . . . . . . . . . . .
b. Revocation and Termination . . . . . . . . . . . . . . . . . . . .
1) Public Amusement Cases . . . . . . . . . . . . . . . . . . .
2) Breach of Contract . . . . . . . . . . . . . . . . . . . . . .
c. Failure to Create an Easement . . . . . . . . . . . . . . . . . .
d. Irrevocable Licenses . . . . . . . . . . . . . . . . . . . . . . . .
.
.
.
.
.
.
.
.
.
.
.
.
.
.
.
.
.
.
.
.
.
.
.
.
.
.
.
.
.
.
.
.
.
.
.
.
.
.
.
.
.
.
.
.
.
.
.
.
.
.
.
.
.
.
.
.
.
.
.
.
.
.
.
.
.
.
.
.
.
.
.
.
.
.
.
.
.
.
.
.
.
.
.
.
.
.
.
.
.
.
.
.
.
.
.
.
.
.
.
.
.
.
.
.
.
.
.
.
.
.
.
.
.
.
.
.
.
.
.
.
.
.
.
.
.
.
.
.
.
.
.
.
.
.
.
.
.
.
.
.
.
.
.
.
.
.
.
.
.
.
.
.
.
.
.
.
.
.
.
.
.
.
.
.
.
.
.
.
.
.
.
.
.
.
.
.
.
.
.
.
.
.
.
.
.
.
.
.
.
.
.
.
.
.
.
.
.
.
.
.
.
.
.
.
.
.
.
.
.
.
.
.
.
.
.
.
.
.
.
.
.
.
.
.
.
.
.
.
.
.
.
.
.
.
.
.
.
.
.
.
73
74
74
74
74
74
74
74
74
75
75
75
75
75
75
75
76
76
76
77
77
77
77
77
78
78
78
78
79
79
79
79
79
79
80
80
80
80
80
81
81
81
81
81
81
81
82
82
x. REAL PROPERTY
1)
2)
C.
D.
E.
Estoppel Theory . . . . . . . . . . . . . . . . . . . .
License Coupled with an Interest . . . . . . . . . .
a) Vendee of a Chattel . . . . . . . . . . . . . . .
b) Termination of Tenancy . . . . . . . . . . . . .
c) Inspection for Waste . . . . . . . . . . . . . . .
PROFITS . . . . . . . . . . . . . . . . . . . . . . . . . . . . . . .
1. Creation . . . . . . . . . . . . . . . . . . . . . . . . . . . . .
2. Alienability . . . . . . . . . . . . . . . . . . . . . . . . . . . .
3. Exclusive and Nonexclusive Prots Distinguished . . . . . .
4. Scope . . . . . . . . . . . . . . . . . . . . . . . . . . . . . . .
a. Apportionment of Profits Appurtenant . . . . . . . . . .
b. Apportionment of Profits in Gross . . . . . . . . . . . .
5. Termination . . . . . . . . . . . . . . . . . . . . . . . . . . .
COVENANTS RUNNING WITH THE LAND AT LAW (REAL
COVENANTS) . . . . . . . . . . . . . . . . . . . . . . . . . . . .
1. Requirements for Burden to Run . . . . . . . . . . . . . . .
a. Intent . . . . . . . . . . . . . . . . . . . . . . . . . . . .
b. Notice . . . . . . . . . . . . . . . . . . . . . . . . . . . .
c. Horizontal Privity . . . . . . . . . . . . . . . . . . . . .
d. Vertical Privity . . . . . . . . . . . . . . . . . . . . . . .
e. Touch and Concern . . . . . . . . . . . . . . . . . . . . .
1) Negative Covenants . . . . . . . . . . . . . . . . . .
2) Affirmative Covenants . . . . . . . . . . . . . . . .
2. Requirements for Benet to Run . . . . . . . . . . . . . . . .
a. Intent . . . . . . . . . . . . . . . . . . . . . . . . . . . .
b. Vertical Privity . . . . . . . . . . . . . . . . . . . . . . .
c. Touch and Concern . . . . . . . . . . . . . . . . . . . . .
3. Modern Status of Running of Burden and Benet . . . . . .
a. Horizontal and Vertical Privity . . . . . . . . . . . . . .
b. Touch and Concern . . . . . . . . . . . . . . . . . . . . .
4. Specic Situations Involving Real Covenants . . . . . . . . .
a. Promises to Pay Money . . . . . . . . . . . . . . . . . .
b. Covenants Not to Compete . . . . . . . . . . . . . . . . .
c. Racially Restrictive Covenants . . . . . . . . . . . . . .
5. RemediesDamages . . . . . . . . . . . . . . . . . . . . . .
6. Termination . . . . . . . . . . . . . . . . . . . . . . . . . . .
EQUITABLE SERVITUDES . . . . . . . . . . . . . . . . . . . .
1. Creation . . . . . . . . . . . . . . . . . . . . . . . . . . . . .
a. Servitudes Implied from Common Scheme . . . . . . . .
1) Common Scheme . . . . . . . . . . . . . . . . . . .
2) Notice . . . . . . . . . . . . . . . . . . . . . . . . .
2. Enforcement . . . . . . . . . . . . . . . . . . . . . . . . . . .
a. Requirements for Burden to Run . . . . . . . . . . . . .
1) Intent . . . . . . . . . . . . . . . . . . . . . . . . . .
2) Notice . . . . . . . . . . . . . . . . . . . . . . . . .
3) Touch and Concern . . . . . . . . . . . . . . . . . .
b. Requirements for Benefit to Run . . . . . . . . . . . . .
c. Privity Not Required . . . . . . . . . . . . . . . . . . . .
d. Implied Beneficiaries of CovenantsGeneral Scheme .
.
.
.
.
.
.
.
.
.
.
.
.
.
.
.
.
.
.
.
.
.
.
.
.
.
.
.
.
.
.
.
.
.
.
.
.
.
.
.
.
.
.
.
.
.
.
.
.
.
.
.
.
.
.
.
.
.
.
.
.
.
.
.
.
.
.
.
.
.
.
.
.
.
.
.
.
.
.
.
.
.
.
.
.
.
.
.
.
.
.
.
.
.
.
.
.
.
.
.
.
.
.
.
.
.
.
.
.
.
.
.
.
.
.
.
.
.
82
82
82
83
83
83
83
83
83
83
83
84
84
.
.
.
.
.
.
.
.
.
.
.
.
.
.
.
.
.
.
.
.
.
.
.
.
.
.
.
.
.
.
.
.
.
.
.
.
.
.
.
.
.
.
.
.
.
.
.
.
.
.
.
.
.
.
.
.
.
.
.
.
.
.
.
.
.
.
.
.
.
.
.
.
.
.
.
.
.
.
.
.
.
.
.
.
.
.
.
.
.
.
.
.
.
.
.
.
.
.
.
.
.
.
.
.
.
.
.
.
.
.
.
.
.
.
.
.
.
.
.
.
.
.
.
.
.
.
.
.
.
.
.
.
.
.
.
.
.
.
.
.
.
.
.
.
.
.
.
.
.
.
.
.
.
.
.
.
.
.
.
.
.
.
.
.
.
.
.
.
.
.
.
.
.
.
.
.
.
.
.
.
.
.
.
.
.
.
.
.
.
.
.
.
.
.
.
.
.
.
.
.
.
.
.
.
.
.
.
.
.
.
.
.
.
.
.
.
.
.
.
.
.
.
.
.
.
.
.
.
.
.
.
.
.
.
.
.
.
.
.
.
.
.
.
.
.
.
.
.
.
.
.
.
.
.
.
.
.
.
.
.
.
.
.
.
.
.
.
.
.
.
.
.
.
.
.
.
.
.
.
.
.
.
.
.
.
.
.
.
.
.
.
.
.
.
.
.
.
.
.
.
.
.
.
.
.
.
.
.
.
.
.
.
.
.
.
84
84
84
85
85
85
86
86
86
87
87
87
87
87
87
88
88
88
88
88
88
88
89
89
89
89
90
90
90
90
90
90
90
90
91
3.
.
.
.
.
.
.
.
.
.
.
.
.
.
.
.
.
.
.
.
.
.
.
.
.
.
.
.
.
.
.
.
.
.
.
.
.
.
.
.
.
.
.
.
.
.
.
.
.
.
.
.
.
.
.
.
.
.
.
.
.
.
.
.
.
.
.
.
.
.
.
.
.
.
.
.
.
.
.
.
.
.
.
.
.
91
91
92
92
92
92
92
93
93
93
93
93
ADVERSE POSSESSION . . . . . . . . . . . . . . . . . . . . . . . . . . . . . .
A. IN GENERAL . . . . . . . . . . . . . . . . . . . . . . . . . . . . . . . . . .
B. REQUIREMENTS . . . . . . . . . . . . . . . . . . . . . . . . . . . . . . .
1. Running of Statute . . . . . . . . . . . . . . . . . . . . . . . . . . . . .
2. Actual and Exclusive Possession . . . . . . . . . . . . . . . . . . . . .
a. Actual Possession Gives Notice . . . . . . . . . . . . . . . . . . .
1) Constructive Possession of Part . . . . . . . . . . . . . . . .
b. Exclusive PossessionNo Sharing with Owner . . . . . . . . . .
3. Open and Notorious Possession . . . . . . . . . . . . . . . . . . . . . .
4. Hostile . . . . . . . . . . . . . . . . . . . . . . . . . . . . . . . . . . .
a. If Possession Starts PermissivelyMust Communicate Hostility .
b. Co-TenantsOuster Required . . . . . . . . . . . . . . . . . . .
c. If Grantor Stays in PossessionPermission Presumed . . . . . .
d. CompareBoundary Line Agreements . . . . . . . . . . . . . .
1) Establishment Requirement . . . . . . . . . . . . . . . . . .
5. Continuous Possession . . . . . . . . . . . . . . . . . . . . . . . . . .
a. Intermittent Periods of Occupancy Not Sufficient . . . . . . . . .
b. Tacking Permitted . . . . . . . . . . . . . . . . . . . . . . . . . .
1) Privity . . . . . . . . . . . . . . . . . . . . . . . . . . . . .
2) Formalities on Transfer . . . . . . . . . . . . . . . . . . . . .
6. Payment of Property Taxes Generally Not Required . . . . . . . . . .
C. DISABILITY . . . . . . . . . . . . . . . . . . . . . . . . . . . . . . . . . .
1. Effect of DisabilitiesStatute Does Not Begin to Run . . . . . . . . .
2. No Tacking of Disabilities . . . . . . . . . . . . . . . . . . . . . . . . .
3. Maximum Tolling Periods . . . . . . . . . . . . . . . . . . . . . . . .
D. ADVERSE POSSESSION AND FUTURE INTERESTS . . . . . . . . . .
1. Possibility of ReverterStatute of Limitations Runs on Happening of
Event . . . . . . . . . . . . . . . . . . . . . . . . . . . . . . . . . . . .
2. Right of EntryHappening of Event Does Not Trigger Statute of
Limitations . . . . . . . . . . . . . . . . . . . . . . . . . . . . . . . . .
a. Grantor Must Act Within Reasonable Time to Avoid Laches . . .
E. EFFECT OF COVENANTS IN TRUE OWNERS DEED . . . . . . . . .
F. LAND THAT CANNOT BE ADVERSELY POSSESSED . . . . . . . . .
.
.
.
.
.
.
.
.
.
.
.
.
.
.
.
.
.
.
.
.
.
.
.
.
.
.
.
.
.
.
.
.
.
.
.
.
.
.
.
.
.
.
.
.
.
.
.
.
.
.
.
.
.
.
.
.
.
.
.
.
.
.
.
.
.
.
.
.
.
.
.
.
.
.
.
.
.
.
.
.
.
.
.
.
.
.
.
.
.
.
.
.
.
.
.
.
.
.
.
.
.
.
.
.
93
93
94
94
94
94
94
94
94
95
95
95
95
95
96
96
96
96
96
96
96
97
97
97
97
97
. . . . 98
.
.
.
.
.
.
.
.
.
.
.
.
.
.
.
.
98
98
98
98
VI.
CONVEYANCING . . . . . . . . . . . . . . . . . . . . . . . . . . . . . . . . . . . .
A. LAND SALE CONTRACTS . . . . . . . . . . . . . . . . . . . . . . . . . . . .
1. Statute of Frauds Applicable . . . . . . . . . . . . . . . . . . . . . . . . .
a. Doctrine of Part Performance . . . . . . . . . . . . . . . . . . . . . .
1) Theories to Support the Doctrine . . . . . . . . . . . . . . . . .
a) Evidentiary Theory . . . . . . . . . . . . . . . . . . . . . .
b) Hardship or Estoppel Theory . . . . . . . . . . . . . . . . .
2) Acts of Part Performance . . . . . . . . . . . . . . . . . . . . . .
3) Can Seller Obtain Specific Performance Based on Buyers Acts?
a) Evidentiary Theory . . . . . . . . . . . . . . . . . . . . . .
b) Hardship or Estoppel Theory . . . . . . . . . . . . . . . . .
2. Doctrine of Equitable Conversion . . . . . . . . . . . . . . . . . . . . . .
a. Risk of Loss . . . . . . . . . . . . . . . . . . . . . . . . . . . . . . . .
1) Casualty Insurance . . . . . . . . . . . . . . . . . . . . . . . . .
b. Passage of Title on Death . . . . . . . . . . . . . . . . . . . . . . . .
1) Death of Seller . . . . . . . . . . . . . . . . . . . . . . . . . . . .
2) Death of Buyer . . . . . . . . . . . . . . . . . . . . . . . . . . .
3. Marketable Title . . . . . . . . . . . . . . . . . . . . . . . . . . . . . . . .
a. Marketability DefinedTitle Reasonably Free from Doubt . . . .
1) Defects in Record Chain of Title . . . . . . . . . . . . . . . . . .
a) Adverse Possession . . . . . . . . . . . . . . . . . . . . . . .
b) Future Interest Held by Unborn or Unascertained Parties .
2) Encumbrances . . . . . . . . . . . . . . . . . . . . . . . . . . . .
a) Mortgages and Liens . . . . . . . . . . . . . . . . . . . . .
b) Easements . . . . . . . . . . . . . . . . . . . . . . . . . . .
c) Covenants . . . . . . . . . . . . . . . . . . . . . . . . . . .
d) Encroachments . . . . . . . . . . . . . . . . . . . . . . . . .
3) Zoning Restrictions . . . . . . . . . . . . . . . . . . . . . . . . .
4) Waiver . . . . . . . . . . . . . . . . . . . . . . . . . . . . . . . .
b. Quitclaim DeedNo Effect . . . . . . . . . . . . . . . . . . . . . . .
c. Time of Marketability . . . . . . . . . . . . . . . . . . . . . . . . . .
1) Installment Land Contract . . . . . . . . . . . . . . . . . . . . .
d. Remedy If Title Not Marketable . . . . . . . . . . . . . . . . . . . . .
1) Rescission, Damages, Specific Performance . . . . . . . . . . . .
2) Merger . . . . . . . . . . . . . . . . . . . . . . . . . . . . . . . .
4. Time of Performance . . . . . . . . . . . . . . . . . . . . . . . . . . . . .
a. PresumptionTime Not of the Essence . . . . . . . . . . . . . . . . .
b. When Presumption Overcome . . . . . . . . . . . . . . . . . . . . . .
c. Effect of Time of the Essence Construction . . . . . . . . . . . . . . .
d. Liability When Time Not of the Essence . . . . . . . . . . . . . . . .
5. Tender of Performance . . . . . . . . . . . . . . . . . . . . . . . . . . . .
a. When Partys Tender Excused . . . . . . . . . . . . . . . . . . . . . .
b. Neither Party Tenders Performance . . . . . . . . . . . . . . . . . .
c. Buyer Finds Sellers Title Unmarketable . . . . . . . . . . . . . . . .
6. Remedies for Breach of the Sales Contract . . . . . . . . . . . . . . . . .
a. Damages . . . . . . . . . . . . . . . . . . . . . . . . . . . . . . . . . .
1) Liquidated Damages . . . . . . . . . . . . . . . . . . . . . . . .
b. Specific Performance . . . . . . . . . . . . . . . . . . . . . . . . . . .
.
.
.
.
.
.
.
.
.
.
.
.
.
.
.
.
.
.
.
.
.
.
.
.
.
.
.
.
.
.
.
.
.
.
.
.
.
.
.
.
.
.
.
.
.
.
.
.
. 98
. 98
. 98
. 99
. 99
. 99
. 99
. 99
. 99
. 99
. 99
100
100
100
100
100
100
101
101
101
101
101
102
102
102
102
102
103
103
103
103
103
103
103
103
104
104
104
104
104
104
104
104
104
105
105
105
105
B.
C.
1) Buyers Remedy . . . . . . . . . . . . . . . . . . . . . . . . . . .
2) Sellers Remedy . . . . . . . . . . . . . . . . . . . . . . . . . . .
c. Special Rules for Unmarketable Title . . . . . . . . . . . . . . . . . .
7. Sellers Liability for Defects on Property . . . . . . . . . . . . . . . . . .
a. Warranty of Fitness or QualityNew Construction Only . . . . . .
b. Negligence of Builder . . . . . . . . . . . . . . . . . . . . . . . . . . .
c. Liability for Sale of Existing Land and Buildings . . . . . . . . . . .
1) Misrepresentation (Fraud) . . . . . . . . . . . . . . . . . . . . .
2) Active Concealment . . . . . . . . . . . . . . . . . . . . . . . . .
3) Failure to Disclose . . . . . . . . . . . . . . . . . . . . . . . . . .
d. Disclaimers of Liability . . . . . . . . . . . . . . . . . . . . . . . . .
1) As Is Clauses . . . . . . . . . . . . . . . . . . . . . . . . . . .
2) Specific Disclaimers . . . . . . . . . . . . . . . . . . . . . . . . .
8. Real Estate Brokers . . . . . . . . . . . . . . . . . . . . . . . . . . . . . .
9. Title Insurance . . . . . . . . . . . . . . . . . . . . . . . . . . . . . . . . .
DEEDSFORM AND CONTENT . . . . . . . . . . . . . . . . . . . . . . . .
1. Formalities . . . . . . . . . . . . . . . . . . . . . . . . . . . . . . . . . . .
a. Statute of Frauds . . . . . . . . . . . . . . . . . . . . . . . . . . . . .
b. Description of Land and Parties . . . . . . . . . . . . . . . . . . . . .
c. Words of Intent . . . . . . . . . . . . . . . . . . . . . . . . . . . . . .
d. Consideration Not Required . . . . . . . . . . . . . . . . . . . . . . .
e. Seal Is Unnecessary . . . . . . . . . . . . . . . . . . . . . . . . . . .
f. Attestation and Acknowledgment Generally Unnecessary . . . . . .
g. Signature . . . . . . . . . . . . . . . . . . . . . . . . . . . . . . . . .
2. Defective Deeds and Fraudulent Conveyances . . . . . . . . . . . . . . .
a. Void and Voidable Deeds . . . . . . . . . . . . . . . . . . . . . . . . .
1) Void Deeds . . . . . . . . . . . . . . . . . . . . . . . . . . . . . .
2) Voidable Deeds . . . . . . . . . . . . . . . . . . . . . . . . . . .
b. Fraudulent Conveyances . . . . . . . . . . . . . . . . . . . . . . . . .
3. Description of Land Conveyed . . . . . . . . . . . . . . . . . . . . . . . .
a. Sufficient Description Provides a Good Lead . . . . . . . . . . . . .
b. Insufficient DescriptionTitle Remains in Grantor . . . . . . . . . .
c. Parol Evidence Admissible to Clear Up Ambiguity . . . . . . . . . .
1) CompareInadequate Description . . . . . . . . . . . . . . . .
d. Rules of Construction . . . . . . . . . . . . . . . . . . . . . . . . . .
e. Land Bounded by Right-of-Way . . . . . . . . . . . . . . . . . . . . .
1) Title Presumed to Extend to Center of Right-of-Way . . . . . . .
a) Evidence to Rebut Presumption . . . . . . . . . . . . . . .
b) Measuring from Monument . . . . . . . . . . . . . . . . . .
2) Variable Boundary Line Cases . . . . . . . . . . . . . . . . . . .
a) Slow Change in Course Changes Property Rights . . . . . .
b) Avulsion Does Not Change Property Rights . . . . . . . . .
c) Encroachment of Water Does Not Change Fixed Boundary
Lines . . . . . . . . . . . . . . . . . . . . . . . . . . . . . .
f. Reformation of Deeds . . . . . . . . . . . . . . . . . . . . . . . . . .
DELIVERY AND ACCEPTANCE . . . . . . . . . . . . . . . . . . . . . . . .
1. DeliveryIn General . . . . . . . . . . . . . . . . . . . . . . . . . . . . .
a. Manual Delivery . . . . . . . . . . . . . . . . . . . . . . . . . . . . .
.
.
.
.
.
.
.
.
.
.
.
.
.
.
.
.
.
.
.
.
.
.
.
.
.
.
.
.
.
.
.
.
.
.
.
.
.
.
.
.
.
.
105
105
105
105
105
106
106
106
106
106
107
107
107
107
107
107
107
107
107
108
108
108
108
108
108
108
108
109
109
109
109
109
109
110
110
110
110
110
111
111
111
111
.
.
.
.
.
111
111
111
111
112
b.
c.
d.
D.
.
.
.
.
.
.
.
.
.
.
.
.
.
.
.
.
.
.
.
.
.
.
.
.
.
.
.
.
.
.
.
.
.
.
.
.
.
.
.
.
.
.
.
.
.
.
.
.
112
112
112
112
113
113
113
113
113
114
114
114
114
114
114
114
114
115
115
115
115
115
116
116
116
116
116
116
117
117
117
117
117
117
117
118
118
118
118
118
118
118
118
119
119
119
119
119
2)
3)
E.
. . 119
.
.
.
.
.
.
.
.
.
.
.
.
.
.
.
.
.
.
.
.
.
.
.
.
.
.
.
.
.
.
.
.
120
120
120
120
120
121
121
121
121
121
121
122
122
122
122
122
.
.
.
.
.
.
.
.
.
.
.
.
.
.
.
.
.
.
.
.
.
.
.
.
.
.
.
.
.
.
.
.
.
.
.
.
.
.
.
.
.
.
.
.
.
.
.
.
.
.
.
.
.
.
.
.
.
.
122
122
122
122
123
123
123
123
124
124
124
124
124
125
125
125
126
126
126
126
127
127
127
127
127
128
128
128
129
e)
F.
VII.
.
.
.
.
.
.
.
.
.
.
.
.
.
.
.
.
.
.
.
.
.
.
.
.
.
.
.
.
.
.
.
.
.
.
.
.
.
.
.
.
.
.
.
.
.
.
.
.
.
.
.
.
.
.
.
.
.
.
.
.
.
.
.
.
.
.
.
.
.
.
.
.
.
.
.
.
.
.
.
.
.
.
.
.
.
.
.
.
.
.
.
.
.
.
.
.
.
.
.
.
.
.
.
.
.
.
.
.
.
.
.
.
.
.
.
.
.
.
.
.
.
.
.
.
.
.
.
.
.
.
.
.
.
.
.
.
.
.
.
.
.
.
.
.
.
.
.
.
.
.
.
.
.
.
.
.
.
.
.
.
.
.
.
.
.
.
.
.
129
129
129
129
130
130
130
130
130
131
131
131
131
131
131
133
133
133
133
133
133
133
134
134
134
134
134
135
135
135
135
135
135
136
136
136
136
136
136
136
137
137
137
137
137
137
B.
C.
D.
E.
F.
.
.
.
.
.
.
.
.
.
.
.
.
.
.
.
.
.
.
.
.
.
.
.
.
.
.
.
.
.
.
.
.
.
.
.
.
.
.
.
.
.
.
.
.
.
.
.
.
138
138
138
138
138
138
139
139
139
139
139
140
140
140
140
140
140
140
141
141
141
141
141
141
141
141
142
142
142
142
142
142
143
143
143
143
143
143
143
144
144
144
144
144
145
145
146
146
2.
3.
4.
5.
Restitution . . . . . .
Treat as a Mortgage .
Waiver . . . . . . . .
Election of Remedies
.
.
.
.
.
.
.
.
.
.
.
.
.
.
.
.
.
.
.
.
.
.
.
.
.
.
.
.
.
.
.
.
.
.
.
.
.
.
.
.
.
.
.
.
.
.
.
.
.
.
.
.
.
.
.
.
.
.
.
.
.
.
.
.
.
.
.
.
.
.
.
.
.
.
.
.
.
.
.
.
.
.
.
.
.
.
.
.
.
.
.
.
.
.
.
.
.
.
.
.
.
.
.
.
.
.
.
.
.
.
.
.
.
.
.
.
.
.
.
.
147
147
147
147
.
.
.
.
.
.
.
.
.
.
.
.
.
.
.
.
.
.
.
.
.
.
.
.
.
.
.
.
.
.
.
.
.
.
.
.
.
.
.
.
.
.
.
.
.
.
.
.
.
.
.
.
.
.
.
.
.
.
.
.
.
.
.
.
.
.
.
.
.
.
.
.
.
.
.
.
.
.
.
.
.
.
.
.
.
.
.
.
.
.
.
.
.
.
.
.
.
.
.
.
.
.
.
.
.
.
.
.
.
.
.
.
.
.
.
.
.
.
.
.
.
.
.
.
.
.
.
.
.
147
147
147
147
147
148
148
148
148
148
148
148
148
149
149
149
149
149
149
149
150
150
150
150
150
150
151
151
151
151
151
151
151
152
152
152
152
152
152
152
152
152
152
IX.
.
.
.
.
.
.
.
.
.
.
.
.
.
.
.
.
.
.
.
.
.
.
.
.
.
.
.
.
.
.
.
.
.
.
.
.
.
.
.
.
.
.
.
.
.
.
.
.
.
.
.
.
.
.
.
.
.
.
.
.
.
.
.
.
.
.
.
.
.
.
.
.
.
.
.
.
.
.
.
.
.
.
.
.
.
.
.
.
.
.
.
.
.
.
.
.
.
.
.
.
.
.
.
.
.
.
.
.
.
.
.
.
.
.
.
.
.
.
.
.
.
.
.
.
.
.
.
.
.
.
.
.
.
.
.
.
.
.
.
.
.
.
.
.
.
.
.
.
.
.
.
.
.
.
.
.
.
.
.
.
.
.
.
.
.
.
.
.
.
.
.
.
.
.
.
.
.
.
.
.
.
.
.
.
.
.
.
.
.
153
153
153
153
153
153
153
153
153
153
154
154
154
154
154
154
155
155
155
155
155
REAL PROPERTY 1.
REAL PROPERTY
I. ESTATES IN LAND
A.
IN GENERAL
Estates in land are possessory interests in land. These interests may be presently possessory
(present estates), or they may become possessory in the future (future interests). They may be
freeholds, which give possession under some legal title or right to hold (e.g., fees or life estates),
or they may be nonfreeholds, which give mere possession (i.e., leases). Estates in land may be
of potentially innite duration, as in the case of a fee simple, or they may be of limited duration,
as in the case of an estate for years. But whatever their characteristics, estates in land must be
distinguished from nonpossessory interests such as easements, prots, covenants, and servitudes.
This section of the outline will examine various estates in land. It divides the interests into two
classes: present interests and future interests. However, some future interests (those following
defeasible fees) will be considered with the present interests to which they are attached.
B.
2.
Defeasible Fees
Defeasible fees are fee simple estates of potentially innite duration that can be terminated
by the happening of a specied event. Because defeasible fees can result in forfeitures, courts
will construe, where possible, a purported limitation as a mere declaration of the grantors
purpose or motive for making the grant (i.e., as precatory language). (See b.1)a), infra.)
a.
2. REAL PROPERTY
may last forever if no one ever quaffs a brew. If A conveys his fee simple
determinable estate to B, B will own the for so long as estate. If A
does not convey his estate, on As death it will pass by will or intestacy
to his successors, and so on. If, however, someone ever consumes an
alcoholic beverage on the premises, the estate will automatically come to
an end according to its own terms; and O will immediately and automatically become the owner of the fee simple, without taking any steps to
terminate As interest.
1)
2)
b.
b)
REAL PROPERTY 3.
1)
b)
c)
2)
4. REAL PROPERTY
3)
c.
REAL PROPERTY 5.
e.
Restraints on Marriage
If the purpose of the condition or limitation is to penalize marriage, it likely
will be struck down. On the other hand, if the purpose is to give support until
marriage, when the new spouses obligation of support arises, the condition or
limitation generally is upheld.
Example:
O conveys land to A, but if she marries, to B. Absent any
evidence as to Os motive, the condition subsequent will be struck
down, leaving A with a fee simple absolute.
Compare:
2)
3.
Fee Tail
The fee tail, typically created by the words to A and the heirs of his body, limited inheritance to lineal descendants of the grantee. If no lineal descendants survived at the grantees
death, the property either reverted to the grantor or her successors or passed to a designated
remainderman. Today, most United States jurisdictions have abolished the fee tail and have
enacted statutes under which any attempt to create a fee tail results in the creation of a fee
simple.
4.
Life Estate
An estate for life is an estate that is not terminable at any xed or computable period of time,
6. REAL PROPERTY
but cannot last longer than the life or lives of one or more persons. It may arise by operation
of law or may be created by an act or agreement of the parties.
a.
b.
REAL PROPERTY 7.
2)
c.
Inheritability
At common law, if A died before B, the property was regarded as without an
owner until B died. Today, statutes provide that such estates are devisable and
inheritable if no special occupant is named in the original grant. (A special
occupant is a person named by the grantor to take the balance of the term, if
any.)
(ii) When the life tenant is expressly given the right to exploit such resources in
the grant;
(iii) When prior to the grant, the land was used in exploitation of such natural
resources, so that in granting the life estate the grantor most likely intended
the life tenant to have the right to exploit (but see open mines doctrine,
below); and
(iv) In many states, where the land is suitable only for such exploitation (e.g., a
mine).
Note: There is a vague reasonableness limit on the amount of oil or coal a life
tenant can remove from the property.
a)
8. REAL PROPERTY
2)
Permissive Waste
Absent a contrary provision in the instrument creating the life estate, a life tenant
has a duty to make repairs to the property to keep it from being damaged by the
weather, and to pay certain carrying charges (e.g., mortgage interest, property
taxes, and special assessments for public improvements). However, this duty is
limited to the extent of the income or prots derived from the land (or if there is
no actual income or prot, to the extent of the reasonable rental value of the land).
Failure to make required repairs or pay required carrying charges constitutes
permissive waste. A future interest holder who expends funds in satisfaction of
the life tenants obligations (e.g., pays the property taxes to avoid a tax foreclosure
sale) is entitled to reimbursement.
a)
Obligation to Repair
A life tenant is obligated to preserve the land and structures in a reasonable
state of repair (to the limited extent stated above). But the tenant is under no
obligation to make permanent improvements on the land, no matter how wise
it might seem to do so.
b)
c)
d)
e)
REAL PROPERTY 9.
f)
3)
Ameliorative Waste
Ameliorative waste consists of acts that economically benet the property.
Ameliorative waste occurs when the use of the property is substantially changed,
but the change increases the value of the property. At common law, any change
to existing buildings or other improvements was always actionable waste, even if
it improved the value of the property. Under modern authorities, however, a life
tenant can substantially alter or even demolish existing buildings if:
(i)
The market value of the future (or other nonpossessory) interests is not
diminished; and either
d.
a)
CompareLeasehold Tenant
Leasehold tenants are treated differently from life tenants. Most leasehold
tenants remain liable for ameliorative waste even if the neighborhood has
changed and the market value of the premises is increased. (See II.C.1.a.3),
infra.)
b)
CompareWorthless Property
Under modern authority, a life tenant may ask for a judicial sale in a partition
proceeding if it appears that the land is practically worthless in its present
state. The proceeds are put in trust with income to the life tenant.
because owning it would be burdensome. If this occurs, the courts generally accelerate
the future interest that follows the life estate, allowing it to become possessory immediately.
5.
C.
FUTURE INTERESTS
A future interest is an estate that does not entitle the owner thereof to possession immediately,
but will or may give the owner possession in the future. A future interest is a present, legally
protected right in property; it is not an expectancy.
Examples:
1) O conveys land to A for life, and on As death to B in fee simple. A has a
present possessory life estate. B has a future interest. (Bs future interest is an
indefeasibly vested remainder.) Upon the termination of As possessory life estate,
Bs remainder in fee simple will become a present possessory estate in fee simple.
2) O conveys land to A for life, and on As death to B in fee simple if B survives
A. A has a present possessory life estate. B has a future interest. (It is a contingent
remainder.) Upon the termination of As life estate, Bs remainder in fee simple
may become a present possessory estate in fee simple. B must survive A in order
to take. (In this example, O also has a future interest. He has not conveyed away
the interest represented by the contingency that B may predecease A. If B does
predecease A, on the termination of As life estate title to the land will revert to O.
Os retained future interest is called a reversion.)
3) After the conveyance to A for life, and on As death to B, B can transfer his
remainder interest to another person. Alternatively, if B dies during As lifetime,
his vested remainder will pass to the devisees under his will or (if B left no will) to
his intestate heirs.
1.
b.
Reversions
A person owning an estate in real property can create and transfer a lesser estate (in the
durational sense). The residue left in the grantor, which arises by operation of law, is a
reversion.
Examples:
1) O, owning land in fee simple, conveys it (i) to A for life, or (ii) to
A for 99 years. In each case, O has a reversion in fee simple. She (or her
successors) will be entitled to present possession of the land when the
granted estate terminates.
2) O, owning a life estate in land, leases it to A for 20 years. O has a
reversion in a life estate. If O is still alive when As lease expires, title
will revert to O for life. What happens if, 10 years after this transfer,
O dies? As lease will come to an end, for he was given a lease by one
holding only a life estate. O cannot convey a greater interest than she
has.
3) O, owning land in fee simple, conveys it to A for life, and on As
death to B if B survives A. A has a life estate, B has a contingent
remainder, and O has a reversion that will take in present possession at
As death if B predeceases A.
Reversions are transferable, devisable by will, and descendible by inheritance. The
holder of a reversion may sue a possessory owner for waste and may recover against
third-party wrongdoers for damages to the property (to the extent of the injury to the
reversion).
c.
2.
Remainders
A remainder is a future interest created in a transferee that is capable of taking in present
possession and enjoyment (i.e., capable of becoming a present interest) upon the natural
termination of the preceding estates created in the same disposition. Unlike a reversion,
which arises by operation of law from the fact that the transferor has not made a complete
disposition of his interest, a remainder must be expressly created in the instrument creating
the intermediate possessory estate. At common law, the only preceding estates that could
support a remainder were life estates and fee tails. Because nearly all American jurisdictions
have abolished the fee tail estate, a safe rule of thumb is that remainders always follow life
estates. (Note: According to the Restatement of Property, under modern law, a remainder
can also follow a term of years. However, there is very little case law on the point, and it is so
rare that it is extremely unlikely to be tested.)
Examples:
1) To A for life, and on As death to B and his heirs. A has a present possessory life estate. B has a remainder in fee simple. It is a remainder because
upon the expiration of As life estate (natural termination of the preceding
estate), B will be entitled to present possession and enjoyment of the property.
The term remainder derives from the consequence that when As life estate
comes to an end, title remains away from the transferor instead of reverting
back to him.
2) On Monday, O conveys Blackacre to A for life. On Wednesday, O
conveys all of my right, title, and interest in Blackacre to B. B holds a reversion, not a remainder. Bs future interest was not created in the same disposition that gave A a life estate. The Monday conveyance gave A a life estate and
raised a reversion in O. The Wednesday conveyance transferred Os reversion
to B. Once a reversion, always a reversion.
A remainder cannot cut short or divest a preceding estate prior to its normal expiration. Therefore, a remainder can never follow a fee simple, which has a potentially innite
duration. Future interests that cut short a preceding estate or follow a gap after it are called
executory interests. (See 3., infra.)
a.
(ii) Must be certain to become possessory on termination of the prior estates (i.e.,
there is no condition that may operate to prevent the remainder from someday
becoming a present interest);
(iii) Must not be subject to being defeated or divested (compare the vested remainder
subject to total divestment, c., infra); and
(iv) Must not be subject to being diminished in size (compare the vested remainder
subject to open, b., infra).
Examples:
b.
in fee in the transferor, which will take in possession if A never has any
children.
Suppose two years later a child, Bob, is born to A. The state of title
is: life estate in A, vested remainder subject to open in Bob. Bobs
remainder is vested because he is in existence and ascertained and his
taking is not subject to any contingency. But it is vested subject to open
because A may have more children.
Two years later another child, Ray, is born to A. Bobs remainder has
been partially divested in favor of Ray, who also meets the description children of A. Bob and Ray now hold the vested remainder as
tenants in common (each with an undivided one-half share) subject to
openi.e., their vested remainders will be partially divested if more
children are born to A.
Two years later Bob dies; shortly thereafter, A dies. Bobs successors (by
will or intestacy) and Ray are entitled to present possession and enjoyment of the property. Bobs share of the remainder was subject to partial
divestment, but it was not subject to being totally defeated. No condition
of survival was attached to Bobs interest. Bob (or his successors) was
certain to take; the only question was the size of his share.
2) Gift by will to my wife, Rowena, for life, and on her death to
my children in equal shares. T is survived by Rowena and by three
children. At rst blush this looks like a vested remainder subject to open
because it is a remainder to someones children. In reality, though, it is
indefeasibly vested. T, being dead, can have no more children. (Slight
qualication of answer: If Rowena is pregnant with Ts child at Ts
death, the posthumous child, if born alive, will share in the gift.)
c.
1)
2)
his right to possession and enjoyment is subject to being defeated by the happening of
some condition subsequent.
Examples:
1) To A for life, remainder to B and his heirs, but if at Bs death he
is not survived by issue, to C and his heirs. Here, B has a vested
remainder in fee simple, but his fee simple interest is subject to being
divested if at his death he is not survived by issue. (C has a shifting
executory interest.)
2) To A for life, then to B for life. A has a life estate. B has a vested
remainder in a life estate subject to total divestment. The transferor has
a reversion in fee. Bs remainder is vested even though (as a practical
matter) he must survive A in order to take. But this practical requirement does not make Bs remainder contingent. The only condition to
Bs taking is the natural termination of As life estate, and this condition is inherent in any remainder life estate. There is no other condition
precedent. However, Bs remainder life estate is not indefeasibly vested,
for it will be defeated if he dies in As lifetime. Therefore, it is a vested
remainder subject to total divestment.
3) To A for life, and on As death to B; but if B predeceases A, on As
death to C. A has a life estate. B has a vested remainder subject to total
divestment. Although Bs taking is contingent on his surviving A, that
contingency is expressed as a condition subsequentmeaning that Bs
remainder is vested subject to total divestment. (C has a shifting executory interest.)
d.
Contingent Remainder
There are two ways to create a contingent remainder.
1)
2)
3)
Examples:
1) O conveyed to A for life, then to the heirs of B. If A predeceased B, there were no heirs of B to take possession so the
remainder was destroyed and O or his estate retook possession.
2) O conveyed to A for life, then to B if she reaches age 21. If A
died before B reached 21, the remainder was destroyed. (Note that
whenever a grantor created a contingent remainder, he retained a
reversion, which was normally defeasible.)
Analysis: Why are the interests in the above examples remainders
and not executory interests? In each instance it is possible that the
future interest will take effect at the natural expiration of As life
estate (as remainders do), or that it will take effect following a gap
after As estate (as executory interests do). The rule is that if an
interest may operate as either a remainder or an executory interest
(depending on the circumstances at As death), it is a remainder.
Thus, at common law, if such a remainder was then called upon to
act as an executory interest, it could not do so and was destroyed.
a)
Rule Abolished
Today, the rule of destructibility has been abolished in all but a few states.
Thus, in the two examples given above, on As death, Os reversion would
take over, and would then give way to a springing executory interest on Bs
death in the rst example, and on Bs attaining age 21 in the second. (Note
that the contingent remainders are not destroyed when the preceding estate
ends, but instead become executory interests because they will divest the
transferors estate.)
b)
Note: This Rule has been abolished in most states today, but arises occasionally where
a conveyance was executed prior to abolition of the Rule.
f.
3.
Executory Interests
Here is a good shorthand rule for classifying executory interests. Remember that there are
two and only two future interests that can be created in a transferee: remainders and executory interests. If it is not a remainder because the preceding estate is not a life estate, then
it must be an executory interest. Thus, an executory interest is any future interest in a transferee that does not have the characteristics of a remainder, i.e., it is not capable of taking on
the natural termination of the preceding life estate. More specically, an executory interest is
an interest that divests the interest of another.
a.
c.
d.
4.
6.
b.
c.
d.
e.
Class Gifts
A class is a group of persons having a common characteristic. Typically, they stand in the
same relation to each other or to some other person (e.g., children, grandchildren, descendants, nephews and nieces). In a gift to a class, the share of each member of the class is
determined by the number of persons in the class.
a.
b.
Denitional Problems
1)
Dispositions to Children
A gift to a persons children generally includes that persons children from all
marriages as well as adopted and nonmarital children. That persons stepchildren
and grandchildren are generally not included in the class.
2)
Dispositions to Heirs
A disposition to the heirs of someone presumptively includes those persons who
would take the named persons estate according to the laws of descent and distribution if she were to die without a will.
3)
4)
2)
3)
one-third. If B lives to attain age 21, but C dies before attaining that
age, on Cs death, A and Bs shares will be increased to one-half.
Two years after Ts death, another child (D) is born to John. D is
excluded by the rule of convenience. The class was closed at Ts
death in order to determine the minimum size of As share so that
this share could be distributed to her.
Suppose none of Johns children is 21 at Ts death. The class
remains open until a child of John reaches the designated age, at
which time the class closes.
2) Ts will devises his residuary estate to Wanda for life, and on
Wandas death to such of Johns children as live to attain the age
of 21. Here, the class will close, and the remaindermen who share
in the disposition will be determined, when two things occur: (i)
Wanda dies; and (ii) a child of John reaches age 21. If at Ts death
one of Johns children is over age 21, it does not matter; the class
remains open until Wandas life estate terminates. Likewise, if
at Wandas death no child of John has attained age 21, the class
remains open until one of Johns children reaches that age. If at
Wandas death a child has attained age 21, the class will close at
that time.
4)
7.
Survival
As a general rule, all future interests can pass at death by will or inheritance; i.e., they are
descendible and devisable. This is true unless the interests taking is subject to an expressed
or implied contingency of survival.
Examples:
1) Ts will devises his residuary estate to my sister Sue for life, and on Sues
death to her children in equal shares. At the time of Ts death, Sue has
three children: A, B, and C. C dies, then Sue dies survived by A and B. The
remainder is shared by A, B, and the estate of C (i.e., the estate takes under
Cs will or by intestacy), each with one-third shares. Analysis: A, B, and C
had vested remainders subject to open, but their interests were not in terms
conditioned on surviving the life beneciaryand the law does not imply
such a condition of survival. On Cs death, his vested remainder subject to
open passes via his will or by intestacy. (Note: This example does not invoke
the lapsed gift doctrine of Wills law, for C was alive at the testators death.)
2) To A for life, and on As death to B if B is then living; but if B is not
then living, to C. C dies, then B dies, then A dies. Who takes? Answer: The
takers under Cs will or by intestacy. B and C were given alternative contingent remainders. Bs remainder was contingent on his surviving A, and B did
not meet the condition; his estate was defeated. Cs remainder was contingent
on Bs not surviving A; it was not in terms contingent on Cs surviving A, and
the law does not imply such a condition.
a.
b.
D.
TRUSTS
An express trust involves the holding of title to property by a trustee, who has an equitable
duciary duty to deal with it for the benet of other persons (the beneciaries).
1.
Settlor
The settlor is the person who creates the trust by manifesting an intent to do so. While
a trust of personal property may be expressed orally, the Statute of Frauds requires a
writing to create a trust of real property. The settlor must own the property at the time
the trust is created and must intend to make the trust effective immediately.
b.
Trustee
The trustee holds legal title to the property, but must act under the instructions of the
settlor who created the trust. The trustee has a duciary duty to use the highest care and
skill for the beneciaries. If the trustee has no duties at all, the trust will fail, and legal
title will vest immediately in the beneciaries. However, if the trustee dies, resigns, or
refuses to serve, the trust will not fail; a court of equity will appoint a substitute trustee.
c.
Beneciaries
The beneciaries are the persons for whose benet the trust is created and held;
they hold equitable title to the property. Every private trust must have at least one
beneciary, and the beneciaries must be denitely identiable by the time their
interest comes into enjoyment and, in all events, within the period of the Rule Against
Perpetuities. Acceptance of the benets of the trust is normally presumed, but a beneciary may renounce his rights under the trust within a reasonable time after learning of
its creation. A trust may be for a class of beneciaries (e.g., all the living descendants
of Mary Jones), provided that the class is small enough to be reasonably denite.
2.
3.
d.
Res
The res is the property that is the subject of the trust. If there is no res, the trust fails.
The res may be real property or personal property (tangible or intangible), and it may
be either a present interest or a future interest (vested or contingent). The trust res must
be segregated from other property of the settlor, but this does not preclude a trust of a
fractional share interest, such as a trust of an undivided one-half interest in Blackacre,
where the settlor owns all of Blackacre.
e.
Creation of Trusts
a.
b.
c.
Testamentary Conveyance
The settlor may create the trust by language in his will, and may also transfer the res
to the trustee by a devise in the will. The trust will come into existence only upon the
death of the settlor.
d.
Charitable Trusts
a.
Beneciaries
A charitable trust, unlike the private trusts described above, must have an indenite
group of beneciaries. The beneciaries must be reasonably numerous and not individually identied. The trust may be for the benet of an established charity (e.g., the
American Red Cross) or for a group of persons (e.g., the victims of Hurricane Sandy).
b.
the Rule Against Perpetuities. Note, however, that if either the rst or second interest is
noncharitable, the exemption from the Rule Against Perpetuities does not apply and the
second gift is void.
Examples:
1) O conveys land to T in trust for the benet of the victims of
Hurricane Sandy, and when all houses destroyed by the hurricane have
been rebuilt, then for the benet of the American Red Cross. The
interest of the Red Cross may not vest until more than 21 years after the
death of any person living when the trust is created, but it is still a valid
interest.
2) O conveys land to T in trust for the benet of my son John, whose
house was destroyed by Hurricane Sandy, and when his house has been
rebuilt, then for the benet of the American Red Cross. The interest of
the Red Cross may not vest until more than 21 years after the death of
any person living when the trust is created, and it is void.
E.
c.
Cy Pres Doctrine
If the purposes of a charitable trust are impossible to fulll, are illegal, or have been
completely fullled, a court may redirect the trust to a different purpose that is as near
as may be (a translation of the Latin cy pres) to the settlors original intent.
d.
Contingent remainders;
1.
b.
2)
3)
4)
Must Vest
To be valid under the Rule, it must be shown that the interest created in the transferee must vest, regardless of what might happen, within lives in being plus 21 years.
An interest becomes vested for purposes of the Rule when: (i) it becomes a present
possessory estate, or (ii) it becomes an indefeasibly vested remainder or a vested
remainder subject to total divestment. Remember that the Rule is applicable only to
future interests created in third persons; consequently, the Rule generally applies only to
contingent remainders, executory interests, and vested remainders subject to open.
Examples:
1) To A for life, then to As children for their lives, and on the death
of the last survivor of As children, to B in fee simple. At the time of
this disposition, A has two very young children and is quite capable of
having more children in the future. (i) A has a present possessory life
estate. As present children have vested remainders in life estates that
are subject to open in favor of any future children born to A. There is
a contingent remainder in a life estate in As unborn childrenbut this
interest is valid under the Rule because the childrens life estates will
vest at their birth, which will be in As lifetime. B has an indefeasibly
vested remainder in fee simple. (ii) Bs interest is valid under the Rule
even though it may be years before B (or her successors) is entitled to
present possession and enjoyment of the property, and even though B
(or her successors) may not succeed to present possession and enjoyment until the death of some person not now in being (i.e., a future-born
child of A may be the last survivor of As children). Despite all this,
c.
If at All
This simply means that the interest does not have to vest within the perpetuities period
in order to be valid; after all, many contingent remainders never vest because the condition precedent to their taking is not satised.
Examples:
1) To A for life, and on As death to B if B is then living. A has a life
Lives in Being
The law allows any lives to be used to show the validity or invalidity of an interest,
but no lives are of any help unless they are somehow connected with the vesting of an
interest. The measuring lives need not be given a benecial interest in the property, and
they need not even be expressly referred to in the instrument, but there must be some
connection that insures vesting or failure of the interest within the perpetuities period.
Examples:
1) To A for life, then to such of As children as attain the age of 21.
Here, the relevant measuring life is A. All of As children are going to
attain age 21, if at all, within 21 years after As death. (This includes
a child in the mothers womb at As death, for the perpetuities period
includes any period of gestation actually involved.)
2) Ts will devises her residuary estate to such of my nephews and
nieces as attain the age of 21. At the time of Ts death, she has two
brothers and six nephews and nieces, all of whom are under age 21. Is
the gift valid under the Rule? The answer: It depends. Specically, it
depends on whether Ts parents are living. The relevant measuring
lives are Ts brothers and sisters, because all of Ts nephews and nieces
will attain age 21, if at all, within 21 years after their parents deaths.
If Ts parents are dead, her two brothers are all the brothers she is ever
going to have; and Ts nephews and nieces will be the children of these
brothers. The disposition is valid.
But if Ts parents are alive, they might have another child (call him
Excelsior), a brother or sister of T not alive at Ts death. Then Ts two
brothers and six nephews and nieces who were alive at Ts death might
die. Then Excelsior might have a child who lives to attain age 21more
than 21 years after any life in being. Because this might happen, the
disposition is invalid under the Rule.
1)
or invalidity of interests are referred to or are indirectly involved in the disposition itself. It is a common drafting practice to use a perpetuities saving clause
(i) to make sure that the Rule has not been accidentally violated, for the Rule
is difcult to master; and (ii) sometimes to extend the duration of trusts to the
maximum extent permitted under the Rule. The clause reads something like this:
Notwithstanding anything herein to the contrary, any trust created hereunder
shall terminate, if it has not previously terminated, 21 years after the death of the
survivor of the following named persons:
; and the remaining
principal and undistributed income of such trusts shall be distributed to . . . .
In the blanks are inserted the names of the persons to be used as articial
measuring lives. Most commonly, the descendants then living of the transferor
are specically named. Alternatively, the clause might provide: after the death of
the survivor of all my descendants who shall be living at the time of my death
this clause works in a will but does not work in an irrevocable trust. A few more
aggressive draftsmen will name 10 healthy babies born in some local hospital on
the day the instrument is executedthe probability is that this will permit the trust
to run for 100 years.
2)
e.
and in that event to the American Cancer Society. The YMCA has
a fee simple subject to an executory interest; the Cancer Society has
a shifting executory interest. The gift over to the Cancer Society is
valid under the charity-to-charity exception to the Rule.
2) To the Georgetown YMCA for so long as the premises are
used for YMCA purposes; and when they shall cease to be so
used, then and in that event to John Hancock, his heirs, successors, and assigns. Classifying the interests without regard to the
Rule, the YMCA would have a fee simple subject to an executory
interest; Hancock would have a shifting executory interestwhich
is stricken because it violates the Rule. The YMCA has a fee simple
determinable, and the transferor has a possibility of reverter.
3) To John Hancock, his heirs, successors, and assigns, provided
that no marijuana is ever smoked on the premises; and if marijuana
is ever smoked on the premises, to the Georgetown YMCA.
Classifying interests without regard to the Rule, Hancock would
have a fee simple subject to an executory interest; the YMCA would
have a shifting executory interestwhich is stricken because it
violates the Rule. Thus, Hancock has a fee simple absolute.
2)
Vested Interests
A vested remainder in an individual is not subject to the Rule. Thus, a devise to A
for life, then to As children for life, then to B in fee simple is wholly valid. B has
a presently vested remainder. It may vest in possession long after lives in being if
A leaves some surviving children born after the testators death, but the remainder
to B is presently vested in interest, and that is what counts.
a)
3)
Reversionary Interests
Reversions, possibilities of reverter, and rights of entry are all vested in interest and
hence are not subject to the Rule Against Perpetuities. (Even so, in many states,
there are statutes expressly limiting the duration of possibilities of reverter or rights
of entry.)
a)
2.
ExceptionInfectious Invalidity
There is an important exception to the preceding statement. Under the principle
of infectious invalidity, if the invalid gift is an essential part of the transferors
dispositive scheme, such that to strike this interest and give effect to the remaining
interests would be to subvert the transferors intentif it is determined that the
transferor would prefer that the entire disposition failthen the entire disposition
is void.
Example:
Irrevocable Trust A directs the trustee to pay the income to my
brothers children for life, and on the death of my brothers last
surviving child, to pay the principal to the issue of my brothers
children. The gift to the issue of my brothers children is void
because the brother might have a child born after the creation of
the trust (who would thus not be a life in being). That child might
not die within 21 years after the death of the last child living at the
time the trust was created. After the invalid interest is stricken, the
brothers children have a life estate in the income, and the settlor
has a reversion in the principal. However, Trust B provides that
if any provision of Trust A is invalid, any income and principal
from Trust A should be held in trust for the benet of the brothers
children and their issue. By striking out all the gifts in Trust A,
valid and invalid, and by giving effect to Trust B, the settlors
presumed intent to benet his brothers children and their issue may
be substantially carried out. [New England Trust Co. v. Sanger, 149
N.E.2d 598 (Mass. 1958)]
long as, until), the estate will still terminate upon the happening of the stated event
and the grantor will have a possibility of reverter. In contrast, if the fee is subject to a
condition subsequent, the condition is also stricken and the estate becomes a fee simple
absolute.
Examples:
1) To John Brown for so long as no marijuana is smoked on the
premises; and if marijuana is ever smoked on the premises, then to
Candy Barr. (i) John Brown has a fee simple subject to an executory
interest; Candy Barr has an executory interest in fee simple. (ii) Candys
interest violates the Rule. Things may stay quiet for generations, long
beyond 21 years after the deaths of John and Candy (the only relevant
lives in this case). Then someone might light up, triggering the executory interest. Because this might happen, the executory interest might
vest (i.e., might become a possessory estate) long after lives in being plus
21 years. Candys interest violates the Rule, and it is stricken. This leaves
a fee simple determinable in John Brown, and a possibility of reverter in
the transferor. The possibility of reverter is valid; retained interests in the
transferor are not subject to the Rule.
What if marijuana is smoked on the premises within ve years after
the transfer, well within lives in being plus 21 years, meaning that in
actuality there was no remote vesting? It does not matter. Under the
Rule we do not wait and see. (But see 1.b.1), supra.) It is what might
happen that counts, viewing the facts as they exist at the time the
interest is created. What actually happens is irrelevant.
2) To John Brown; provided, however, that if marijuana is ever smoked
on the premises, then to Candy Barr. (i) John Brown has a fee simple
subject to an executory interest; Candy Barr has an executory interest
in fee simple. (ii) Candys interest violates the Rule, under the analysis
given in Example 1). Again we strike Candys interestleaving John
Brown with a fee simple absolute. (Contrast this with the result in
Example 1), where the transferor had a possibility of reverter. This is
the result of two different forms of expressing the gift to John Brown.
Whenever a fee simple determinable is created (so long as), the transferor automatically has a possibility of reverter. But when a fee on a
condition subsequent is created, the transferor does not have a right of
entry unless the right of entry is expressly raised.)
b.
being) attains age four, A, X, and Y might die; then Z might live on to
age 25, at which time the remainder to As children would vest in Z. But
if this were to happen (and it might), the remainder would vest beyond
lives in being plus 21 years. Because the interest might vest remotely, it
is stricken.
c.
d.
e.
"To A for life, then to his widow for life; and on the death of As widow,
to A's children." (i) A has a life estate; there is a contingent remainder in
a life estate in his widow; As present children have vested remainders
in fee simple that are subject to open in favor of any future children
born to A, and As unborn children have a contingent remainder in fee
simple. (ii) The remainder in A's children is valid under the Rule because
their interests will vest (and the class will close) within As lifetime. The
difference here is that the childrens interest was not contingent on their
surviving As widow, and the law does not imply such a condition.
Options
An option creates in the optionee a right to purchase the property on terms
provided in the option. Options are typically considered to be assignable unless
the parties provide otherwise and thus are subject to the Rule Against Perpetuities
(but see b), infra). If an option is structured so that it might be exercised later than
the end of the Rules period, it is usually held void.
Examples:
1) A is a subdivision developer and gives B an option to purchase a
lot in the subdivision to be exercised within 60 days after the City
Council grants approval for the ling of a subdivision plat. While
the parties may expect this to occur soon, it is possible that it will
not occur within 21 years after any life in being at the creation of
the option. Hence, the option may be held void.
2) O conveys to A and her heirs an option to purchase Blackacre
for $25,000. This option is not limited to As life, but can be
exercised by As heirs and their heirs long after As death. Thus, the
option is void.
a)
b)
the lease term, the Rule is not applied, no matter how long the term. If a
tenant assigns the leasehold, the option generally is considered a running
covenant, exercisable by the assignee in the absence of contrary intent.
However, a tenant may attempt to transfer the option to some other party,
thereby separating it from the leasehold estate. While some courts do not
permit the option to be transferred separately, most courts hold that the
transferability of the option depends on the original parties intent when
they entered into the lease and option agreement. If the court nds that the
option has been separated from the leasehold estate, so that it is no longer
exercisable by the tenant, the option becomes subject to the Rule Against
Perpetuities.
2)
3.
(ii) All conditions precedent for every member of the class must be satised, if at all,
within the perpetuities period.
If it is possible that a disposition might vest remotely with respect to any member of the
class, the entire class gift is invalid.
Examples:
1) To A for life, then to such of As children as live to attain the age of
35. At the time of this disposition, A has two children: X (age 38) and
Y (age 33). (i) A has a life estate. X has a vested remainder subject to
open. There is a contingent remainder in such of As other children as
live to attain the age of 35. (ii) The remainder to As children violates the
Rule; the transferor has a reversion in fee. While Xs remainder is vested
subject to open, it is not vested for purposes of the Rule. Here is what
might happen: A might have another child (Z); before Z attains age 14,
A, X, and Y might die, etc. The gift with respect to any afterborn child
of A clearly violates the Rule; under the bad-as-to-one, bad-as-to-all
class gift rule, the entire class gift is void.
Why does the class closing rule not save the gift? Because X is already
age 35, doesnt this mean that the class will close on the life tenants
death? The answer is: yes it does, but this does not help. Although we
will close the class at that point, the class as closed might include the
afterborn Z, who might be under age 14 at that time; and it still might be
more than lives in being plus 21 years before Zs interest might vest.
2) To A for life, then to As children for their lives, and on the death of
the last survivor of As children, to As grandchildren in fee. At the time
this disposition takes effect, A is alive and has two children and three
grandchildren. A has a life estate, the two children have vested remainders subject to open in a life estate, and the three grandchildren have
vested remainders subject to open in fee. The remainder to As grandchildren is void because every member of the class will not be ascertained until the death of the survivor of As children; and that surviving
child might be born to A after the date of this disposition. Then all of As
children and grandchildren who are lives in being might die and 21 years
after their deaths, this afterborn child might give birth to a child (GC-4);
although GC-4s interest would vest at birth, under the hypothesized
facts it would vest remotely.
b.
c.
d.
4.
5.
A wait and see statute, under which the validity of an interest following one or more
life estates is determined on the basis of facts existing at the end of the life estate rather
than at the creation of the interest (see 1.b.1), supra);
b.
A cy pres approach, borrowed from Trusts law, under which an invalid interest is
reformed to comply with the Rule and carry out the grantors intent as nearly as
possible;
c.
A statute dealing with specic perpetuities problems (e.g., age contingencies reduced to
age 21, women over age 55 presumed incapable of childbearing, gift to widow presumed
to mean the person who was the spouse on the date the gift was created); or
d.
b.
must vest within lives in being plus 21 years or that the interest might not vest within
that period. At this step, assume that there is no such thing as a perpetuities reform
statute, for the statute is not brought into play unless there is a perpetuities problem.
c.
F.
2.
Total Restraints
Any total restraint on a fee simpleeither forfeiture, disabling, or promissoryis void.
The grantee may ignore the restraint and freely transfer the property.
b.
Partial Restraints
A partial restraint is one that purports to restrict the power to transfer to specic
persons, or by a specic method, or until a specic time.
1)
promissory restraint for a limited time and for a reasonable purpose may be
upheld.
Example:
A owns and resides in a house. He conveys a one-half interest in
the house to his brother, B, including in the deed a covenant that
during their joint lifetimes, each party promises not to convey his
interest to any other person without the consent of the other party.
This promissory restraint is limited to the joint lifetimes of the
parties and is a reasonable way to ensure that neither party will be
faced with the prospect of residing with a stranger. The restraint
would probably be upheld.
2)
3.
Discriminatory Restraints
Restraints prohibiting the transfer or use of property to or by a person of a specied racial, religious, or ethnic group are not enforceable.
a)
Fourteenth Amendment
Judicial enforcement of a covenant forbidding use of property by persons of
a particular race is discriminatory state action forbidden by the Fourteenth
Amendment to the United States Constitution. [Shelley v. Kraemer, 334 U.S.
1 (1948)]
Example:
O conveys Blackacre to A and his heirs, and A promises
that Blackacre will never be used or occupied by nonwhite
persons. Thereafter A sells to B, a black man, who moves
onto Blackacre. O sues for an injunction prohibiting B from
using Blackacre, and sues A for damages for having sold to B.
Injunction and damages are judicial remedies ordinarily available for breach of a covenant. The court cannot grant O either
an injunction or damages, because such judicial action would
be state action interfering with Bs right to enjoy property free
of racial discrimination. [Barrows v. Jackson, 346 U.S. 249
(1953)]
b)
b.
exact opposite of the rule applicable to legal interests. Spendthrift clauses, which are
true disabling restraints, are given effect in the great majority of American jurisdictions.
(See Trusts outline.)
4.
5.
b.
c.
Contingent Remainders
The law is unsettled as to the validity of forfeiture and promissory restraints on contingent remainders. Thus, this issue is not likely to be tested.
b.
or the right to meet any offer, is valid if reasonable (e.g., by specifying fair market
value or other reasonable price). [Restatement (Third) of Property: Servitudes 3.4]
c.
G. CONCURRENT ESTATES
Any of the estates in land previously discussed can be held concurrently by several persons. These
persons all have the right to the enjoyment and possession of the land at the same time. Three of
the chief forms of concurrent ownership in land are discussed here: joint tenancy, tenancy by the
entirety, and tenancy in common.
1.
Joint Tenancy
A joint tenancy can be created between two or more co-tenants. Its distinguishing feature
is the right of survivorship. Conceptually, when one joint tenant dies, the property is freed
from his concurrent interest; the survivor or survivors retain an undivided right in the
property, which is no longer subject to the interest of the deceased co-tenant. The survivors
do not succeed to the decedents interest; they hold free of it.
a.
Creation
1)
b)
c)
d)
2)
Modern Law
The above requirements have been eroded in some jurisdictions; e.g., by statute
in some states, an owner can create a joint tenancy in herself and another by a
single deed (she need not use a strawman conveyance), even though the unities
of time and title are not satised. Similarly, as indicated below, a number
of transactions are no longer found to sever a joint tenancy despite the seeming
absence of continued unities.
3)
b.
Severance
A joint tenancy can be terminated by a suit for partition, which can be brought by any
joint tenant. It may also be terminated by various acts by any joint tenant.
1)
b)
4)
2.
CompareSecret Deeds
As indicated above, an inter vivos conveyance severs a joint tenancy. This is
true even though the deed is kept secret and the interest transferred is to
take effect only upon the death of the grantor. However, if the grantee does
not know about the deed, the grantees acceptance after the death of the
grantor does not relate back to defeat the right of survivorship. (See VI.C.4.b.,
infra.)
Right of Survivorship
The estate carries a right of survivorship, which operates in the same manner as the
right of survivorship incident to a joint tenancy.
b.
Severance Limited
The major distinction between a joint tenancy and a tenancy by the entirety concerns
3.
Tenancy in Common
A tenancy in common is a concurrent estate with no right of survivorship. Each owner has a
distinct, undivided interest in the property. This interest is freely alienable by inter vivos and
testamentary transfer, is inheritable, and is subject to claims of the tenants creditors. The
only unity involved is possession: Each tenant is entitled to possession of the whole estate.
Today, by statute, multiple grantees are presumed to take as tenants in common. The same is
true where multiple transferees take by descent.
4.
Incidents of Co-Ownership
a.
Possession
Each co-tenant has the right to possess all portions of the property; no co-tenant has the
right to exclusive possession of any part. A co-tenant out of possession cannot bring a
possessory action unless there has been an ouster by the tenant in possession. A claim
of right to exclusive possession can constitute an ouster.
b.
c.
in the land, but it does not cause a severance. If A dies before the lien is
foreclosed and is survived by B, B owns the land free and clear of the
lien. But if P forecloses the lien before As death, the foreclosure sale will
cause a severance, and the buyer at the sale will own a one-half interest
in the land as a tenant in common with B.
d.
Ouster
Under the unity of possession, each co-tenant is entitled to possess and enjoy the whole
of the property subject to the equal right of her co-tenant. If one tenant wrongfully
excludes another co-tenant from possession of the whole or any part of the whole of
the premises, there is an ouster. The ousted co-tenant is entitled to receive his share of
the fair rental value of the property for the time he was wrongfully deprived of possession.
e.
Remedy of Partition
A joint tenant or tenant in common has a right to judicial partition, either in kind
(division of the tract into parcels) or by sale and division of the proceeds (in accordance with the ownership interests as modied by permitted recoupments for improvements, repairs, taxes, and the like). Although partition in kind is generally preferred,
partition by sale and division of the proceeds is permitted when a fair and equitable
physical division of the property cannot be made. [Nordhausen v. Christner, 338 N.W.2d
754 (Neb. 1983)]
Examples:
1) A and B own a single-family home as joint tenants. A brings an
appropriate action to partition the land. Because physical division of the
home is not feasible, the court will order a sale of the home and division
of the proceeds equally between A and B.
2) A owns a three-fourths interest and B owns a one-fourth interest
in a four-acre undeveloped parcel of land as tenants in common. The
applicable zoning ordinance requires that a buildable lot contain at least
two acres. A seeks to partition the land into a three-acre lot for himself
and a one-acre lot for B. B argues that the land should be sold and the
proceeds divided between A and B according to their respective shares.
B will prevail, because the zoning ordinance makes it impossible to
divide the land fairly.
1)
f.
1)
g.
2)
3)
NATURE OF LEASEHOLD
A leasehold is an estate in land. The tenant has a present possessory interest in the leased
premises, and the landlord has a future interest (reversion). Certain rights and liabilities ow from
this property relationship between landlord and tenant. The three major types of leasehold estates
are tenancies for years, periodic tenancies, and tenancies at will. There is a fourth category
called tenancies at sufferance.
1.
b.
Creation
Tenancies for years are normally created by written leases. In most states, the Statute
of Frauds requires that a lease creating a tenancy for more than one year be memorialized in writing. In addition, most states have statutes that restrict the number of years
for which a leasehold estate may be created (e.g., 51 years for farm property and 99
years for urban property). When the lease term exceeds the statutory maximum, most
courts hold that the lease is entirely void. Likewise, where the lease contains an option
to renew for a period beyond the permitted maximum, most courts hold the entire lease
void.
c.
Termination
A tenancy for years ends automatically on its termination date.
1)
Breach of Covenants
In most tenancy for years leases, the landlord reserves the right to terminate if the
tenant breaches any of the leasehold covenants. This reserved power is called the
landlords right of entry.
a)
2)
2.
Surrender
A tenancy for years also terminates upon surrender. Surrender consists of the
tenant giving up his leasehold interest to the landlord and the landlord accepting.
Usually the same formalities are required for the surrender of a leasehold as are
necessary for its creation. Thus, a writing is necessary for the surrender of a leasehold if the unexpired term is more than one year.
Periodic Tenancies
A periodic tenancy is a tenancy that continues from year to year or for successive fractions of
a year (e.g., weekly or monthly) until terminated by proper notice by either party. The beginning date must be certain, but the termination date is always uncertain until notice is given.
All conditions and terms of the tenancy are carried over from one period to the next unless
there is a lease provision to the contrary. Periodic tenancies do not violate the rules limiting
the length of leaseholds because each party retains the power to terminate upon giving
notice.
a.
Creation
Periodic tenancies can be created in three ways:
1)
By Express Agreement
Periodic tenancies can be created by express agreement (e.g., Landlord leases to
Tenant from month to month).
2)
By Implication
A periodic tenancy will be implied if the lease has no set termination but does
provide for the payment of rent at specic periods.
Example:
Landlord leases to Tenant at a rent of $1,000 payable monthly
in advance. The reservation of monthly rent will give rise to a
periodic tenancy from month to month.
Note: If the lease reserves an annual rent, payable monthly (e.g., $12,000 per
annum, payable $1,000 on the rst day of every month commencing January 1),
the majority view is that the periodic tenancy is from year to year.
3)
b.
By Operation of Law
A periodic tenancy may arise even without an express or implied agreement
between the parties.
a)
b)
Lease Invalid
If a lease is invalid (e.g., because of failure to satisfy the Statute of Frauds)
and the tenant nonetheless goes into possession, the tenants periodic payment
of rent will convert what would otherwise be a tenancy at will into a periodic
tenancy. The period of the tenancy coincides with the period for which the
rent is paid.
TerminationNotice Required
A periodic tenancy is automatically renewed, from period to period, until proper notice
of termination is given by either party. Many jurisdictions have statutorily prescribed
the notice required to terminate a periodic tenancy. In general, the guidelines are as
follows:
(i)
(ii) For a tenancy from year to year, six months notice is required.
(iii) For tenancies less than one year in duration, a full period in advance of the period
in question is required by way of notice (e.g., for a month-to-month periodic
tenancy, one full months notice is required).
In general, the notice required to terminate a periodic tenancy must be in writing and
must actually be delivered to the party in question or deposited at his residence in a
manner similar to that required for service of process.
3.
Tenancies at Will
A tenancy at will is an estate in land that is terminable at the will of either the landlord or the
tenant. To be a tenancy at will, both the landlord and the tenant must have the right to terminate the lease at will.
(i)
If the lease gives only the landlord the right to terminate at will, a similar right will
generally be implied in favor of the tenant so that the lease creates a tenancy at will.
(ii) If the lease is only at the will of the tenant (e.g., for so long as the tenant wishes),
courts usually do not imply a right to terminate in favor of the landlord. Rather, most
courts interpret the conveyance as creating a life estate or fee simple, either of which is
terminable by the tenant. (If the Statute of Frauds is not satised, the conveyance is a
tenancy at will.)
4.
a.
Creation
A tenancy at will generally arises from a specic understanding between the parties
that either party may terminate the tenancy at any time. Note that unless the parties
expressly agree to a tenancy at will, the payment of regular rent (e.g., monthly,
quarterly, etc.) will cause a court to treat the tenancy as a periodic tenancy. Thus, tenancies at will are quite rare. Although a tenancy at will can also arise when the lease is for
an indenite period (one that does not satisfy the requirements for creating a tenancy
for years), or when a tenant goes into possession under a lease that does not satisfy the
requisite formalities (usually the Statute of Frauds), rent payments will usually convert
it to a periodic tenancy.
b.
Termination
A tenancy at will may be terminated by either party. At common law, no notice was
required to terminate a tenancy at will. But the majority of states now require that a
party give the other notice of termination and a reasonable time to quit the premises. A
tenancy at will also terminates by operation of law if:
1)
2)
3)
4)
5)
Tenancies at Sufferance
A tenancy at sufferance (sometimes called occupancy at sufferance) arises when a tenant
wrongfully remains in possession after the expiration of a lawful tenancy (e.g., after the
stipulated date for the termination of a tenancy for years; or after the landlord has exercised
a power of termination). Such a tenant is a wrongdoer and is liable for rent. The tenancy at
sufferance lasts only until the landlord takes steps to evict the tenant. No notice is required to
end the tenancy, and authorities are divided as to whether this is even an estate in land.
5.
Eviction
The landlord may treat the hold-over tenant as a trespasser and evict him under an
unlawful detainer statute.
b.
Terms
The terms and conditions of the expired tenancy (e.g., rent, covenants, etc.) apply
to the new tenancy. In commercial leases, if the original lease term was for one
year or more, a year-to-year tenancy results from holding over. If the original
term was for less than one year, the periodic term is determined by the manner in
which the rent was due and payable under the prior tenancy. In residential leases,
however, most courts would rule the tenant a month-to-month tenant (or a weekto-week tenant if the tenant was a roomer paying weekly rent), irrespective of the
term of the original lease.
Example:
A nonresidential tenant was holding under a six-month term
tenancy with rent payable monthly. The tenant holds over and the
landlord binds him to a new tenancy. The new periodic tenancy is a
month-to-month tenancy.
2)
Altered Terms
If the landlord noties the tenant before termination of the tenancy that occupancy
after termination will be at an increased rent, the tenant will be held to have acquiesced to the new terms if he does not surrender. The tenant will be held to the new
terms even if he objects to the increased rent, provided that the rent increase is
reasonable.
c.
d.
e.
B.
LEASES
A lease is a contract containing the promises of the parties. It governs the relationship between the
landlord and tenant over the term of the lease.
1.
2.
In the example in 1., above, Ls breach of a covenant to paint likely would not
be considered material.
C.
Types of Waste
1)
2)
Permissive Waste
Unless the lease provides otherwise, the tenant has no duty to the landlord to make
any substantial repairs (i.e., to keep the premises in good repair). However, the
tenant has a duty to make ordinary repairs to keep the property in the same condition as at the commencement of the lease term, excluding ordinary wear and tear
(unless the tenant covenanted to repair ordinary wear and tear; see c.2), infra).
For example, it is the tenants duty to repair broken windows or a leaking roof
and to take such other steps as are needed to prevent damage from the elements
(i.e., keep the premises wind and water tight). If the tenant fails to do so, he is
liable to the landlord for any resulting damage, but not for the cost of repair. Under
the URLTA, residential tenants have additional duties: (i) not to cause housing
code violations; (ii) to keep the premises clean and free of vermin; and (iii) to use
plumbing, appliances, etc., in a reasonable manner. Note that even when the burden
of repair is on the landlord, the tenant does have a duty to report deciencies
promptly to the landlord.
3)
Ameliorative Waste
A tenant is under an obligation to return the premises in the same nature and
character as received. Therefore, a tenant is not permitted to make substantial
alterations to leased structures even if the alteration increases the value of the
property.
a)
LiabilityCost of Restoration
The tenant is liable for the cost of restoration should he commit ameliorative
waste.
b)
(3) The change reects a change in the nature and character of the neighborhood.
b.
c.
2)
2.
3.
4.
b.
b.
Rent Deposits
Landlords often require a deposit by the tenant at the outset of the lease. If the money
is considered a security deposit, the landlord will not be permitted to retain it beyond
the extent of his recoverable damages. But if the deposit is denominated a bonus or a
future rent payment (e.g., the last months rent), then most courts permit the landlord to
retain it after the tenant has been evicted.
c.
Landlord Remedies
a.
severely limit the issues that may be raised. Under most statutes, the only issue properly
before the court is the landlords right to rent and possession. The tenant cannot raise
counterclaims.
1)
b.
DistressLandlords Lien
In some states (especially in nonresidential leases), a landlord who does not receive
rent when due can assert a lien on the personal property found on the leased
premises. This applies to property owned by sublessees as well as by the original
tenant.
2)
b)
between the future rent under the lease and the fair market or relet rental
value.
D.
2.
b.
Tenant RemedyDamages
In states following the majority rule, a tenant is entitled to damages against a landlord
in breach of the duty to deliver possession. If, e.g., the tenant had to nd more expensive housing during the interim or suffered business losses as a consequence of the
landlords breach, he may recover for these items.
Quiet Enjoyment
There is implied in every lease a covenant that neither the landlord nor someone with
paramount title (e.g., a prior mortgagee of the landlord who forecloses) will interfere with
the tenants quiet enjoyment and possession of the premises. The covenant of quiet enjoyment may be breached in any one of three ways: actual eviction, partial actual eviction, or
constructive eviction.
a.
Actual Eviction
Actual eviction occurs when the landlord, a paramount title holder, or a hold-over tenant
excludes the tenant from the entire leased premises. Actual eviction terminates the
tenants obligation to pay rent.
b.
for the entire premises, even though the tenant continues in possession of the
remainder of the premises.
2)
c.
Constructive Eviction
Constructive eviction occurs when a landlords breach of duty renders the premises
untenantable (i.e., unsuitable for occupancy). To establish a claim for constructive
eviction, the tenant must prove:
(i)
The landlord, or persons acting for him, breached a duty to the tenant (acts of
neighbors or strangers will not sufce);
(ii) The breach substantially and materially deprived the tenant of her use and enjoyment of the premises (e.g., ooding, absence of heat in winter, loss of elevator
service in a warehouse);
(iii) The tenant gave the landlord notice and a reasonable time to repair; and
(iv) After such reasonable time, the tenant vacated the premises.
A tenant who has been constructively evicted may terminate the lease (i.e., is relieved
of her duty to pay rent from the date of abandonment) and may also seek damages.
Constructive eviction is often raised as a defense in a landlords suit for damages or rent.
3.
b.
Remedies
The following remedies have been adopted by various courts for violation of the implied
warranty (although few courts have adopted all):
1)
Tenant may move out and terminate lease (as in a constructive eviction).
2)
Tenant may make repairs directly, and offset the cost against future rent obligations. (Some states limit this remedy by statute to a xed amount, such as one
months rent, or to only one occasion each year.)
3)
Tenant may reduce or abate rent to an amount equal to the fair rental value in
view of the defects in the property. (In many jurisdictions, the tenant may withhold
all rent until the court determines the amount of this fair rental value, and may
then pay it without risk of the landlords terminating the lease for rent delinquency.)
4)
E.
Tenant may remain in possession, pay full rent, and seek damages against the
landlord.
4.
Retaliatory Eviction
If a tenant exercises the legal right to report housing or building code violations or other
rights provided by statute (e.g., a residential landlord-tenant act), the landlord is not permitted
to terminate the tenants lease in retaliation. The landlord is also barred from penalizing the
tenant in other ways, such as raising the rent or reducing tenant services. This protection is
recognized by residential landlord-tenant acts in nearly half the states. These statutes usually
presume a retaliatory motive if the landlord acts within, say, 90 to 180 days after the tenant
exercises his rights. In other states, the same conclusion is reached by judicial construction
of the eviction and code statutes. The protection generally applies to tenants under both
periodic leases when the landlord gives notice to terminate and xed-term leases when the
landlord refuses to renew. To overcome the presumption, the landlord must show a valid,
nonretaliatory reason for his actions.
5.
Discrimination
The Civil Rights Act of 1866 bars racial or ethnic discrimination in the sale or rental of
all property. The Fair Housing Act bars discrimination based on race, ethnicity, religion,
national origin, gender, and disability in the sale or rental of a dwelling. Discrimination
against families with children is also barred except in senior citizen housing. The Act does
not apply to religious organizations, private clubs, and owners who have no more than three
single-family dwellings or who have an owner-occupied apartment with no more than four
units. [42 U.S.C. 3603(b), 3607]
Consequences of Assignment
The label given to a transferan assignment or subleasedetermines whether the landlord
can proceed directly against the transferee or only against the transferor. To be an assignment, the transfer must be on the same terms as the original lease except that the tenant
may reserve a right of termination (reentry) for breach of the terms of the original lease
that has been assigned; e.g., to A for the balance of the leasehold term. However, should A
fail to make the rental payments to the landlord, the right to reenter and reclaim the premises
is reserved. If the transfer is an assignment, the assignee stands in the shoes of the original
tenant in a direct relationship with the landlord. The assignee and the landlord are in privity
of estate, and each is liable to the other on all covenants in the lease that run with the land.
a.
b.
2)
2.
Consequences of Sublease
In a sublease, the sublessee is considered the tenant of the original lessee, and usually pays
rent directly to the original lessee, who in turn pays rent to the landlord under the main lease.
a.
3.
b.
Assumption by Sublessee
It is possible for the sublessee to assume the rent covenant and other covenants in the
main lease. An assumption is not implied, but must be expressed. If this occurs, the
sublessee is bound by the assumption agreement and becomes personally liable to the
landlord on the covenants assumed. The landlord is considered a third-party beneciary
of the assumption agreement.
c.
Rights of Sublessee
The sublessee can enforce all covenants made by the original lessee in the sublease, but
has no direct right to enforce any covenants made by the landlord in the main lease.
However, it is likely (although there is very little case law on point) that a sublessee in
a residential lease would be permitted to enforce the implied warranty of habitability
against the landlord.
b.
Waiver of Covenant
Even if the lease has a valid covenant against assignment, the covenant may be held
waived if the landlord knows of the assignment and does not object. This often occurs
when the landlord knowingly accepts rent from the assignee.
c.
Continuing Waiver
If the landlord grants consent to one transfer, the Rule in Dumpors Case provides that
he waives his right to avoid future transfers unless he expressly reserves the right to do
so. Reservation of right must take place at the time of granting consent.
4.
F.
d.
e.
Reasonableness
In a minority of states, the landlord may not unreasonably withhold consent to transfers
by the tenant. The majority imposes no such limitation.
Assignments by Landlords
a.
Right to Assign
A landlord may assign the rents and reversion interest that he owns. This is usually
done by an ordinary deed from the landlord to the new owner of the building. Unless
required by the lease (which is very unlikely), the consent of the tenants is not required.
b.
c.
CONDEMNATION OF LEASEHOLDS
1.
2.
Landlords Liability
At common law, subject to a few exceptions, a landlord had no duty to make the premises
safe. Today there are six exceptions to this rule:
a.
b.
Common Areas
The landlord has a duty to exercise reasonable care over common areas, such as halls,
walks, elevators, etc., that remain under his control. The landlord is liable for any injury
resulting from a dangerous condition that could reasonably have been discovered and
made safe. This duty is the same as the duty an owner-occupier owes his guests (see
Multistate Torts outline).
c.
Public Use
A landlord is liable for injuries to members of the public if, at the time of the lease, he:
(i) knows or should know of a dangerous condition, (ii) has reason to believe that the
tenant may admit the public before repairing the condition (e.g., because of short lease
term), and (iii) fails to repair the condition. The landlords liability extends only to
people who enter the premises for the purpose for which the public is invited. Note that
the tenants promise to repair does not relieve the landlord of liability if the landlord has
reason to suspect that the tenant will admit the public before making the repair.
d.
e.
f.
2.
3.
b.
c.
Security
Some cases have held landlords liable for injuries inicted on tenants by third-party
criminals, where the landlord failed to comply with housing code provisions dealing
with security, or failed to maintain ordinary security measures (e.g., working locks on
apartment doors), or where he advertised extraordinary security measures (e.g., television surveillance, doormen, security patrols) and then failed to provide them.
Tenants Liability
The tenant, as occupier of the premises, may be liable in tort to third persons for dangerous
conditions or activities on the leased property. The duty of care owed by the tenant as an
occupier of land is discussed in the Multistate Torts outline.
III. FIXTURES
A.
IN GENERAL
A xture is a chattel that has been so afxed to land that it has ceased being personal property
and has become part of the realty. For example, S and B contract to sell and buy a house. Before
vacating, S removes a built-in refrigerator. B claims that the item was part of the house. Is the
refrigerator a xture? If so, B is entitled to its return or appropriate compensation.
It is important in dealing with xture problems to distinguish between common ownership
cases and divided ownership cases. Courts treat them differently even though they often purport
to apply the same tests. Common ownership cases are those in which the person who brings
the chattel onto the land owns both the chattel and the realty (e.g., X installs a furnace in her own
home). Divided ownership cases are either ones where the person who owns and installs the
chattel does not own the land (e.g., T installs a furnace in her rented home, which belongs to L);
or the person owns the land but does not own the chattel (e.g., it is subject to a security interest
held by the seller). In addition, there are cases involving more than two persons (e.g., conicting
claims are made by the person having a security interest in the chattel and the mortgagee of the
land).
B.
C.
The nature of the article (i.e., how essential the item is to normal use of the premises);
(ii) The manner in which it is attached to the realty (the more substantially attached, the
more likely it was intended to be permanent);
(iii) The amount of damage that would be caused by its removal; and
(iv) The adaptation of the item to the use of the realty (e.g., custom window treatments,
wall-to-wall carpet).
a.
Constructive Annexation
In some cases, an article of personal property is considered a xture even though it is
not physically annexed to the real estate at all. This is because it is so uniquely adapted
to the real estate that it makes no sense to separate it. Examples include the keys to the
doors of a house; curtain rods that have been cut and sized to the brackets on the walls
of a house, even if the rods themselves are not presently installed; and a carpet that has
been cut to t an unusually shaped room, even if the carpet is not nailed or glued in
place.
b.
Vendor-Purchaser Cases
The typical situation is where the owner of land afxes chattels to the land and subsequently conveys the land without expressly providing whether the chattels are to pass
with the realty. The intention test works fairly well. The question boils down to whether
an owner bringing the disputed chattel to the realty would intend that it become part of
the realty. Or to put it another way, whether a reasonable purchaser would expect that
the disputed item was part of the realty.
c.
2.
D.
Mortgagor-Mortgagee Cases
The intention test is universally applied to determine whether the owner (mortgagor)
intended the chattels to become part of the realty. Where the mortgagor has made the
annexation prior to the giving of the mortgage, the question is what the reasonably
objective lender expects to come within the security of her lien. However, where the
annexation is made after the giving of the mortgage, the same considerations arguably
should not apply because each item that is added to the lien of the mortgage represents a windfall to the mortgagee should foreclosure occur. Nevertheless, courts universally apply the same intention test regardless of when the annexation was made. (Courts
also usually apply the intention test where items are annexed by one in possession of
land under an executory contract to purchase.)
Conveyance
If a chattel has been categorized as a xture, it is part of the real estate. A conveyance
of the real estate, in the absence of any specic agreement to the contrary, passes the
xture with it. The xture, as part of the realty, passes to the new owner of the real
estate.
b.
Mortgage
To the extent that the owner of the real estate mortgages the realty, in the absence of an
agreement to the contrary, the mortgage attaches to all xtures on the real estate.
c.
Agreement to Contrary
Even though the concept of xtures may apply and a chattel becomes a xture, an
agreement between a buyer and seller (similarly, between a mortgagor and mortgagee)
can cause a severance of title. For example, a buyer and seller may agree that the seller
will retain the right to remove xtures. Similarly, a mortgagor and mortgagee can agree
that the mortgage lien shall not attach to specied xtures. The effect of such an agreement is to de-annex, so far as relevant, the chattel from the realty and reconvert the
xture into a chattel.
Landlord-Tenant
Early English law favored the landlord. However, American law created a trade xtures
exception under which tradesmen-tenants could remove an item used in their trade or
business, that otherwise would have been a xture, unless its removal would cause substantial damage to the premises. Later, this exception was expanded to include all tenants
generally. Some courts have treated the trade xtures exception as consistent with the annexors-intention test; i.e., a tenants annexations are removable because it was not the intention
of the tenant to make them permanent annexations to the freehold and thereby donations to
the owner of it.
a.
Agreement
An agreement between the landlord and tenant is controlling on whether the chattel
annexed to the premises was intended to become a xture. To the extent that the
landlord and tenant specically agree that such annexation is not to be deemed a xture,
the agreement controls.
b.
c.
d.
2.
3.
4.
E.
THIRD-PARTY CASES
Any of the foregoing cases is complicated by the addition of third-person claimants.
1.
UCC Rules
Normally, the rule is that whichever interest is rst recorded in the local real estate
records wins. (Thus, if the chattel security interest was recorded rst, it constitutes
constructive notice to all subsequent lenders or purchasers.) However, an exception
allows a purchase money security interest in an afxed chattel (here, the interest
given Seller to secure payment on the furnace) to prevail even over a prior recorded
mortgage on the land, as long as the chattel interest is recorded within 20 days after the
chattel is afxed to the land. [UCC 9-334]
The document used to record the chattel security interest is known as a xture ling.
(This is a separate instrument from the nancing statement, which is required to be
led to perfect the chattel security interest in the rst place.)
b.
IN GENERAL
Easements, prots, covenants, and servitudes are nonpossessory interests in land. They create a
right to use land possessed by someone else. For example, A, the owner of Blackacre, grants to
B, the owner of an adjacent parcel, Whiteacre, the right to use a path over Blackacre connecting
Whiteacre to a public road. An easement has been created, giving B the right to usebut not to
possessthe pathway over Blackacre. Easements, prots, covenants, and servitudes have many
similarities in operation, coverage, creation, and termination. They also have important differences, mainly in the requirements that must be met for their enforcement.
B.
EASEMENTS
1.
Introduction
The holder of an easement has the right to use a tract of land (called the servient tenement)
for a special purpose, but has no right to possess and enjoy the tract of land. The owner of
the servient tenement continues to have the right of full possession and enjoyment subject
only to the limitation that he cannot interfere with the right of special use created in the
easement holder. Typically, easements are created in order to give their holder the right of
access across a tract of land, e.g., the privilege of laying utility lines, or installing sewer pipes
and the like. Easements are either afrmative or negative, appurtenant or in gross.
a.
b.
Types of Easements
1)
Afrmative Easements
Afrmative easements entitle the holder to enter upon the servient tenement and
make an afrmative use of it for such purposes as laying and maintaining utility
lines, draining waters, and polluting the air over the servient estate. The right-ofway easement is another instance of an afrmative easement. Thus, an afrmative
easement privileges the holder of the benet to make a use of the servient estate
that, absent the easement, would be an unlawful trespass or nuisance.
2)
Negative Easements
A negative easement does not grant to its owner the right to enter upon the servient
tenement. It does, however, entitle the privilege holder to compel the possessor
of the servient tenement to refrain from engaging in activity upon the servient
tenement that, were it not for the existence of the easement, he would be privileged
to do. Courts historically recognized negative easements only for light, air, subjacent or lateral support, and for the ow of an articial stream. Today, a negative
easement is simply a restrictive covenant. (See D.1.e.1), infra.)
Example:
A owns Lot 6. By written instrument, he stipulates to B that he will
not build any structure upon Lot 6 within 35 feet of the lot line. B
has acquired a negative easement in Lot 6.
Easement Appurtenant
An easement is deemed appurtenant when the right of special use benets the holder of
the easement in his physical use or enjoyment of another tract of land. For an easement
appurtenant to exist, there must be two tracts of land. One is called the dominant
tenement, which has the benet of the easement. The second tract is the servient
tenement, which is subject to the easement right. One consequence of appurtenance
is that the benet passes with transfers of the beneted land, regardless of whether the
easement is mentioned in the conveyance.
Example:
A owns Lot 6 and B owns Lot 7, which are adjoining tracts of land. By a
written instrument, B grants to A the right to cross Bs tract (Lot 7). As
use and enjoyment of Lot 6 is beneted by virtue of the acquisition of
the right to use Lot 7 for this special purpose. The right is an easement
appurtenant. B remains the owner of Lot 7. A has only a right to use Lot
7 for a special purpose, i.e., the right to cross the tract.
1)
Example:
c.
2)
3)
Easement in Gross
An easement in gross is created where the holder of the easement interest acquires a
right of special use in the servient tenement independent of his ownership or possession
of another tract of land. In an easement in gross, the easement holder is not beneted in
his use and enjoyment of a possessory estate by virtue of the acquisition of that privilege. There is no dominant tenement. An easement in gross passes entirely apart from
any transfer of land.
Example:
A owns Lot 6. By a written instrument, she grants to B the right to
build a pipeline across Lot 6. B receives the privilege independent of his
ownership or possession of a separate tract of land. B has acquired an
easement in gross.
Easements in gross can be either personal (e.g., O gives friend right to swim and boat on
lake) or commercial (e.g., utility or railroad track easements). Generally, an easement in
gross is transferable only if the easement is for a commercial or economic purpose.
d.
2.
Creation of Easements
The basic methods of creating an easement are: express grant or reservation, implication, and
prescription.
a.
Express Grant
Because an easement is an interest in land, the Statute of Frauds applies. Therefore, any
easement must be memorialized in a writing that is signed by the grantor (the holder of
the servient tenement) unless its duration is brief enough (commonly one year or less)
to be outside the coverage of a particular states Statute of Frauds. An easement can be
created by conveyance. A grant of an easement must comply with all the formal requisites of a deed. An easement is presumed to be of perpetual duration unless the grant
specically limits the interest (e.g., for life, for 10 years).
b.
Express Reservation
An easement by reservation arises when the owner (of a present possessory interest) of a
tract of land conveys title but reserves the right to continue to use the tract for a special
purpose after the conveyance. In effect, the grantor passes title to the land but reserves
unto himself an easement interest. Note that, under the majority view, the easement can
be reserved only for the grantor; an attempt by the grantor to reserve an easement for
anyone else is void. (There is a growing trend to permit reservations in third parties, but
it remains a minority view.)
Example:
G owns Lot 6 and Lot 7, which are adjacent. G sells Lot 7 to B. Later,
when G is about to sell Lot 6 to A, B asks G to reserve an easement over
Lot 6 in favor of B. G agrees to do so, and executes a deed of Lot 6 to
A that contains the following language: Reserving an easement for a
driveway in favor of Lot 7, which is owned by B. The reservation clause
is void and no easement is created.
c.
Implication
An easement by implication is created by operation of law rather than by written
instrument. It is an exception to the Statute of Frauds. There are only three types of
implied easements: (i) an intended easement based on a use that existed when the
dominant and servient estates were severed, (ii) an easement implied from a recorded
subdivision plat or prot a prendre, and (iii) an easement by necessity.
1)
2)
3)
b)
Reasonable Necessity
Whether a use is reasonably necessary to the enjoyment of the dominant
parcel depends on many factors, including the cost and difculty of the alternatives and whether the price paid reects the expected continued use of the
servient portion of the tract.
c)
Grant or Reservation
An easement implied in favor of the grantee is said to be created by implied
grant, while an easement implied in favor of the grantor is said to be created
by implied reservation.
Subdivision Plat
When lots are sold in a subdivision with reference to a recorded plat or map
that also shows streets leading to the lots, buyers of the lots have implied
easements to use the streets in order to gain access to their lots. These
easements continue to exist even if the public easements held by the city or
county in the streets are later vacated.
b)
Prot a Prendre
When a landowner grants a prot a prendre to a person to remove a valuable
product of the soil (e.g., grass, asphalt, ore, etc.), the holder of the prot also
has an implied easement to pass over the surface of the land and to use it as
reasonably necessary to extract the product.
Easement by Necessity
When the owner of a tract of land sells a part of the tract and by this division
deprives one lot of access to a public road or utility line, a right-of-way by absolute
necessity is created by implied grant or reservation over the lot with access to the
public road or utility line. The owner of the servient parcel has the right to locate
3.
Prescription
Acquiring an easement by prescription is analogous to acquiring property by adverse
possession. (See V., infra.) Many of the requirements are the same: To acquire a
prescriptive easement, the use must be open and notorious, adverse, and continuous
and uninterrupted for the statutory period. Note that the public at large can acquire an
easement in private land if members of the public use the land in a way that meets the
requirements for prescription.
1)
2)
Adverse
The use must not be with the owners permission. Unlike adverse possession, the
use need not be exclusive. The user of a common driveway, e.g., may acquire a
prescriptive easement even though the owner uses it too.
3)
Continuous Use
Continuous adverse use does not mean constant use. A continuous claim of right
with periodic acts that put the owner on notice of the claimed easement fullls the
requirement. Note that tacking is permitted for prescriptive easements, just as for
adverse possession (see V.B.5.b., infra).
4)
Scope
Courts enforcing easements are often called upon to interpret the arrangement in order to
determine the scope and intended beneciaries of the interest. The key to interpretation
employed in all these cases is the reasonable intent of the original parties. What would the
parties reasonably have provided had they contemplated the situation now before the court?
What result would reasonably serve the purposes of the arrangement?
a.
If the location or scope of the permitted use is spelled out in detail, the specics will
govern, and reasonable interpretation will be excluded.
Examples:
1) In 1890, A, the owner of Blackacre, granted to B, the owner of
Whiteacre, a right-of-way over Blackacre for purposes of ingress and
egress to Whiteacre from the public highway running along the western
boundary of Blackacre. At the time of the grant, there were only horses
and buggies, no automobiles. Applying a rule of reasonableness to the
general language creating the right-of-way, a court would probably nd
that the right-of-way could today be used for cars. If, however, the use
of cars would impose a substantially greater burden on Blackacre, the
court would probably nd against this use on grounds that it was outside
the scope reasonably contemplated by A and B.
2) If, in the example just given, the right-of-way was specically
dedicated (only to the use of horses and carriages), automobile use
would be excluded. Similarly, if the right-of-way was specically located
(e.g., over the southern 10 feet of Blackacre), the rule of reasonableness could not be invoked to change or enlarge the location.
b.
Absence of Location
If an easement is created but not specically located on the servient tenement, an
easement of sufcient width, height, and direction to make the intended use reasonably
convenient will be implied. The owner of the servient tenement may select the location
of the easement so long as her selection is reasonable.
c.
Changes in Use
In the absence of specic limitations in the deed creating an easement, the courts will
assume that the easement is intended by the parties to meet both present and future
reasonable needs of the dominant tenement.
Examples:
1) A roadway easement of unspecied width was created in 1920,
when cars were only six feet wide. Today, however, cars are considerably wider. Because the original roadway easement was not specically
limited in width, the easement will expand in size to accommodate the
changing and expanding needs of the owner of the dominant tenement.
2) But a basic change in the nature of the use is not allowed. Thus, a
telephone or power line may not be added on the roadway. (Many courts
are more liberal in allowing such additions if the roadway easement is
public rather than private.)
d.
e.
4.
Duty to Repair
If the holder of the benet is the only party making use of the easement, that party
has the duty to make repairs (e.g., ll in potholes on a right-of-way) and, absent
a special agreement, the servient owner has no duty to do so. If the easement is
nonexclusive and both the holder of the benet and the servient owner are making
use of the easement, the court will apportion the repair costs between them on the
basis of their relative use.
f.
g.
Termination of Easements
An easement, like any other property interest, may be created to last in perpetuity or for a
limited period of time. To the extent the parties to its original creation provide for the natural
termination of the interest, such limitations will control.
a.
Stated Conditions
If the parties to the original creation of an easement set forth specic conditions upon
the happening of which the easement right will terminate, the conditions will be recognized. On this basis, the following conditions are valid: an easement granted so long
as repairs are maintained, an easement granted so long as X is the holder of the
dominant tenement, an easement granted until the dominant tenement is used for
commercial purposes, etc.
b.
Unity of Ownership
By denition, an easement is the right to use the lands of another for a special purpose.
On this basis, the ownership of the easement and of the servient tenement must be in
different persons. If ownership of the two comes together in one person, the easement is
extinguished.
1)
2)
No Revival
If complete unity of title is acquired, the easement is extinguished. Even though
there may be later separation, the easement will not be automatically revived.
Example:
A owns Lot 6, the servient tenement. B owns adjacent Lot 7. A
grants to B the privilege of crossing Lot 6, i.e., grants an easement
appurtenant to B. Assume A conveys Lot 6 to B in fee simple. The
easement would be extinguished because B then holds both the
easement and title to the servient tenement. If, thereafter, B conveys
Lot 6 to C, the easement is not revived. Of course, it could be
created anew.
c.
d.
Release
An easement may be terminated by a release given by the owner of the easement
interest to the owner of the servient tenement. A release requires the concurrence of
both owners and is, in effect, a conveyance. The release must be executed with all the
formalities that are required for the valid creation of an easement.
1)
Easement Appurtenant
The basic characteristic of an easement appurtenant is that it becomes, for the
purpose of succession, an incident of possession of the dominant tenement. This
basic characteristic requires that the easement interest not be conveyed independently of a conveyance of the dominant tenement. However, an easement appurtenant may be conveyed to the owner of the servient tenement without a conveyance
(to the same grantee) of the dominant tenement. This is an exception to the general
alienability characteristics of an easement appurtenant (see 1.b., supra).
2)
Easement in Gross
The basic characteristic of an easement in gross is that unless it is for a commercial purpose, it is inalienable. However, an easement in gross can be released; i.e.,
can be conveyed to the owner of the servient tenement. This is an exception to the
general characteristics of an easement in gross.
3)
Statute of Frauds
The Statute of Frauds requires that every conveyance of an interest in land that
has a duration long enough to bring into play a particular states Statute of Frauds
(typically one year) must be evidenced by a writing. This writing requirement is
also applicable to a release of an easement interest. If the easement interest that
is being conveyed has a duration of greater than one year, a writing is required in
order to satisfy the Statute of Frauds. An oral release is ineffective, although it may
become effective by estoppel.
Abandonment
It has become an established rule that an easement can be extinguished without conveyance where the owner of the privilege demonstrates by physical action an intention to
permanently abandon the easement. To work as an abandonment, the owner must have
manifested an intention never to make use of the easement again.
Example:
A owns Lot 6 and B owns Lot 7, which are immediately adjacent. A
grants to B an easement across Lot 6. This easement is specically
located on the servient tenement and is a walkway. Subsequently, B
constructs a house on Lot 7 that completely blocks his access to the
walkway. By the physical action of constructing the house in such a way
that access to the walkway (i.e., the easement) is denied, B has physically indicated an intent not to use the easement again. The easement is
extinguished by abandonment.
1)
e.
2)
3)
Estoppel
While the assertions of the holder of the easement are insufcient to work a termination
unless there is valid compliance with the requirements of a release, an easement may be
extinguished by virtue of the reasonable reliance and change of position of the owner of
the servient tenement, based on assertions or conduct of the easement holder.
Example:
The owner of a right-of-way tells the owner of the servient tenement that
the owner of the servient tenement may build a building on the servient
tenement in such a way as to make the right-of-way no longer usable, and
the servient owner does in fact build the building. There will be an extinguishment of the easement by estoppel.
For an easement to be extinguished by estoppel, three requirements must be satised.
Namely, there must be (i) some conduct or assertion by the owner of the easement,
(ii) a reasonable reliance by the owner of the servient tenement, (iii) coupled with a
change of position. Even though there is an assertion by the easement holder, if the
owner of the servient tenement does not change her position based upon the assertion,
the easement will not be terminated.
f.
Prescription
An easement may be extinguished, as well as created, by prescription. Long continued
possession and enjoyment of the servient tenement in a way that would indicate to the
public that no easement right existed will end the easement right. Such long continued
use works as a statute of limitations precluding the whole world, including the easement
holder, from asserting that his privilege exists.
The termination of an easement by prescription is xed by analogy to the creation of
an easement by prescription. The owner of the servient tenement must so interfere with
the easement as to create a cause of action in favor of the easement holder. The interference must be open, notorious, continuous, and nonpermissive for the statutory period
(e.g., 20 years).
g.
Necessity
Easements created by necessity expire as soon as the necessity ends.
Example:
A, the owner of a tract of land, sells a portion of it that has no access to a
highway except over the remaining lands of A. B, the purchaser, acquires
5.
h.
Condemnation
Condemnation of the servient estate will extinguish the nonpossessory interest. Courts
are split, however, on whether the holder of the benet is entitled to compensation for
the value lost.
i.
CompareLicenses
Licenses, like afrmative easements, privilege their holder to go upon the land of another
(the licensor). Unlike an afrmative easement, the license is not an interest in land. It is
merely a privilege, revocable at the will of the licensor. (Although licenses may acquire some
of the characteristics of easements through estoppel or by being coupled with an interest.)
The Statute of Frauds does not apply to licenses, and licensees are not entitled to compensation if the land is taken by eminent domain. Licenses are quite common; examples of
licensees include delivery persons, plumbers, party guests, etc.
a.
Assignability
An essential characteristic of a license is that it is personal to the licensee and therefore
not alienable. The holder of a license privilege cannot convey such right. In fact, most
courts have held that the license privilege is so closely tied to the individual parties that
it is revoked, by operation of law, upon an attempted transfer by the licensee.
b.
2)
Breach of Contract
A license may be granted pursuant to an express or implied contract between the
licensor and licensee. On this basis, the termination of the licensees privilege may
constitute a breach of contract. While many courts may grant a cause of action for
money damages for a revocation of a license in breach of contract, they continue
to sustain the licensors right to terminate the licensees privilege to continue to
remain on the servient tenement.
Example:
A pays a $70 greens fee to play 18 holes of golf on Bs property.
After A has played only nine holes, B terminates As right to be
on Bs property. Because A acquired a license and it is revocable
by its very nature, Bs action is not, in property terms, wrongful.
However, A may have a cause of action against B to recoup part or
all of As $70.
c.
d.
Irrevocable Licenses
1)
Estoppel Theory
If a licensee invests substantial amounts of money or labor in reliance on a license,
the licensor may be estopped to revoke the license, and the license will thus
become the equivalent of an afrmative easement.
Example:
A orally licenses B to come onto Blackacre to excavate a drainage
ditch connected to Bs parcel, Whiteacre. B does so at substantial
expense. A will probably be estopped to revoke the license and
prevent B from using the ditch.
Under the majority view, such irrevocable licenses or easements by estoppel last
until the owner receives sufcient benet to reimburse himself for the expenditures
made in reliance on the license. A minority of courts treat easements by estoppel
like any other afrmative easements and give them a potentially innite duration.
2)
Vendee of a Chattel
The purchaser of a chattel located upon the sellers land is, in the absence of
an express stipulation to the contrary, given the privilege to enter upon the
sellers land for the purpose of removing the chattel. The purchasers right is
irrevocable. He must, however, enter at reasonable times and in a reasonable
manner.
Example:
A, the owner of Blackacre, sells 100 crates of oranges stored
in a shed on Blackacre and at the same time licenses B to
C.
b)
Termination of Tenancy
If a tenants right to possess land has been lawfully terminated, the tenant
may still reenter the land at reasonable times and in a reasonable manner for
the purpose of removing his chattels. This is an irrevocable privilege.
c)
PROFITS
Like an easement, a prot (prot a prendre) is a nonpossessory interest in land. The holder of the
prot is entitled to enter upon the servient tenement and take the soil or a substance of the soil
(e.g., minerals, timber, oil, or game). Also, like an easement, a prot may be appurtenant or in
gross. In contrast to easements, however, there is a constructional preference for prots in gross
rather than appurtenant.
1.
Creation
Prots are created in the same way as easements.
2.
Alienability
A prot appurtenant follows the ownership of the dominant tenement. A prot in gross may
be assigned or transferred by the holder.
3.
4.
Scope
The extent and nature of the prot is determined by the words of the express grant (if there
was a grant), or by the nature of the use (if the prot was acquired by prescription). Note
that implied in every prot is an easement entitling the prot holder to enter the servient
estate to remove the resource.
Example:
A, the owner of Blackacre, grants B the right to come onto Blackacre to carry
off gravel from a pit on Blackacre. B has a prot with respect to the gravel
and also the benet of an implied afrmative easement to go onto Blackacre
by reasonable means to remove the gravel.
a.
subdivision of land with an easement appurtenant. The benet of the prot will attach
to each parcel in a subdivision only if the burden on the servient estate is not as a result
overly increased.
Example:
A, the owner of Blackacre, grants B, the owner of adjacent Whiteacre,
the right to remove rock from Blackacre. If the prot was to take the
rock for purposes of maintaining a boat launch on Whiteacre, then an
increase in use from one to 50 boat launches when Whiteacre is subdivided will probably be viewed as overburdensome to Blackacre.
If, however, the prot was to take rock for purposes of reinforcing
Whiteacres coastline to prevent erosion, apportionment would likely be
allowed because subdivision would not increase the number of acres to
be reinforced and consequently would not impose a greater burden on
Blackacre.
b.
5.
D.
Termination
Prots are terminated in the same way as easements. In addition, misuse of a prot, unduly
increasing the burden (typically through an improper apportionment), will be held to
surcharge the servient estate. The result of surcharge in this case is to extinguish the prot.
(Contrast this with the result when the benet of an afrmative easement is misused:
Improper or excessive use increasing the burden on the servient estate is enjoinable but, in
most jurisdictions, does not extinguish the easement; see B.3.g., supra.)
Intent
The covenanting parties must have intended that successors in interest to the covenantor
be bound by the terms of the covenant. The requisite intent may be inferred from
circumstances surrounding creation of the covenant, or it may be evidenced by language
in the conveyance creating the covenant (e.g., this covenant runs with the land, or
grantee covenants for herself, her heirs, successors, and assigns).
b.
Notice
Under the common law, a subsequent purchaser of land that was subject to a covenant
took the land burdened by the covenant, whether or not she had notice. However, under
American recording statutes (see VI.E., infra), if the covenant is not recorded, a bona
de purchaser who has no notice of the covenant and who records her own deed will
take free of the covenant. Hence, as a practical matter, if the subsequent purchaser
pays value and records (as will nearly always be true), she is not bound by covenants of
which she has no actual or constructive notice.
c.
Horizontal Privity
This requirement rests on the relationship between the original covenanting parties.
Specically, horizontal privity requires that, at the time the promisor entered into the
covenant with the promisee, the two shared some interest in the land independent of
the covenant (e.g., grantor-grantee, landlord-tenant, mortgagor-mortgagee).
Examples:
1) A and B are neighboring landowners, neither having any rights in
the others land. For good consideration, A promises B, for herself,
her heirs, successors, and assigns, that As parcel will never be used
for other than residential purposes. The horizontal privity requirement
is not met, and successors in interest to A will not be bound because
at the time A made this covenant, she and B shared no interest in land
independent of the covenant.
2) A, the owner of Blackacre in fee, promised B, the holder of a rightof-way easement over Blackacre, always to keep the right-of-way free
of snow or other impediment to Bs use of the right-of-way. Horizontal
privity is met because, at the time the covenant was made, A owned the
parcel in fee and B held the benet of an easement in it.
3) A, the owner of Blackacre and Whiteacre, deeds Whiteacre to B,
promising not to use Blackacre for other than residential purposes.
Horizontal privity exists here by virtue of the grantor-grantee relationship between A and B.
d.
Vertical Privity
To be bound, the successor in interest to the covenanting party must hold the entire
durational interest held by the covenantor at the time she made the covenant.
Example:
A, who owns Blackacre and Whiteacre in fee simple absolute, sells
Whiteacre to B and, in the deed, covenants for herself, her heirs, successors, and assigns to contribute one-half the expense of maintaining a
common driveway between Blackacre and Whiteacre. A then transfers
Blackacre to C for life, retaining a reversionary interest for herself. B
cannot enforce the covenant against C because C does not possess the
entire interest (fee simple absolute) held by her predecessor in interest,
A, at the time A made the promise.
e.
Negative Covenants
For the burden of a negative covenant to touch and concern the land, the covenant
must restrict the holder of the servient estate in his use of that parcel of land.
Examples:
1) A, who owned Blackacre and Whiteacre, covenanted with B, the
grantee of Whiteacre, that she would not erect a building of over
two stories on Blackacre. The burden of the covenant touches and
concerns Blackacre because it diminishes As rights in connection
with her enjoyment of Blackacre.
2) A, who owned Blackacre and Whiteacre, covenanted with B,
the grantee of Whiteacre, that she would never operate a shoe store
within a radius of one mile of Whiteacre. The covenant does not
touch and concern Blackacre because its performance is unconnected to the enjoyment of Blackacre.
2)
Afrmative Covenants
For the burden of an afrmative covenant to touch and concern the land, the
covenant must require the holder of the servient estate to do something, increasing
her obligations in connection with enjoyment of the land.
Examples:
1) A, who owned Blackacre and Whiteacre, covenanted with B,
the grantee of Whiteacre, to keep the building on Blackacre in
good repair. The covenant touches and concerns Blackacre because
it increases As obligations in connection with her enjoyment of
Blackacre.
2) A owned Blackacre and Whiteacre, which were several miles
apart. A covenanted with B, the grantee of Whiteacre, to keep the
building on Whiteacre in good repair. The covenant does not touch
and concern Blackacre because its performance is unconnected to
the use and enjoyment of Blackacre.
3) A, the grantee of a parcel in a residential subdivision, covenants
to pay an annual fee to a homeowners association for the maintenance of common ways, parks, and other facilities in the subdivision. At one time, it was thought that such covenants, because
physically unconnected to the land, did not touch and concern. The
prevailing view today is that the burden will run because the fees
are a charge on the land, increasing As obligations in connection
with the use and enjoyment of it. (See 4.a., infra.)
2.
Intent
The covenanting parties must have intended that the successors in interest to the
covenantee be able to enforce the covenant. Surrounding evidence of intent, as well as
language in the instrument of conveyance, is admissible.
b.
Vertical Privity
The benet of a covenant runs to the assignees of the original estate or of any lesser
estate (e.g., a life estate). The owner of any succeeding possessory estate can enforce
the benet at law. In the majority of states today, horizontal privity is not required for
the benet to run. As a consequence, if horizontal privity is missing, the benet may
run to the successor in interest to the covenantee even though the burden is not enforceable against the successor in interest of the covenantor.
Example:
A, who owns Blackacre, covenants with her neighbor, B, who owns
Whiteacre, that A, her successors, and assigns will keep the building on
Blackacre in good repair. Horizontal privity is missing. B then conveys
Whiteacre, the dominant estate, to C. C can enforce the benet of the
afrmative covenant against A because horizontal privity is not needed
for the benet to run. If, however, A conveys Blackacre to D, neither
B nor C could enforce the covenant against D, for horizontal privity is
required for the burden to run.
c.
3.
of the burden, and further discards the requirement of vertical privity for running of
both the burden and the benet. Instead, the Restatement draws a distinction between
afrmative and negative covenants. Negative covenants are treated like easements,
which run to successors because they are interests in land. The burdens and benets of
afrmative covenants run to persons who succeed to an estate of the same duration as
owned by the original parties, including in most cases an adverse possessor. But afrmative covenants do not run to persons who hold lesser estates than those held by the
original parties to the covenant. Special rules are set forth for when afrmative burdens
run to lessees and life tenants, and when they can enforce the benets. [Restatement
(Third) of Property: Servitudes 5.2 - 5.5]
b.
4.
b.
c.
5.
RemediesDamages
A breach of a real covenant generally is remedied by an award of money damages. If
equitable relief, such as an injunction, is sought, the promise may be enforced as an equitable
servitude (see E., infra). Note that a real covenant gives rise to personal liability only. The
damages are collectible out of the defendants general assets.
6.
Termination
As with all other nonpossessory interests in land, a real covenant may be terminated by: (i)
the holder of the benet executing a release in writing; (ii) merger (fee simple title to both
the beneted and burdened land comes into the hands of a single owner); and (iii) condemnation of the burdened property. (See B.4.b., c., h., supra.)
E.
EQUITABLE SERVITUDES
If a plaintiff wants an injunction or specic performance, he may show that the covenant qualies as an equitable servitude. An equitable servitude is a covenant that, regardless of whether it
runs with the land at law, equity will enforce against the assignees of the burdened land who have
notice of the covenant. The usual remedy is an injunction against violation of the covenant.
1.
Creation
Generally, equitable servitudes are created by covenants contained in a writing that satises the Statute of Frauds. As with real covenants, acceptance of a deed signed only by the
grantor is sufcient to bind the grantee as promisor. There is one exception to the writing
requirement: Negative equitable servitudes may be implied from a common scheme for
development of a residential subdivision.
a.
Common Scheme
Reciprocal negative covenants will be implied only if at the time that sales of
parcels in the subdivision began, the developer had a plan that all parcels in the
subdivision be developed within the terms of the negative covenant. If the scheme
arises after some lots are sold, it cannot impose burdens on the lots previously sold
without the express covenants. The developers common scheme may be evidenced
by a recorded plat, by a general pattern of prior restrictions, or by oral representations, typically in the form of statements to early buyers that all parcels in the
development will be restricted by the same covenants that appear in their deeds.
On the basis of this scheme, it is inferred that purchasers bought their lots relying
on the fact that they would be able to enforce subsequently created equitable servitudes similar to the restrictions imposed in their deeds.
2)
2.
Notice
To be bound by the terms of a covenant that does not appear in his deed, a
grantee must, at the time he acquired his parcel, have had notice of the covenants
contained in the deeds of other buyers in the subdivision. The requisite notice may
be acquired through actual notice (direct knowledge of the covenants in the prior
deeds); inquiry notice (the neighborhood appears to conform to common restrictions); or record notice (if the prior deeds are in the grantees chain of title he will,
under the recording acts, have constructive notice of their contents).
Enforcement
For successors of the original promisee and promisor to enforce an equitable servitude,
certain requirements must be met.
a.
Intent
The covenanting parties must have intended that the servitude be enforceable
by and against assignees. No technical words are required to express this intent.
In fact, the intent may be ascertained from the purpose of the covenant and the
surrounding circumstances.
2)
Notice
A subsequent purchaser of land burdened by a covenant is not bound by it in equity
unless she had actual or constructive notice of it when she acquired the land. This
rule is part of the law of equitable servitudes, and exists apart from the recording
acts.
3)
b.
c.
because horizontal privity is missing. However, under an equitable servitude theory, the burden will run, and an injunction will issue against
other than residential uses.
3) Same as above, but A transfers only a life estate to C. Again, the
burden would not run at law because of the absence of vertical privity.
The burden would, however, be enforceable as an equitable servitude.
d.
3.
Unclean Hands
A court will not enforce a servitude if the person seeking enforcement is violating a
similar restriction on his own land. This defense will apply as long as the plaintiffs
violation is of the same general nature.
b.
Acquiescence
If a beneted party acquiesces in a violation of the servitude by one burdened party,
he may be deemed to have abandoned the servitude as to other burdened parties.
(Equitable servitudes, like easements, may be abandoned.) Note that this defense will
not apply if the prior violation occurred in a location so distant from the complainant
that it did not really affect his property.
c.
Estoppel
If the beneted party has acted in such a way that a reasonable person would believe
that the covenant was abandoned or waived, and the burdened party acts in reliance
thereon, the beneted party will be estopped to enforce the covenant. Similarly, if the
beneted party fails to bring suit against a violator within a reasonable time, the action
may be barred by laches.
d.
Zoning
Zoning plays an important role in determining whether changed conditions will
be allowed as a defense to enforcement of an equitable servitude. Zoning that is
inconsistent with the private restriction imposed by the equitable servitude will
not of itself bar the injunction, but it will provide good evidence that neighborhood
conditions have changed sufciently to make the injunction unjust. Thus, in the
example above, the position of B or her successors would be fortied by a showing
that the area in which Blackacre is situated is presently zoned for commercial uses.
2)
F.
Termination
Like other nonpossessory interests in land, an equitable servitude may be terminated by a
written release from the benet holder(s), merger of the beneted and burdened estates, or
condemnation of the burdened property. (See B.4.b., c., h., supra.)
Creation
While a written agreement is required by the Statute of Frauds for the express creation of
a party wall or common driveway agreement, an irrevocable license can arise if there has
been detrimental reliance on a parol agreement. Party walls and common driveways can also
result from implication or prescription.
2.
Running of Covenants
If party wall or common driveway owners agree to be mutually responsible for maintaining
the wall or driveway, the burdens and benets of these covenants will run to successive owners of each parcel. The cross-easements for support satisfy the requirement of
horizontal privity because they are mutual interests in the same property. Each promise
touches and concerns the adjoining parcels, and the grantee will be charged with notice of
the covenant because of the visibility of the common wall or driveway.
V. ADVERSE POSSESSION
A.
IN GENERAL
Title to real property may be acquired by adverse possession. (Easements may also be acquired
by prescription.) Gaining title by adverse possession results from the operation of the statute of
limitations for ejectment, or recovery of real property. If an owner does not, within the statutory period, take legal action to eject a possessor who claims adversely to the owner, the owner
is thereafter barred from bringing suit for ejectment. Moreover, title to the property vests in the
possessor.
B.
REQUIREMENTS
To establish title by adverse possession, the possessor must show (i) an actual entry giving
exclusive possession that is (ii) open and notorious, (iii) adverse (hostile), and (iv) continuous
throughout the statutory period.
1.
Running of Statute
The statute of limitations begins to run when the claimant goes adversely into possession of
the true owners land (i.e., the point at which the true owner could rst bring suit). The ling
of suit by the true owner is not sufcient to stop the period from running; the suit must be
pursued to judgment. However, if the true owner les suit before the statutory period (e.g.,
20 years) runs out and the judgment is rendered after the statutory period, the judgment will
relate back to the time that the complaint was led.
2.
b.
3.
the land. The adverse possessors occupation must be sufciently apparent to put the true
owner on notice that a trespass is occurring.
Examples:
1) Water Company runs a pipe under Owners land, and there is no indication
of the pipes existence from the surface of the land. Water Company cannot
gain title by adverse possession because there is nothing to put Owner on
notice of the trespass.
2) As use of Bs farmland for an occasional family picnic will not satisfy the
open and notorious requirement because picnicking is not necessarily an act
consistent with the ownership of farmland.
4.
Hostile
The possessors occupation of the property must be hostile (adverse). This means merely
that the possessor does not have the true owners permission to be on the land. It does not
mean anger or animosity. The state of mind of the adverse possessor is irrelevant. By the
large majority view, it does not matter whether the possessor believes she is on her own land,
knows she is trespassing on someone elses land, or has no idea who owns the land.
a.
b.
Co-TenantsOuster Required
Possession by one co-tenant is not ordinarily adverse to her co-tenants because each
co-tenant has a right to the possession of all the property. Thus, sole possession or use
by one co-tenant is not adverse, unless there is a clear repudiation of the co-tenancy;
e.g., one co-tenant ousts the others or makes an explicit declaration that he is claiming
exclusive dominion over the property.
c.
d.
1)
5.
6.
Establishment Requirement
The establishment requirement can be implied by acquiescence. A past dispute is
not necessary to show uncertainty, although it can be good evidence of it. But a
showing of original uncertainty is required; otherwise, in a courts view, a parol
transfer of land would result.
Continuous Possession
The adverse claimants possession must be continuous throughout the statutory period.
Continuous possession requires only the degree of occupancy and use that the average owner
would make of the property.
a.
b.
Tacking Permitted
There need not be continuous possession by the same person. Ordinarily, an adverse
possessor can take advantage of the periods of adverse possession by her predecessor.
Separate periods of adverse possession may be tacked together to make up the full
statutory period with the result that the nal adverse possessor gets title, provided there
is privity between the successive adverse holders.
1)
Privity
Privity is satised if the subsequent possessor takes by descent, by devise, or
by deed purporting to convey title. Tacking is not permitted where one adverse
claimant ousts a preceding adverse claimant or where one adverse claimant
abandons and a new adverse claimant then goes into possession.
2)
Formalities on Transfer
Even an oral transfer of possession is sufcient to satisfy the privity requirement.
Example:
A received a deed describing Blackacre, but by mistake built a
house on an adjacent parcel, Whiteacre. A, after pointing the house
out to B and orally agreeing to sell the house and land to her,
conveyed to B, by a deed copied from her own deed, describing the
property as Blackacre. The true owner of Whiteacre argues that
there was no privity between A and B because the deed made no
reference to Whiteacre, the land actually possessed. Nonetheless,
the agreed oral transfer of actual possession is sufcient to permit
tacking.
C.
DISABILITY
1.
2.
O, the true owner, is declared insane six months after A begins using a
pathway adversely. The statute has begun to run because Os disability arose
after As adverse use began.
No Tacking of Disabilities
Only a disability of the owner existing at the time the cause of action arose is considered.
Thus, disabilities of successors in interest or subsequent additional disabilities of the owner
have no effect on the statute.
Examples:
1) O is a minor at the time A goes into adverse possession of Os land. One
year before O reaches the age of majority, O is declared insane (a subsequent
disability). The statute begins to run from the date O reaches the age of
majority, whether she is then sane or insane.
2) O, the true owner, is insane when A begins an adverse use. Ten years later,
O dies intestate and the land goes to her heir, H, who is then 10 years old. The
statute of limitations begins to run upon Os death despite Hs minority. Hs
minority is a supervening disability and cannot be tacked to Os.
3.
D.
2.
E.
F.
VI. CONVEYANCING
A.
party to be charged. The writing need not be a formal contract; a memorandum sufces
e.g., escrow instructions or e-mails can be contracts of sale. The Statute of Frauds requires
that the writing contain all essential terms of the contract. These are: (i) a description of
the property (see B.3., infra), (ii) identication of the parties to the contract, and (iii) the
price and manner of payment (if agreed upon). Incidental matters (e.g., prorating of taxes,
furnishing of deeds, title insurance, etc.) can be determined by custom; they need not appear
in the writing nor even have been agreed upon.
a.
2)
Evidentiary Theory
Courts state that if acts done by a party can be explained only by reference
to an agreement, these acts unequivocally establish the existence of an oral
contract.
b)
Evidentiary Theory
Under the evidentiary theory, it is immaterial who performed the acts constituting the part performance. Because they refer unequivocally to a contract,
the seller may obtain specic performance based on the buyers acts.
b)
make sure that you ascertain whether the seller has done anything that would
cause him a hardship if the Statute of Frauds were successfully asserted by
the buyer.
2.
Risk of Loss
If the property is destroyed (without fault of either party) before the date set for closing,
the majority rule is that, because the buyer is deemed the owner of the property, the risk
of loss is on the buyer. Thus, the buyer must pay the contract price despite a loss due
to re or other casualty, unless the contract provides otherwise. Some states, however,
have adopted the Uniform Vendor and Purchaser Risk Act, which places the risk on the
seller unless the buyer has either legal title or possession of the property at the time of
the loss.
1)
b.
Casualty Insurance
Suppose the buyer has the risk of loss, as is true under the majority view, but
the seller has re or casualty insurance that covers the loss. In the event of loss,
allowing the seller to recover the full purchase price on the contract and to collect
the insurance proceeds would be unjust enrichment. Hence, the courts require
the seller to give the buyer credit, against the purchase price, in the amount of the
insurance proceeds.
Death of Seller
If the seller dies, the bare legal title passes to the takers of his real property,
but they must give up the title to the buyer when the contract closes. When the
purchase price is paid, the money passes as personal property to those who take
the sellers personal property. Note that if the property is specically devised, the
specic devisee may take the proceeds of the sale. (See F.1.b., infra.)
2)
Death of Buyer
If the buyer dies, the takers of his real property can demand a conveyance of the
land at the closing of the contract. Moreover, under the traditional common law
rule, they are entitled to exoneration out of the personal property estate (see F.2.,
infra). Thus, the takers of his personal property will have to pay the purchase price
out of their share of the buyers estate. However, a majority of states have enacted
statutes abolishing the doctrine of exoneration, and in those states the takers of the
real property will take it subject to the vendors lien for the purchase price. In these
states, as a practical matter, the takers of the real property will have to pay the
price unless the testator specically provided to the contrary.
3.
Marketable Title
There is an implied covenant in every land sale contract that at closing the seller will provide
the buyer with a title that is marketable.
a.
Adverse Possession
Historically, a title acquired by adverse possession was not considered
marketable because the purchaser might be later forced to defend in court the
facts that gave rise to the adverse possession against the record owner. On
the bar exam, title acquired by adverse possession is unmarketable, despite
the fact that most modern cases are contra. Most of the modern cases hold
adverse possession titles to be marketable if: (i) the possession has been for a
very lengthy period; (ii) the risk that the record owner will sue appears to be
very remote; and (iii) the probability of the record owners success in such a
suit appears to be minimal. Because the bar examiners have yet to recognize
this line of cases, the modern view should be considered only as a fallback
position on the bar exam.
b)
Encumbrances
Generally, mortgages, liens, easements, and covenants render title unmarketable
unless the buyer waives them.
a)
b)
Easements
An easement that reduces the value of the property (e.g., an easement of way
for the benet of a neighbor) renders title unmarketable. The majority of
courts, however, have held that a benecial easement (e.g., utility easement
to service property) or one that was visible or known to the buyer does not
constitute an encumbrance. Some courts go so far as to hold that the buyer is
deemed to have agreed to take subject to any easement that was notorious or
known to the buyer when she entered into the contract.
c)
Covenants
Restrictive covenants render title unmarketable.
d)
Encroachments
A signicant encroachment constitutes a title defect, regardless of whether an
adjacent landowner is encroaching on the sellers land or vice versa. However,
the encroachment will not render title unmarketable if: (i) it is very slight
(only a few inches) and does not inconvenience the owner on whose land it
encroaches; (ii) the owner encroached upon has indicated that he will not sue
on it; or (iii) it has existed for so long (many decades) that it has become legal
by adverse possession, provided that the state recognizes adverse possession
titles as being marketable (see 1)a), supra).
3)
Zoning Restrictions
Generally, zoning restrictions do not affect the marketability of title; they are not
considered encumbrances. An existing violation of a zoning ordinance, however,
does render title unmarketable.
4)
Waiver
Any of the above-mentioned title defects can be waived in the contract of sale.
b.
c.
Time of Marketability
If, as is usual, the seller has agreed to furnish title at date of closing, the buyer cannot
rescind prior to that date on grounds that the sellers title is not marketable.
1)
d.
2)
Merger
If the buyer permits the closing to occur, the contract is said to merge with the
deed (i.e., it disappears) and, in the absence of fraud, the seller is no longer liable
on the implied covenant of marketable title. However, the buyer may have an
action for violation of promises made in the deed, if any (see D., infra). Note: The
merger rule does not apply to most nontitle matters, such as covenants regarding
the physical condition of the property. [Campbell v. Rawls, 381 So. 2d 744 (Fla.
1980)]
4.
5.
Time of Performance
a.
b.
2)
The circumstances indicate it was the parties intention; e.g., the land is rapidly
uctuating in value or a party must move from out of town and has no other place
to go; or
3)
One party gives the other notice that she desires to make time of the essence, and
does so within a reasonable time prior to the date designated for closing.
c.
d.
Tender of Performance
In general, the buyers obligation to pay the purchase price and the sellers obligation to
convey the title are deemed to be concurrent conditions. This means that neither party is in
breach of the contract until the other party tenders her performance, even if the date designated for the closing has passed.
a.
b.
c.
6.
Damages
The usual measure of damages is the difference between the contract price and the
market value of the land on the date of the breach. Incidental damages, such as title
examination and moving or storage costs, can also be recovered.
1)
b.
Liquidated Damages
Sales contracts usually require the buyer to deposit earnest money with the
seller, and provide that if the buyer defaults in performance, the seller may retain
this money as liquidated damages. The courts routinely uphold the sellers retention of the deposit if the amount appears to be reasonable in light of the sellers
anticipated and actual damages. Many courts will uphold a retention of a deposit of
up to 10% of the sales price without further inquiry into its reasonableness. Even
without a liquidated damages clause, courts may uphold retention of the deposit, on
the ground that giving restitution of the funds to the buyer would unjustly reward a
party in breach.
Specic Performance
1)
Buyers Remedy
A court of equity will order a seller to convey the title if the buyer tenders the
purchase price. The remedy at law (damages) is deemed inadequate because the
buyer is getting land and land is unique.
If the seller cannot give marketable title, but the buyer wishes to proceed with the
transaction, she can usually get specic performance with an abatement of the
purchase price in an amount reecting the title defect.
2)
c.
7.
Sellers Remedy
Somewhat illogically, the courts also generally will give a specic performance
decree for the seller if the buyer is in breach. This is sometimes explained as
necessary to have mutuality of remedy. A few courts in recent years have refused
to award specic performance to sellers if the property is not unique (e.g., if a
developer is selling a house in a large subdivision of similar houses).
the purpose intended. One exception is a contract for the sale of a residential building
under construction or to be constructed, on the ground that the buyer has no opportunity
to inspect. Most courts extend the implied warranty of tness or quality to the sale of
any new house by the builder. The warranty implied is that the new house is designed
and constructed in a reasonably workmanlike manner and suitable for human habitation. Courts are split, however, on whether a subsequent purchaser can recover from the
original builder because of the lack of privity. [See Speight v. Walters, 744 N.W.2d 108
(Iowa 2008); Conway v. Cutler Group, Inc., 2014 WL 4064261 (Pa. 2014)]
b.
Negligence of Builder
A person who contracts for construction may always sue a builder for negligence in
performing a building contract. Moreover, many courts now permit the ultimate vendee
(e.g., a subdivision buyer) to sue the builder despite the fact that the seller hired the
builder and the buyer thus lacks privity.
c.
Misrepresentation (Fraud)
This theory requires proof that the seller made a false statement of fact (oral or
written) to the buyer, that the buyer relied on the statement, and that it materially affected the value of the property. The seller must either have known that the
statement was false, or have made it negligently (without taking reasonable care to
determine its truth).
2)
Active Concealment
The seller is liable as above, even without making any statement, if the seller took
steps to conceal a defect in the property (e.g., paneling over a wall to conceal
cracks).
3)
Failure to Disclose
A majority of states now hold sellers liable for failure to disclose defects if the
following factors are present:
(i)
(ii) The defect is not obvious or apparent, and the seller realizes that the buyer is
unlikely to discover it by ordinary inspection; and
(iii) The defect is serious and would probably cause the buyer to reconsider the
purchase if it were known.
These decisions are more likely to impose liability on the seller if the property is
a personal residence, if the defect is dangerous, and if the seller personally created
the defect or previously attempted to repair it and failed to do so.
d.
B.
Disclaimers of Liability
Sellers sometimes attempt to avoid liability for property defects by inserting clauses in
sales contracts exculpating the seller.
1)
As Is Clauses
A general clause, such as property sold as is or with all defects, is not sufcient to overcome a sellers liability for fraud, concealment, or (in the states that
recognize it) failure to disclose.
2)
Specic Disclaimers
If the exculpatory clause identies and disclaims liability for specic types of
defects (e.g., seller is not liable for leaks in the roof), it is likely to be upheld.
8.
9.
Title Insurance
A title insurance policy insures that a good record title of the property exists as of the
policys date and agrees to defend the record title if litigated. The insurance can be taken out
by either the owner of the property or the mortgage lender. An owners policy protects only
the person who owns the policy (i.e., the property owner or the mortgage lender) and does
not run with the land to subsequent purchasers. In contrast, a lenders policy follows any
assignment of the mortgage loan.
Formalities
a.
Statute of Frauds
The Statute of Frauds requires that a deed be in writing and signed by the grantor.
b.
with the identity of the grantee left blank, the courts will presume that the person taking
delivery has authority to ll in the name of the grantee, and if she does so, the deed is
valid. But if the land description is left blank, no such authority is presumed, and the
deed is void unless the grantee was explicitly given authority to ll in the description,
and did so.
2.
c.
Words of Intent
The deed must evidence an intention to transfer realty, but technical words are unnecessary. The word grant by itself is sufcient in many states.
d.
e.
Seal Is Unnecessary
A seal is unnecessary.
f.
g.
Signature
A deed must be signed by the grantor. The grantor may designate an agent to sign on
the grantors behalf, but if the signing is not done in the grantors presence, the Statute
of Frauds generally requires that the agents authority be written. In the case of deeds
by corporations, statutes usually provide for execution by two ofcers of the corporation and the afxing of the corporations seal. If the deed represents a conveyance of
all or a substantial part of the corporations assets, a resolution of the board of directors
approving the transfer may be necessary. The grantees signature is not necessary even
if the deed contains covenants on her part. Her acceptance of the deed (called a deed
poll when signed only by the grantor) is sufcient to make the covenants enforceable.
Void Deeds
Deeds considered void include those that are forged, were never delivered, were
issued to a nonexistent grantee (e.g., a grantee who is in fact dead at the time of
delivery, or a corporation that has not yet been legally formed), or were obtained
by fraud in the factum (i.e., the grantor was deceived and did not realize that he
was executing a deed).
2)
b.
3.
Voidable Deeds
Deeds considered voidable include those executed by persons younger than the age
of majority or who otherwise lack capacity (e.g., because of insanity), and deeds
obtained through fraud in the inducement, duress, undue inuence, mistake, and
breach of duciary duty.
Fraudulent Conveyances
Even when a deed complies with the required formalities mentioned above, it may be
set aside by the grantors creditors if it is a fraudulent conveyance. Under the Uniform
Fraudulent Transfer Act, which nearly all states have adopted, a conveyance is fraudulent if it was made: (i) with actual intent to hinder, delay, or defraud any creditor of
the grantor; or (ii) without receiving a reasonably equivalent value in exchange for the
transfer, and the debtor was insolvent or became insolvent as a result of the transfer.
However, the deed will not be set aside as against any grantee who took in good faith
and paid reasonably equivalent value.
b.
c.
1)
d.
e.
CompareInadequate Description
If, however, A grants to B my house in San Francisco, and it turns out that A
owns three houses in that city, the conveyance would probably fail for lack of
adequate description. (But note: If there is an underlying original agreement in
which there was no mistake or ambiguity, and the only mistake was in the writing
of the instrument, relief might be available by way of reformation of the deed.)
Rules of Construction
Where there is a mistake or inconsistency in the description (as where the deed leaves
in doubt the exact location of a property line, or measurements give two different
locations for the line), the following rules of construction are applied to carry out the
parties probable intent. (These are not rules of law and will not be applied where
there is clear evidence showing a contrary intent.)
1)
2)
Articial monuments (e.g., stakes, buildings, etc.) prevail over all but natural
monuments.
3)
Courses (e.g., angles) prevail over distances (e.g., west 90 degrees to Main St.
prevails over west 100 feet to Main St.).
4)
All of the foregoing prevail over general descriptions such as name (e.g., Walkers
Island) or quantity (e.g., being 300 acres).
2)
f.
C.
b)
c)
Reformation of Deeds
Reformation is an equitable action in which the court rewrites the deed to make it
conform to the intention of the parties. It will be granted if the deed does not express
what the parties agreed to, either because of their mutual mistake or a scriveners
(drafters) error. It will also be granted for unilateral mistake, but only if the party who
is not mistaken induced the mistake by misrepresentation or some other inequitable
conduct. If the property has passed to a bona de purchaser who relied on the original
language of the deed, the court will not reform it.
DeliveryIn General
A deed is not effective to transfer an interest in realty unless it has been delivered. Physical
transfer of a deed is not necessary for a valid delivery. Nor does physical transfer alone establish delivery (although it might raise a presumption thereof). Rather, delivery refers to the
grantors intent; it is satised by words or conduct evidencing the grantors intention that
the deed have some present operative effect; i.e., that title pass immediately and irrevocably,
even though the right of possession may be postponed until some future time.
Examples:
1) O drafts an instrument conveying Blackacre to A and hands the instrument
to A for safekeeping. Although handed to the named grantee, this is not a
valid delivery. There is no evidence that O intended the instrument to have
any present operative effect.
2) O drafts an instrument conveying Blackacre to A. O attempts to give the
instrument to A personally but is unable to nd A; nevertheless, O quits
possession of Blackacre and thereafter treats A as the owner thereof. Nearly
all courts would hold that there has been a sufcient delivery.
Under some circumstances (i.e., when a third party is involved), conditional delivery is
permissible. This type of delivery becomes effective only upon the occurrence of a condition,
but the transfer then relates back to the time of the conditional delivery. The grantor has
only limited rights to revoke prior to the occurrence of the condition. (See further discussion
of conditional deliveries, 3., infra.)
a.
Manual Delivery
The delivery requirement will be satised where the grantor physically or manually
delivers the deed to the grantee. Manual delivery may be accomplished by means of
the mails, by the grantors agent or messenger, or by physical transfer by the grantors
attorney in the grantors presence.
b.
c.
d.
Parol Evidence
1)
2)
3)
b)
2.
3.
Comment
Obviously, the above rules give the courts exibility to nd either delivery or
nondelivery in many situations. It is also evident that there exists a theoretical
inconsistency in admitting parol evidence to show that no delivery was intended,
but not to show that delivery was conditional. This inconsistency has been criticized by numerous commentators.
b.
No RecordingTitle Passes
If the grantor executes and delivers a deed but fails to have it recorded, title passes.
Therefore, an agreement between the grantor and grantee to the effect that the deed will
not be recorded until some event takes place in the future does not affect the passage of
title.
c.
d.
e.
TestRelinquishment of Control
To make an effective delivery, the grantor must relinquish absolute and unconditional
control.
situations should be distinguished: (i) where the grantor gives the deed to a third party, there
being no conditions appended; (ii) where the grantor in a commercial context gives such
a deed to a third party, there being conditions appended; and (iii) where the situation is the
same as in (ii), but the transaction is donative.
a.
b.
2)
3)
4)
Estoppel Cases
A few cases have held that where the escrow holder was chosen by the
grantor, the grantor is bound by the escrow holders acts and is estopped to
deny a valid delivery and passage of title to the grantee. Thus, an innocent
purchaser (BFP) from the grantee may acquire good title. An important
factor is whether the grantor has allowed the grantee to take possession of the
property prior to completion of the conditions of the escrow. If the grantor
has remained in possession, the purchaser may be held to have notice of the
grantors interest and cannot be a BFP.
Relation-Back Doctrine
In an escrow transaction, title does not pass to the grantee until performance of
the named conditions. However, where justice requires, the title of the grantee
will relate back to the time of the deposit of the deed in escrow. Generally, the
relation-back doctrine will be applied if:
(i)
The grantor dies (doctrine applied to avoid the rule that title must pass before
death if instrument is not a will);
(ii) The grantor becomes incompetent (doctrine applied to avoid the rule that an
incompetent cannot convey title); or
(iii) A creditor of the grantor (who is not a BFP or mortgagee) attaches the
grantors title (doctrine applied to cut off the creditors claim).
a)
b)
2)
4.
Acceptance
a.
Usually Presumed
There must be an acceptance by the grantee in order to complete a conveyance. In most
states, acceptance is presumed if the conveyance is benecial to the grantee (whether
or not the grantee knows of it). In other states, acceptance is presumed only where the
grantee is shown to have knowledge of the grant and fails to indicate rejection of it.
Acceptance is presumed in all states if the grantee is an infant or an incompetent.
b.
5.
D.
Dedication
Land may be transferred to a public body (e.g., a city or county) by dedication. An offer of
dedication may be made by written or oral statement, submission of a map or plat showing
the dedication, or opening the land to public use. An acceptance by the public agency is
necessary. This may be accomplished by a formal resolution, approval of the map or plat, or
actual assumption of maintenance or construction of improvements by the agency.
Usual Covenants
A grantor may give any or all of the following covenants, which are classied as the
usual covenants for title. A deed containing such covenants is called a general
warranty deed.
1)
Covenant of Seisin
The covenant of seisin is a covenant that the grantor has the estate or interest that
she purports to convey. Both title and possession at the time of the grant are necessary to satisfy the covenant.
2)
3)
4)
b.
5)
Covenant of Warranty
The covenant of warranty is a covenant wherein the grantor agrees to defend on
behalf of the grantee any lawful or reasonable claims of title by a third party, and
to compensate the grantee for any loss sustained by the claim of superior title. This
covenant is generally considered to be similar to the covenant for quiet enjoyment.
6)
7)
Breach of Covenants
Three of the covenants (seisin, right to convey, against encumbrances) are present
covenants and are breached, if at all, at the time of conveyance. Quiet enjoyment,
warranty, and further assurances are future covenants and are breached only upon
interference with the possession of the grantee or her successors. This distinction
is important in that it determines when the statute of limitations begins running and
whether a remote grantee of the covenantor can sue.
1)
2)
c.
b)
Requirement of Notice
The covenantor is not liable on her covenant of warranty or of further assurances unless the party seeking to hold her liable gives her notice of the claim
against the title she conveyed.
c)
Many states permit C to recover to the extent of the consideration received by the
defendant-covenantor (even though it exceeds the consideration paid by C). Under this
view, defendant-shopping is advisable (to sue whomever received most). The defendant who is held liable then has a cause of action against any prior covenantor, until
ultimately O is held liable. In other states, C can recover only the actual consideration
she paid (but not to exceed the amount received by the defendant-covenantor).
2.
3.
Quitclaim Deeds
A quitclaim deed is basically a release of whatever interest, if any, the grantor has in the
property. Hence, the use of covenants warranting the grantors title is basically inconsistent
with this type of deed; i.e., if the deed contains warranties, it is not a quitclaim deed.
4.
Estoppel by Deed
If a grantor purports to convey an estate in property that she does not then own, her subsequent acquisition of title to the property will automatically inure to the benet of the
grantee. In other words, the grantor impliedly covenants that she will convey title immediately upon its acquisition.
Example:
On Day 1, A, who does not own Blackacre, purports to convey Blackacre to B
by general warranty deed. On Day 1, B has no interest in Blackacre. On Day
3, A acquires Blackacre from O. That interest automatically passes to B, so
that on Day 3 B owns Blackacre. As warranties will prevent her from denying
ownership when she executed the deed on Day 1.
a.
b.
E.
Remedies of Grantee
In jurisdictions following the estoppel rationale, the original grantee, at her election,
may accept title to the land or sue for damages for breach of covenants for title.
However, if an innocent purchaser of the after-acquired title is involved, the grantee has
no rights against the BFP.
RECORDING
At common law, in nearly all cases priority was given to the grantee rst in time. Thus, if O
conveyed Blackacre to A and then made an identical conveyance to B, A prevailed over B on the
theory that after the rst conveyance O had no interest left to convey.
1.
Purpose of RecordationNotice
Recordation is not essential to the validity of a deed, as between the grantor and
grantee. However, if a grantee does not record her instrument, she may lose out against
a subsequent BFP. By recording, the grantee gives constructive (or record) notice to
everyone. Hence, as stated earlier, proper recording prevents anyone from becoming a
subsequent BFP.
b.
c.
1)
2)
Mechanics of Recording
1)
Filing Copy
The grantee or her agent normally presents the deed to the county recorder, who
photographs it and les the copy in the ofcial records. These records are kept
chronologically.
2)
2.
Indexing
The recorder also indexes the deed to permit title searches. The usual indexes
are the grantor-grantee and grantee-grantor indexes, which are arranged by reference to the parties to the conveyance. Tract indexes, which index the property by
location, exist in some urban localities.
Notice Statutes
Under a notice statute, a subsequent BFP (i.e., a person who gives valuable consideration and has no notice of the prior instrument) prevails over a prior grantee who failed
to record. The important fact under a notice statute is that the subsequent purchaser had
no actual or constructive notice at the time of the conveyance. Constructive notice
includes both record notice and inquiry notice (see 3.b.3), infra). A typical notice statute
provides:
A conveyance of an interest in land, other than a lease for less than one year,
shall not be valid against any subsequent purchaser for value, without notice
thereof, unless the conveyance is recorded.
Note also that the subsequent BFP is protected, regardless of whether she records at all.
Example:
b.
Race-Notice Statutes
Under a race-notice statute, a subsequent BFP is protected only if she records before
the prior grantee. Rationale: The best evidence of which deed was delivered rst is to
determine who recorded rst. To obviate questions about the time of delivery and to add
an inducement to record promptly, race-notice statutes impose on the BFP the additional
requirement that she record rst. A typical race-notice statute provides:
Any conveyance of an interest in land, other than a lease for less than one
year, shall not be valid against any subsequent purchaser for value, without
notice thereof, whose conveyance is rst recorded.
Example:
c.
3.
Race Statutes
Under a pure race statute, whoever records rst wins. Actual notice is irrelevant. The
rationale is that actual notice depends upon extrinsic evidence, which may be unreliable.
Very few states have race statutes.
Example:
On January 1, O conveys Blackacre to A. A does not record. On January
15, O conveys Blackacre to B. B knows of the deed to A. B records.
Then A records. B prevails over A because she recorded rst. It is
immaterial that she had actual notice of As interest.
(ii) Take without notice (actual, constructive, or inquiry) of the prior instrument; and
(iii) Pay valuable consideration.
Note: If these requirements are not met, the person is not protected by the recording acts, so
that the common law rule of rst in time prevails.
Example:
O, the owner of Blackacre, executes a contract of sale of the land to A on
Monday. A immediately records the contract. On Tuesday, O deeds the land
to B. B pays valuable consideration for the land, but is not a BFP because B is
held to have constructive notice of As rights. Result: A is entitled to enforce
the contract against B, paying B the rest of the price and compelling B to
deliver a deed to A. (If A had failed to record the contract, and B had no other
notice of it, B would have taken free of As contract rights. A would have
an action in damages against O for breach of contract, but would not have a
claim for specic performance against B.)
a.
Purchasers
All recording acts protect purchasers (of the fee or any lesser estate).
1)
2)
Example:
An heir who purchases the interests of her co-heirs, without notice of the prior
unrecorded conveyance, is entitled to the same protection as any other purchaser to
the extent of her purchase.
3)
Mortgagees
Mortgagees for value are treated as purchasers, either expressly by the recording
act or by judicial classication.
Example:
O, the owner of a parcel in State X known as Blackacre, deeds the
parcel to A on Monday, but A fails to record the deed. On Tuesday,
O executes a mortgage to Bank. State X has a race-notice recording
statute. Bank is a good faith purchaser for value, and immediately
records its mortgage. Result: Bank has a valid mortgage on the
land, while the title to the land is held by A. (If Bank had not been
a BFP, or had failed to record, A would hold the title free of Banks
mortgage.)
4)
Judgment Creditors
In nearly all states, a plaintiff who obtains a money judgment can obtain, by
statute, a judgment lien on the defendants real estate. A typical statute reads as
follows:
Any judgment properly led shall, for 10 years from ling, be a lien on
the real property then owned or subsequently acquired by any person
against whom the judgment is rendered.
Is a plaintiff who obtains a judgment lien under such a statute protected by the
recording acts from a prior unrecorded conveyance made by the defendant? The
cases are split, but the majority holds that the judgment lienor is not protected.
These courts usually reason either (i) the plaintiff is not a BFP because he did not
pay value for the judgment, or (ii) the judgment attaches only to property owned
by the defendant, and not to property the defendant has previously conveyed away,
even if that conveyance was not recorded.
Example:
On January 1, O grants a mortgage on Blackacre to A. A does not
record the mortgage. On January 15, B, who had previously sued
O on a tort claim, obtains and properly les a judgment against O.
B has no knowledge of the mortgage from O to A. Which lien has
priority, As mortgage or Bs judgment lien? By the majority view,
A has priority despite As failure to record the mortgage. B is not
protected by the recording act.
5)
Example:
6)
a)
Rationale
If the rule were otherwise, a BFP might not be able to convey an interest in
the land. The transferee is not protected for her own sake, but rather for the
sake of the BFP from whom she received title.
b)
(ii) Award the land to the prior claimant, but give the contract purchaser a lien on
the property to the extent of the amount paid [Westpark, Inc. v. Seaton Land
Co., 171 A.2d 736 (Md. 1961)]; or
(iii) Award the land to the contract purchaser, but give the prior claimant a lien on
the property to the extent of the balance still owed [Sparks v. Taylor, 90 S.W.
485 (Tex. 1906)].
Example:
a)
ExceptionShelter Rule
If B in the example above were a BFP, the shelter rule would apply and C
would be fully protected even though C had notice of the O-A conveyance
partway through Cs payments.
b.
Without Notice
Without notice means that the purchaser had no actual, record, or inquiry notice of
the prior conveyance at the time she paid the consideration and received her interest
in the land. While no one has a legal duty to perform a title search, a subsequent
purchaser will be charged with the notice that such a search would provide, whether or
not she actually searches. However, the fact that the purchaser obtains knowledge of the
adverse claim after the conveyance but before she records it is immaterial; she only has
to be without notice at the time of the conveyance.
1)
Actual Notice
The subsequent purchaser must show that she did not actually know of any prior
unrecorded conveyance. Actual notice includes knowledge obtained from any
source (e.g., newspaper, word-of-mouth, etc.).
2)
Wild Deeds
A wild deed is a recorded deed that is not connected to the chain of title. It
does not give constructive notice because the subsequent BFP cannot feasibly
nd it.
Example:
O owns Blackacre, which she contracts to sell to A. The
contract is not recorded, and O remains in possession. A
thereupon conveys Blackacre by deed to B, and B records.
O then conveys Blackacre by deed to C. Did Bs recordation
charge C with constructive notice of Bs claim to equitable title
to Blackacre derived through A? No. C is not charged with
notice because there was no way for him to nd the A-B deed.
Nothing related it to O. It was not in Os chain of title; it was a
wild deed.
Compare:
b)
e)
f)
3)
Inquiry Notice
Inquiry notice means that if the subsequent grantee is bound to make reasonable
inquiry, she will be held to have knowledge of any facts that such inquiry would
have revealed (even though she made none).
a)
b)
c)
d)
4.
Valuable Consideration
A person is protected by the recording statute only from the time that valuable consideration was given. Thus, if a deed was delivered before the consideration was paid,
the purchaser will not prevail over deeds recorded before the consideration was given.
Valuable consideration must be more than merely nominal. A person who claims to be a
BFP must prove that real consideration was paid.
1)
2)
Title Search
Suppose O has contracted to sell Blackacre to A. Prior to closing, A, the buyer, will have
a title search performed to assure herself that O really owns Blackacre and to determine if
there are any encumbrances on Os title. How will As title searcher(s) proceed?
a.
b.
from 1965 backward. The searcher will then look in the grantor index
under Vs name from 1935 to 1965, under Ws name from 1965 to 1990,
under Xs name from 1990 to 2005, and under Os name from 2005 to
the present. In this manner, she will pick up the mortgage to B Bank,
which was recorded in 1995 in the grantor index under Xs name.
2)
2014 C(R)
1980 A(N/R)
2000 A(R)
1980
O conveys to A.
1990
2000
A records.
2014
In searching the title, C will rst look in the grantee index under Bs
name to discover if his seller, B, ever acquired title. He will nd that B
acquired title in 1990 from O. Next, he will look in the grantor index
under Bs name to discover if B made any prior conveyances, and then
will look in the grantee index once again, this time under Os name, to
discover if O ever acquired title. C will nd that O acquired title in 1970.
Then he will look in the grantor index under Os name to discover if O
made a conveyance prior to his conveyance to B. Under the majority
rule, C is required to look under Os name in the grantor index only
through 1990. C will nd no conveyances. In 1990, C will nd the
recorded conveyance to B and he need look no further. On this basis,
C will not nd the recorded conveyance to A because A recorded after
1990. As recording is out of the chain of title, and therefore C is not
charged with notice.
Under a pure notice or race-notice statute, as between A and B, A will
prevail because B, having knowledge of As unrecorded conveyance, is
not a protected party. However, as between A and C, many courts hold
5.
Effect of Recordation
Proper recordation gives all prospective subsequent grantees constructive notice of the
existence and contents of the recorded instruments; i.e., there can be no subsequent BFPs.
Recordation also raises presumptions that the instrument has been validly delivered and that
it is authentic. These presumptions are rebuttable, not conclusive.
a.
b.
Exception
A court may protect a subsequent BFP from an unrecorded implied easement that
is not visible upon inspection of the premises (e.g., an underground sewer).
c.
Recorders Mistakes
An instrument is considered recorded from and after the time it is led at the recorders
ofce, irrespective of whether it is actually listed on the indexes. If the recorders ofce
has made an error in recording, the subsequent purchaser has an action against the
recorders ofce. There is a strong minority view that protects the searcher.
d.
the recorder, by oversight, records a deed that has not been acknowledged or has been
defectively acknowledged?
F.
1)
2)
CompareDefective Acknowledgment
When a recorded instrument has been acknowledged, but the acknowledgment is
defective for some reason not apparent on the face of the instrument, the better
view is that the recordation does impart constructive notice. Rationale: A hidden
defect in the acknowledgment should not be allowed to destroy the constructive
notice that the document otherwise clearly imparts. Purchasers should be entitled
to rely on what appears to be a perfectly recorded document.
Example:
A deed bears what appears to be a valid acknowledgment but is in
fact invalid because the notary was disqualied to act or because
the grantor did not appear personally in front of the notary to
acknowledge her signature, as required by law.
CONVEYANCE BY WILL
A will is a conveyance that is prepared and executed by the property owner during life, but which
does not speak or operate until the date of the owners death. Thus, a will is ambulatory,
meaning that it can be revoked or modied so long as the testator is alive. Some special situations
arise when there is a change in the status of the property or beneciaries between the time the
will is executed and the testators death.
1.
Ademption
If property is specically devised or bequeathed in the testators will, but the testator no
longer owns that property at the time of death, the gift is adeemed. This means that the gift
fails and is not replaced by other property. The reason that the property is no longer owned
by the testator generally does not matter; i.e., it does not matter whether the testator sold the
property or it was accidentally destroyed.
Example:
T owns Blackacre and executes a will devising Blackacre to my daughter
Mary. Prior to his death, T sells Blackacre to A and deposits the proceeds of
the sale in a bank account. Upon Ts death, Mary is not entitled to Blackacre
or to its proceeds. Note that if the will had provided for Ts executor to sell
Blackacre and distribute the proceeds to Mary, she would be entitled to the
proceeds even though the sale occurred before Ts death.
a.
or legacy is one that can be satised only by the delivery of a particular item; it cannot
be satised by money. Thus, a bequest of $10,000, or even of $10,000 to be paid out
of the sale of my IBM stock cannot be adeemed.
b.
c.
d.
Partial Ademption
If the testator specically devises property and then sells or gives away a part of that
property, only that portion is adeemed; the remainder passes to the devisee.
2.
Exoneration
At common law and in some states today, if a testator makes a specic devise of real estate
that is subject to a mortgage or other lien, the devisee is entitled to have the land exonerated by the payment of the lien from the testators residuary estate. Thus, the property will
pass to the devisee free of encumbrances. However, a majority of states have, by statute,
abolished the exoneration doctrine. In these states, the property will pass to the devisee
subject to a preexisting mortgage or other lien unless the will expressly provides for a payoff
of the lien. [See UPC 2-607]
3.
statutes that prevent lapse by permitting the gift to pass to the predeceasing beneciarys
living descendants under certain circumstances. These statutes vary as to the scope of
beneciaries covered.
a.
4.
b.
c.
Abatement
If the estate assets are not sufcient to pay all claims against the estate and satisfy all devises
and bequests, the gifts are abated (i.e., reduced). Absent a contrary will provision, estates
in most jurisdictions abate in the following order: (i) property passing by intestacy; (ii) the
residuary estate; (iii) general legacies, which abate pro rata; and (iv) specic devises and
bequests. Some states provide that within each category personal property abates before real
property.
1.
Mortgage
The debtor/notemaker is usually the mortgagor; he gives the mortgage (along with the note)
to the lender, who is the mortgagee. But note that the debtor and mortgagor can be different
people (e.g., a mother agrees to place a mortgage on her house to secure a loan to her
daughter). Most states require that a lender realize on the real estate to satisfy the debt only
by having a judicial (court-ordered) foreclosure sale conducted by the sheriff.
2.
Deed of Trust
The debtor/notemaker is the trustor. The trustor gives the deed of trust to a third-party
trustee, who is usually closely connected with the lender (e.g., the lenders lawyer, afliated
corporation, or ofcer). In the event of default, the lender (termed the beneciary) instructs
the trustee to proceed with foreclosing the deed of trust by sale. Many states allow the sale
to be either judicial (as with a mortgage) or nonjudicial, under a power of sale clause that
authorizes the trustee to advertise, give appropriate notices, and conduct the sale personally.
3.
4.
5.
Sale-Leaseback
A landowner needing to raise money may sell her land to another for cash and may then
lease the land back for a long period of time. As in the case of the absolute deed, the grantor/
lessee may attack such a transaction later as a disguised mortgage. Factors that will lead the
court to such a result are: (i) the fact that the regular rent payments on the lease are virtually
identical to payments that would be due on a mortgage loan; (ii) the existence of an option
to repurchase by the grantor/lessee; and (iii) the fact that the repurchase option could be
exercised for much less than the probable value of the property at that time, so that the repurchase would be very likely to occur.
6.
(E.2.b.3), infra), a seller may nance the buyers purchase of the land by an equitable
vendors lien. The lien does not result from an agreement, but rather arises by implication
of law when the seller transfers title to the buyer and the purchase price or a portion of the
purchase price remains unpaid.
B.
Transfer by Mortgagee
a.
b.
(4) The transferee must take the note in good faith and must pay value for
it. (Value implies an amount that is more than nominal, although it
need not be as great as the notes fair market value.) The transferee must
not have any notice that the note is overdue or has been dishonored, or
that the maker has any defense to the duty to pay it.
b)
2)
2.
Assumption
Often the grantee signs an assumption agreement, promising to pay the mortgage loan.
If she does so, she becomes primarily liable to the lender (usually considered a thirdparty beneciary), while the original mortgagor becomes secondarily liable as a surety.
Note, however, that the mortgagee may opt to sue either the grantee or the original
mortgagor on the debt. If the mortgagee and grantee modify the obligation, the original
mortgagor is completely discharged of liability.
b.
Nonassuming Grantee
A grantee who does not sign an assumption agreement does not become personally
liable on the loan. Instead, the original mortgagor remains primarily and personally
liable. However, if the grantee does not pay, the mortgage may be foreclosed, thus
wiping out the grantees investment in the land.
c.
C.
Due-on-Sale Clauses
Most modern mortgages contain due-on-sale clauses, which purport to allow the
lender to demand full payment of the loan if the mortgagor transfers any interest in the
property without the lenders consent. Such clauses are designed to both: (i) protect
the lender from sale by the mortgagor to a poor credit risk or to a person likely to
commit waste; and (ii) allow the lender to raise the interest rate or charge an assumption fee when the property is sold. Federal law preempts state law and makes due-onsale clauses enforceable for all types of institutional mortgage lenders on all types of
real estate. The preemption does not apply to isolated mortgage loans made by private
parties.
2.
3.
Payment
Generally, full payment of the note discharges the mortgage lien. The agreement in the
note, however, will normally govern whether the mortgagor may prepay the obligation.
If the note or mortgage does not provide for prepayment, the mortgagor has no right to
prepayment. The mortgage may also provide for a prepayment fee or a total prohibition
of prepayment for part or all of the mortgage term. Note: The Dodd-Frank Act prohibits
prepayment penalties on certain loans (e.g., adjustable rate, interest-only). [15 U.S.C.
1639c(c)]
b.
Merger
A mortgage lien vests the mortgagee with either an equitable interest (lien theory) or
a legal interest (title theory), while the mortgagor retains the other interest (see D.1.,
infra). If the mortgagee subsequently acquires the mortgagors interest, the mortgage
is said to merge with the title, and the mortgagors personal liability on the underlying
debt is discharged up to the value of the land.
c.
D.
Theories of Title
The mortgagee may have a right to take possession before foreclosure, depending on the
theory the state follows.
a.
b.
c.
2.
3.
risks include a very strict duty to account for all rents received, a duty to manage the
property in a careful and prudent manner, and potential liability in tort to anyone injured on
the property.
4.
E.
Receiverships
Instead of becoming a mortgagee in possession, most mortgagees attempt to intercept the
rents before foreclosure by getting a receiver appointed by the court to manage the property.
Courts will generally appoint receivers for rental property upon a showing of some combination of three factors: (i) that waste is occurring, (ii) that the value of the property is inadequate to secure the debt, and (iii) that the mortgagor is insolvent.
FORECLOSURE
Foreclosure is a process by which the mortgagors interest in the property is terminated. The
property is generally sold to satisfy the debt in whole or in part (foreclosure by sale). Almost all
states require foreclosure by sale. All states allow judicial sale, while about one-half also allow
nonjudicial sale under a power of sale. The nonjudicial sale is often permitted with deeds of trust
but not with mortgages. Foreclosure sales are conducted by auction, with the highest bidder taking
the property. The lender may bid at the sale, and in many cases the lender is the sole bidder.
(Note: For convenience, the discussion below speaks of mortgagors and mortgagees, but the same
principles apply to deed-of-trust trustors and beneciaries.)
1.
2.
Redemption
a.
Redemption in Equity
At any time prior to the foreclosure sale, the mortgagor has the right to redeem the
land or free it of the mortgage by paying off the amount due, together with any accrued
interest. If the mortgagor has defaulted on a mortgage or note that contains an acceleration clause permitting the mortgagee to declare the full balance due in the event of
default, the full balance must be paid in order to redeem. A mortgagors right to redeem
her own mortgage cannot be waived in the mortgage itself; this is known as clogging
the equity of redemption and is prohibited. However, the right can be waived later for
consideration.
b.
Statutory Redemption
About half the states give the mortgagor (and sometimes junior lienors) a statutory right
to redeem for some xed period after the foreclosure sale has occurred; this period is
usually six months or one year. The amount to be paid is usually the foreclosure sale
price, rather than the amount of the original debt. Be careful to distinguish equitable
redemption, which is universally recognized (but only up to the date of the sale), from
statutory redemption, which is only recognized by about half the states and applies only
after foreclosure has occurred.
Priorities
Generally, the priority of a mortgage is determined by the time it was placed on the property.
When a mortgage is foreclosed, the buyer at the sale will take title as it existed when the
mortgage was placed on the property. Thus, foreclosure will terminate interests junior to the
mortgage being foreclosed but will not affect senior interests.
a.
b.
2)
Modication of Priority
As noted above, priorities among mortgages on the same real estate are normally
determined simply by chronology: the earliest mortgage placed on the property is rst
in priority, the next mortgage is second, and so on. However, the chronological priority
may be changed in the following ways:
1)
Failure to Record
If the rst mortgagee fails to record, and the second mortgagee records, gives
value, and takes without notice of the rst, the second mortgagee will have priority
over the rst by virtue of the normal operation of the recording acts.
2)
Subordination Agreement
A rst mortgagee may enter into an agreement with a junior mortgagee, subordinating its priority to the junior mortgagee. Such agreements are generally enforced.
However, a broad promise to subordinate to any mortgage (or a vaguely described
mortgage) to be placed on the property in the future may be considered too inequitable to enforce.
3)
(ii) A third-party lender who is lending the funds to allow the buyer to purchase
the property.
A PMM, whether recorded or not, has priority over mortgages, liens, and other
claims against the mortgagor that arise prior to the mortgagors acquisition of title.
However, PMM priority is subject to being defeated by subsequent mortgages or
liens by operation of the recording acts or may be altered through a subordination
b)
4)
5)
6)
Subrogation
A mortgage taken out for the purpose of renancing a preexisting senior mortgage
takes the priority position of the senior mortgage.
Example:
3.
Proceeds of Sale
The proceeds of the foreclosure sale are used rst to pay expenses of the sale, attorneys
fees, and court costs; then to pay the principal and accrued interest on the loan that was
foreclosed; next to pay off any junior liens or other junior interests in the order of their
priority; and nally, any remaining proceeds are distributed to the mortgagor. In many
cases, there is no surplus remaining after the principal debt is paid off.
4.
Deciency Judgments
If the proceeds of the sale are insufcient to satisfy the mortgage debt, the mortgagee can
bring a personal action against the mortgagor/debtor for the deciency. However, a number
of states limit the deciency that can be recovered to the difference between the debt and the
propertys fair market value when the fair market value is higher than the foreclosure price.
Other states prohibit deciency judgments entirely on PMMs and on deeds of trust that are
foreclosed by power of sale.
Examples:
1) Assume that land has a fair market value of $50,000 and is subject to three
mortgages executed by its owner, whose name is MR. The mortgages have
priorities and secure outstanding debts in the amounts shown below, which
are owed to three different creditors, ME1, ME2, and ME3:
Mortgage 1
MR
$30,000
ME1
Mortgage 2
MR
$15,000
ME2
Mortgage 3
MR
$10,000
ME3
Assume that Mortgage 1 is foreclosed, and the bid at the sale is $50,000 (the
fair value of the land). How will the funds be distributed?
In an actual case, the funds would rst be used to pay any attorneys fees and
expenses of the foreclosure, and then to any accrued interest on Mortgage 1.
However, we will assume that these items are zero.
The $50,000 in funds from the sale will then be used to pay off the mortgages
in the order of their priority. Thus, $30,000 is applied to fully pay off
Equity of Redemption
Several states allow the contract purchaser who is in default to pay off the accelerated full
balance of the contract and to keep the land. In other words, they grant the purchaser a grace
period. This is roughly analogous to the equity of redemption in mortgage law. A few states
have statutory schedules of grace periods, which often provide for a longer time if a greater
percentage of the total price has been paid.
2.
Restitution
A number of decisions allow actions by the vendor for forfeiture of the land but require her
to refund to the purchaser any amount by which his payments exceed the vendors damages.
The court may measure these damages by the propertys fair rental value while the purchaser
was in possession or by any drop in market value since the contract was executed.
3.
Treat as a Mortgage
A few states, by statute or case law, now treat installment contracts like mortgages, at least
for purposes of the vendors remedies. In effect, the vendor must foreclose the contract by
judicial sale in order to realize on the real estate, and she cannot simply reclaim the land.
4.
Waiver
Many cases hold that where a vendor has established a pattern of accepting late payments
from the purchaser, she cannot suddenly insist on strict on-time payment and declare a forfeiture if such payment is not forthcoming. Such a pattern is said to constitute a waiver of strict
performance. To reinstate strict performance, the vendor must send the purchaser a notice of
her intention to do so and must allow a reasonable time for the purchaser to make up any late
payments and to get back on stream.
5.
Election of Remedies
It is commonly held that the vendor who elects to pursue a forfeiture cannot also bring an
action for damages or for specic performance. The vendor must choose only one remedy
and forgo all others.
IN GENERAL
The owner of real property has the exclusive right to use and possess the surface, the airspace, and
the soil of the property. This right is subject to restrictions in the chain of title (e.g., easements and
covenants), to the law of nuisance, and to any valid laws or regulations that restrict the use of the
land (e.g., zoning ordinances).
B.
b.
2.
C.
b.
WATER RIGHTS
Different rules apply depending on whether the water rights claimed involve (i) water in watercourses (e.g., streams, rivers, and lakes, including underground watercourses); (ii) ground or
percolating water (e.g., water normally pumped or drawn from wells); or (iii) surface water (e.g.,
rainfall, seepage). Exam questions normally concern who has priority to use the water from
watercourses and from the ground, and to what extent a landowner may obstruct or divert the ow
of surface water.
1.
Watercourses
There are two major systems for allocation of water in watercourses: (i) the riparian
doctrine (generally applied in the eastern states where water is or was relatively abundant),
and (ii) the prior appropriation doctrine (generally used in the 17 western states where
water is relatively scarce).
a.
Riparian Doctrine
Under the riparian doctrine, water does not belong to the public generally or to the
state (with certain exceptions) but rather to the riparian proprietors who own land
bordering on the watercourse. All of these landowners have riparian rights and none
can use the water so as to deprive the others of these rights.
1)
owner purchases a parcel which is contiguous to the riparian parcel, riparian rights
attach to the newly acquired parcel. The minority rule limits riparian rights to
the smallest tract of land ever owned abutting the water. Under this view, if a back
portion of a riparian tract is sold, it becomes nonriparian and can never regain
riparian rights.
2)
a)
Riparian Owner
Riparian owners include the fee owner of the abutting land and, to the extent
of their title, lessees and easement owners of such land.
b)
b)
c)
c.
2.
Groundwater
If water comes from an underground watercourse (e.g., a dened stream or river), the
riparian or prior appropriation doctrines apply. However, the presumption is that underground water is percolating (i.e., the water moves through the ground diffusely and is usually
withdrawn by wells from the underground water table). There are four different rules for
determining rights in underground water.
a.
b.
the other hand, virtually all benecial uses of water on the land are considered reasonable and are allowed.
3.
c.
d.
e.
Restatement Approach
A few states follow the Restatement approach, which is based on principles of nuisance
law. This approach allows the surface owner to pump groundwater for a benecial
purpose unless the withdrawal: (i) unreasonably causes harm to neighboring landowners
through lowering of the water table; (ii) exceeds the pumpers reasonable share of the
annual supply or total store of groundwater; or (iii) directly and substantially affects
surface waters and unreasonably causes harm to a surface water user. [Restatement
(Second) of Torts 858]
Surface Waters
Diffused surface waters are those that have no channel but pass over the surface of the land.
The source may be rainfall, melting snow, seepage, etc. A landowner can use surface waters
within her boundaries for any purpose she desires. Problems concern the right of a lower
owner to restrict a ow that would naturally cross his land (e.g., by dikes) and the right of
an upper owner to alter or divert a natural ow onto other lands (e.g., by drains, channels, or
sloughs). The acting landowners liability to other landowners depends upon which doctrine
the state follows.
a.
b.
c.
watercourse cases, requires balancing the utility of the use against the gravity of the
harm. Judicial mitigation of both the natural ow and common enemy doctrines often
results in an approximation of the reasonable use theory.
d.
D.
RIGHTS IN AIRSPACE
The right to the airspace above a parcel is not exclusive, but the owner is entitled to freedom
from excessive noise and transit by aircraft. If ights are so low as to be unreasonably disturbing,
they constitute a trespass or (if the airport is government-owned) a taking by inverse condemnation.
E.
Trespass
If the land is invaded by a tangible physical object that interferes with the right of exclusive
possession, there is a trespass.
2.
Private Nuisance
If the land is invaded by intangibles (e.g., odors or noises) that substantially and unreasonably interfere with a private individuals use or enjoyment of her property, the possessor may
bring an action for private nuisance.
a.
ComparePublic Nuisance
Public nuisance is an invasion by intangibles that unreasonably interfere with the health,
safety, or property rights of the publici.e., a broad segment of the community, rather
than one or a few individuals.
3.
Continuing Trespass
If the land is repeatedly invaded by a trespasser (e.g., the invader repeatedly swings a crane
over the property), the possessor may sue for either trespass or nuisance.
4.
Law or Equity
If the possessor wants to force the invader to stop the invasion of the property, the remedy is
an injunction in equity. If the possessor wants damages, the remedy is an action at law.
a.
Ejectment
The remedy at common law to remove a trespasser from the property is ejectment.
b.
Unlawful Detainer
In the landlord-tenant situation, the landlord may force the tenant to vacate the premises
by the statutory remedy of unlawful detainer. (In some states, the term used to describe
this action is forcible detainer or summary ejectment.) The action may be joined with a
demand for money damages in rent due.
B.
C.
COOPERATIVES
In the most common form of housing cooperative, title to the land and buildings is held by a
corporation that leases the individual apartments to its shareholders. Thus, the residents in a
cooperative are both tenants of the cooperative (by virtue of their occupancy leases) and owners
of the cooperative (by virtue of their stock interests). Stock interests in the cooperative are not
transferable apart from the occupancy lease to which they are attached.
1.
2.
Mortgages
Permanent nancing is provided through a blanket mortgage on the entire property owned
by the cooperative corporation (land and buildings). This mortgage has priority over the
occupancy leases. Failure to meet the payments on the blanket mortgage may result in the
termination of the leases through foreclosure of the mortgage. Thus, each cooperative tenant
is vitally concerned that the other tenants pay their shares of the blanket mortgage.
3.
Maintenance Expenses
Ordinarily, cooperative tenants are not personally liable on the note or bond of the blanket
mortgage. However, under their occupancy leases, each tenant is liable for her proportionate
share of all of the expenses of the cooperative (including payments on the mortgage as well
as other operating expenses).
CONDOMINIUMS
In a condominium, each owner owns the interior of her individual unit plus an undivided interest
in the exterior and common elements.
1.
2.
Mortgages
Each unit owner nances the purchase of her unit by a separate mortgage on her unit.
Consequently, unit owners need not be as concerned about defaults by others as they must be
in a cooperative.
3.
Maintenance Expenses
Each unit owner is personally liable on her own mortgage and each pays her own taxes
(unlike the cooperative situation, but like any other homeowner). In addition, each unit
owner is liable to contribute her proportionate share to the common expenses of maintaining
the common elements, including insurance thereon.
ZONING
The state may enact statutes to reasonably control the use of land for the protection of the health,
safety, morals, and welfare of its citizens. Zoning is the division of a jurisdiction into districts in
which certain uses and developments are permitted or prohibited. The zoning power is based on
the states police power and is limited by the Due Process Clause of the Fourteenth Amendment.
Other limitations are imposed by the Equal Protection Clause of the Fourteenth Amendment
and the no taking without just compensation clause of the Fifth Amendment. (See Multistate
Constitutional Law outline.) Cities and counties can exercise zoning power only if authorized to
do so by state enabling acts. Ordinances that do not conform to such acts are ultra vires (beyond
the authority of the local body) and void.
1.
Nonconforming Use
A use that exists at the time of passage of a zoning act and that does not conform to the
statute cannot be eliminated at once. Some statutes provide for amortizationi.e., the
gradual elimination of such nonconforming uses (e.g., the use must end in 10 years).
2.
3.
Variance
A variance from the literal restrictions of a zoning ordinance may be granted by administrative action. The property owner must show that the ordinance imposes a unique hardship on
him and that the variance will not be contrary to the public welfare.
4.
b.
Generally, the regulation will be found to be a taking only if it unjustly reduces the
economic value of the property (e.g., greatly reduces the property value and only
slightly promotes the public welfare). [Pennsylvania Coal Co. v. Mahon, 260 U.S. 393
(1922); Keystone Bituminous Coal Association v. DeBenedictis, 480 U.S. 470 (1987)]
c.
d.
Unconstitutional Exactions
Local governments often demand, in exchange for zoning approval for a new project,
that the landowner give up some land for a public purpose, such as street widening.
However, such demands are unconstitutional under the Fifth and Fourteenth
Amendments unless they meet the tests set out below. [Nollan v. California Coastal
Commission, 483 U.S. 825 (1987); Dolan v. City of Tigard, 512 U.S. 374 (1994)]
1)
Essential Nexus
The local governments demand must be rationally connected to some additional
burden that the proposed project will place on public facilities or rights. Thus, a
city could demand land for a street widening upon a showing that the proposed
project would otherwise increase trafc congestion and pollution along the street in
question.
2)
Rough Proportionality
Even if the essential nexus test above is met, the local government must not
demand too much. The required dedication must be reasonably related, both in
nature (the essential nexus) and extent (the amount of the exaction), to the impact
of the proposed development.
3)
Burden of Proof
The local government has the burden of showing that both the essential nexus and
rough proportionality tests are met.
Remedy
If a property owner challenges a regulation and the court determines that there was
a taking, the government will be required to either: (i) compensate the owner for the
taking, or (ii) terminate the regulation and pay the owner for any damages that occurred
while the regulation was in effect. [First English Evangelical Lutheran Church v.
County of Los Angeles, 482 U.S. 304 (1987)]
CMR
COMPARISON
CHART
CONCURRENT OWNERSHIP
Definition
Creation
Termination
Joint Tenancy
To A and B as joint
tenants with the right
of survivorship.
(Without survivorship
language, it may be
construed as a
tenancy in common.)
Joint tenants must
take:
(i) identical interests;
(ii) from the same
instrument;
(iii) at the same time;
(iv) with an equal
right to possess
(the four unities).
The right of
survivorship may be
severed, and the
estate converted to a
tenancy in common,
by: a conveyance by
one joint tenant,
agreement of joint
tenants, murder of
one co-tenant by
another, or
simultaneous deaths
of co-tenants.
A joint tenancy can
be terminated by
partition (voluntary or
involuntary).
Tenancy by the
Entirety
To H and W. Some
states presume a
tenancy by the
entirety in any joint
conveyance to
husband and wife
where the four unities
(above) are present.
The right of
survivorship may be
severed by death,
divorce, mutual
agreement, or
execution by a joint
creditor. Tenancy by
the entirety cannot be
terminated by
involuntary partition.
Tenancy in Common
To A and B or,
sometimes, To A and
B as joint tenants.
Only unity required is
possession.
May be terminated by
partition.
Type of Tenancy
CMR
COMPARISON
CHART
Assignment by
Tenant
Sublease by
Tenant
Consent
Landlords consent
may be required by
lease.
Landlords consent
may be required by
lease.
Privity of Estate
Assignee and
landlord are in privity
of estate.
Sublessee and
landlord are not in
privity of estate.
Original tenant
remains in privity of
estate with landlord.
Privity of Contract
Assignee and
landlord are not in
privity of contract.
Original tenant and
landlord remain in
privity of contract.
Sublessee and
landlord are not in
privity of contract.
Original tenant and
landlord remain in
privity of contract.
Liability for
Covenants in Lease
Assignee liable to
tenant on all
covenants that run
with the land.
Assignee liable to
landlord on all
covenants that run
with the land.
Sublessee is not
personally liable on
any covenants in the
original lease and
cannot enforce the
landlords covenants.
Original landlord
remains liable on all
covenants in the
lease.
Original tenant
remains liable for rent
and all other
covenants in the
lease.
Original tenant
remains liable for rent
and all other
covenants in the
lease and can
enforce the landlords
covenants.
CMR
COMPARISON
CHART
Type of Leasehold
LEASEHOLD ESTATES
Definition
Creation
Termination
To T for 10 years.
Terminates at the
end of the stated
period without either
party giving notice.
Periodic Tenancy
To T from month to
month.
Terminates by
notice from one
party at least equal
to the length of the
time period (e.g.,
one full month for a
month-to-month
tenancy). Exception:
Only six months
notice is required to
terminate a year-toyear tenancy.
or
To T, with rent
payable on the first
day of every month.
or
L elects to bind
hold-over T for an
additional term.
Tenancy at Will
Tenancy of no
stated duration that
lasts as long as
both parties desire.
Usually terminates
after one party
displays an intention
that the tenancy
should come to an
end. May also end
by operation of law
(e.g., death of a
party, attempt to
transfer interest).
To T for as many
years as T desires.
(Only T may
terminate.)
Tenancy at
Sufferance
Tenant wrongfully
holds over after
termination of the
tenancy.
Ts lease expires,
but T continues to
occupy the
premises.
Terminates when
landlord evicts
tenant or elects to
hold tenant to
another term.
CMR
COMPARISON
CHART
NONPOSSESSORY INTERESTS
Easement
License
Profit
Real Covenant/
Equitable
Servitude
Definition
A grant of an
interest in land
that allows
someone to use
anothers land
0ERMISSION TO GO 2IGHT TO TAKE
onto anothers
resources from
land
anothers land
0ROMISE TO DO OR
not to do
something on the
land
Example
Owner of parcel
A grants owner of
parcel B the right
to drive across
parcel A
O allows the
electrician to
come onto Os
land to fix an
outlet
O allows A to
come onto Os
land to cut and
REMOVE TIMBER
O conveys an
adjoining parcel to
A. A promises not
TO BUILD A
swimming pool on
the property
Writing
Generally
required.
Exceptions:
s ,ESS THAN ONE
year
s )MPLICATION
s .ECESSITY
s 0RESCRIPTION
.OT REQUIRED
Note: An invalid
oral easement is
a license
2EQUIRED
2EQUIRED
Exception:
%QUITABLE
SERVITUDE MAY BE
implied from
common scheme
of development of
residential
SUBDIVISION
Termination
s 3TATED
conditions
s 2ELEASE
s -ERGER
s !BANDONMENT
s %STOPPEL
s 0RESCRIPTION
s %ND OF
necessity
5SUALLY REVOCABLE 3AME AS
AT WILL -AY BE
easement
IRREVOCABLE IF
coupled with an
interest or if
licensor estopped
BY LICENSEES
expenditures
s
s
s
s
2ELEASE
-ERGER
#ONDEMNATION
!LSO EQUITABLE
defenses may
apply to
enforcement of
servitude
Question 1
Question 2
Question 3
Question 4
(D) A license.
Question 5
A developer owned several acres zoned for
mixed use development. The developer prepared
a subdivision of his various parcels, led a
subdivision map showing commercial lots,
obtained all the necessary approvals, and began
selling the lots. Each of the deeds conveying lots
sold by the developer contained the following:
It is hereby covenanted by the seller
that the property conveyed shall be
used for commercial or residential
purposes only, that no industrial,
warehouse, or other manufacturing
structures shall be erected or
maintained thereon, and that this
covenant shall bind the buyer, his heirs
and assigns, and their successors.
Two years later, after all but two of the lots
had been developed as small businesses, the
developer sold his remaining two lots to a real
estate speculation rm. The deed to the rm did
not contain any language restricting the use of
the property. The rm then sold the property to
a giant supermarket chain, which intended to
construct a warehouse and distribution center
thereon. A shopkeeper who had purchased a lot
from the developer located next to the proposed
warehouse brings suit against the supermarket
chain seeking to enjoin construction of the
warehouse. Her attorney argues that the lots
sold by the developer to the rm and then to
the supermarket chain are bound by the same
restrictions on use that are contained in the deed
by which the shopkeeper took her property.
Is the shopkeeper likely to win?
(A) Yes, because the developer established a
common development scheme for his entire
subdivision and the subdivision appeared to
conform to the scheme.
(B) No, because the rm and the supermarket
were not aware of the restrictions when they
purchased the property.
(C) No, because the restrictions in the shopkeepers deed bind only the purchaser of the land.
Question 7
Question 8
If the charity brings suit to impose a constructive trust on the land, will it prevail?
(A) Yes, because past and future care is not
adequate consideration.
(B) Yes, because the deed to the nephew was
void.
(C) No, because the land has been adeemed.
(D) No, because the buyer is a bona de
purchaser.
Question 9
Question 10
Question 11
Question 12
remote event (the charity-to-charity exception). Here, the rst gift is to an individual and the gift
over (a shifting executory interest) is to a charity; because the triggering event that will transfer
the property (no dogs kept on the property) may occur more than 21 years after lives in being at
the creation of the interest, the interest is stricken under the Rule Against Perpetuities.
Answer to Question 3
(B) The tenant will prevail because a full months notice is required. In this case, there was no written
lease outlining the terms of the tenancy, but only an oral agreement regarding the payment of
monthly rent. That led to the creation of a periodichere, a monthlytenancy. The termination date of a periodic tenancy is uncertain, until notice of termination is given. For a tenancy
that is less than a year in duration, such as this, notice must be given a full period in advance of
the termination. Although the landlord gave notice, because the lease was monthly, the landlord
was required to provide a full months notice of termination. (A) is incorrect because six months
notice would be required for a tenancy from year to year, but this tenancy was a month-to-month
tenancy. (C) is incorrect. A tenancy at will arises from an agreement between the parties that
either party may terminate the tenancy at any time. However, in the absence of a specic agreement to create a tenancy at will, regular rent payments will lead to the tenancy being treated as a
periodic tenancy. (D) is incorrect. Tenancies at sufferance arise from a tenant wrongfully holding
over after the termination of a tenancy. It is true that no notice is necessary in order to terminate
such a tenancy. Nothing in the facts presented, however, allows the conclusion that the tenant was
wrongfully holding over when the landlord gave notice.
Answer to Question 4
(A) If the court rules for the landowner, it will be because the shermans right to hunt and sh on the
western parcel is a prot appurtenant. A prot is a nonpossessory interest in land that entitles the
holder of the prot to enter on the servient tenement and take the soil or a substance of the soil
(e.g., minerals, timber, oil, or game). Like an easement, a prot may be appurtenant or in gross.
If the prot exists to serve a dominant estate, the prot is appurtenant and can only be transferred
along with the dominant estate. Conversely, if the prot does not exist to serve a dominant estate,
it is a prot in gross and may be transferred separate and apart from the dominant estate. Here,
the sherman has a prot with respect to the game on the western parcel. Because the prot is
for the benet of the campground on the eastern parcel, it is appurtenant rather than in gross
because it serves the dominant estate (the eastern parcel). Thus, the shermans assignment of
the prot to the hunter is void. (B) is incorrect because it supports the hunters rather than the
landowners claim. As discussed above, if the prot were in gross, it could have been transferred
to the hunter. (C) is incorrect because the sherman has a prot rather than an easement. Like a
prot, an easement is a nonpossessory interest in land. However, the holder of an easement only
has the right to use the servient land and not to remove the soil or products of the soil therefrom
(including game). (D) is incorrect because, as explained above, the shermans interest is alienable, whereas a license is personal to the licensee and therefore not alienable. The problem here
is that the sherman did not transfer the dominant parcel with the prot, and thus the attempted
transfer of the prot appurtenant alone is void.
Answer to Question 5
(A) The shopkeeper will win because the developer established a common development scheme for the
entire subdivision and the subdivision appeared to conform to the scheme. An injunction against
contracts for the sale of land contain, unless the contract expressly provides otherwise, an implied
covenant by the seller that she will deliver to the buyer a marketable title at the date of closing.
Marketability refers to freedom from the possibility of litigation concerning the title; title is
marketable if a reasonably prudent buyer, ready and able to purchase, will accept it in the exercise
of ordinary prudence. At times, sellers will rely on adverse possession to show that defects in title
have been cleared. However, courts generally will not permit such reliance when proof of adverse
possession rests only on oral evidence that will not be available to the buyer in the future. Here,
title to the property described in the contract is unmarketable because the ve-foot strip was a
private right-of-way and not part of the owners record title. The owners adverse possession of
the strip will not be sufcient by itself to establish marketable title; there is no longer any physical
evidence of the owners possession. Thus, at the least the owner must offer the buyer additional
proof that the buyer can use to defend any lawsuit challenging title. (B) is wrong because the
owner removed the fence after she had acquired title by adverse possession. While that makes
it more difcult for her to establish marketable title in selling the property, it does not affect
the ownership rights she gained by adverse possession. (C) is a misstatement of law. Although
government property, including public rights-of-way, is generally exempt from the operation of
statutes of limitations, the facts of this question specically state that this is a private right-of-way.
(D) is wrong because, as discussed above, the fact that the owner has title to the strip does not
mean that she has marketable title.
Answer to Question 8
(D) The charity will not prevail in a suit to impose a constructive trust on the land because the buyer
is a bona de purchaser. A deed is voidable if it is executed by a minor or incapacitated person
or is obtained through fraud in the inducement, duress, undue inuence, mistake, or breach of
duciary duty. Undue inuence exists where (i) inuence is exerted on the grantor, (ii) the effect
of the inuence is to overpower the mind and free will of the grantor, and (iii) the product of the
inuence is a deed that would not have been executed but for the inuence. A voidable deed will
be set aside only if the property has not passed to a bona de purchaser (i.e., a purchaser who pays
valuable consideration and takes without notice). Here, although the nephews acts may not have
risen to the level of duress, they likely constitute undue inuence. His threat of cessation of care
caused the aunt to convey the land to him, when her desire was to devise the land to a charity.
Thus, the deed is voidable. However, because the buyer paid market value for the land without
notice of the circumstances surrounding the aunt-nephew deed, the deed will not be set aside.
(A) is incorrect because consideration is not required in order to make a deed valid. (B) is incorrect because the deed was voidable rather than void. Deeds considered void include those that are
forged, were never delivered, or were obtained by fraud in the factum; none of those apply here.
(C) is incorrect because, although specically devised property that is no longer in the testators
estate at her death is adeemed, the charity would have prevailed in its suit to impose a constructive trust on the land if it had not been conveyed to a bona de purchaser. A constructive trust
is an equitable remedy imposed by a court to prevent an unjust enrichment of one person at the
expense of another as the result of wrongful conduct, such as fraud, undue inuence, or breach of
duciary duty. Here, the nephew would have been unjustly enriched if he were allowed to retain
title to land he acquired through undue inuence. However, this remedy was cut off by the
nephews conveyance to the buyer, a bona de purchaser.
Answer to Question 9
(C) The nephews express rejection of the deed was sufcient to rebut any presumption of acceptance. As a general rule, delivery of the deed is the nal operative act to complete a conveyance
of title to the grantee, because courts will infer the grantees acceptance if the conveyance is
benecial to him. However, all courts will consider evidence that is contrary to the presumption
or inference. Hence, the nephews express rejection of the gift is sufcient to establish that no
conveyance of the property took place. (A) is an incorrect statement of law. If the grantor intends
the recording of the deed to be the nal act in vesting title in the grantee, then such recording
creates a presumption of delivery even where the grantee did not know of the recordation. (B) is
wrong because there is no such thing as a constructive reconveyance. Had the nephew accepted
the gift (completing the conveyance) and later changed his mind, the nephew would have had to
execute a new deed to convey the property back to the uncle. (D) is wrong because knowledge
or permission of the grantee has no effect on the validity of the recordation; rather, it determines
whether there has been an effective acceptance.
Answer to Question 10
(D) This answer states the traditional rule where the amount of land in a land sale contract is less than
as agreed. When a buyer has a remedy of specic performance in a land sale contract, a court of
equity will order a seller to convey the title if the buyer tenders the purchase price. If the seller
cannot provide marketable title under the terms of the contract, but the buyer wishes to proceed
with the transaction, the buyer can usually get specic performance with an abatement of the
purchase price in an amount reecting the title defect. A defect as to the quantity of land conveyed
is usually corrected by a pro rata abatement of the price. (D) states the factors that a court of
equity will look for when deciding whether to grant specic performance with abatement. (A) is
incorrect because the parties contract did not merely refer to the farm as a named parcel of land;
it recited that it contained 250 acres. Based on this recital, a court could readily conclude that the
difference of two acres is a material change in the terms of the contract and that the sellers tender
of 248 acres was not substantial performance. (B) is incorrect because viewing the property did
not put the buyer on notice as to the discrepancy; the buyer is not required to visually calculate the
amount of acreage a parcel of land contains. (C) is not as good an answer as (D) even though it
is probably a true statement. Not only must the defect as to quantity be material, so that the buyer
is not receiving what he bargained for, but the abatement amount must be appropriate and not an
excessive reduction of the purchase price, as choice (D) states.
Answer to Question 11
(B) The credit unions mortgage has priority because the bank agreed to subordinate its interest.
Priority among mortgages on the same real estate is normally determined by chronology.
However, this may be modied by the operation of a recording act, special rules governing
purchase money mortgages, or the execution of a subordination agreement. A rst mortgagee
may enter into an agreement with a junior mortgagee to subordinate its priority to the junior
mortgagee, and such agreements generally are enforced if they are specic. (C) is incorrect
because a subordination agreement will determine priority despite the existence of a recording
act. This jurisdiction has a race-notice statute, under which a subsequent bona de purchaser
(i.e., a person, including a mortgagee, who gives valuable consideration and has no notice of the
prior instrument) prevails over a prior grantee only if he records before the prior grantee records.
Here, only the bank recorded its interest. Thus, the bank would prevail under the recording act if
there were no subordination agreement. (A) would state the priority rule for a notice jurisdiction,
in which a subsequent bona de purchaser prevails over a prior grantee regardless of whether the
subsequent purchaser records. However, a valid subordination agreement would govern in a notice
jurisdiction as well. Furthermore, the credit union had notice of the banks mortgage here because
the bank entered into a subordination agreement with the credit union. (D) is incorrect. A purchase
money mortgage, given when the mortgagor buys the property, has priority over non-purchase
money mortgages that arise prior to the mortgagors acquisition of title. However, priority can be
defeated by subsequent mortgages or liens through operation of the recording acts, and it can be
relinquished through a subordination agreement.
Answer to Question 12
(C) The sellers mortgage remains on the property and the banks mortgage is extinguished, and
the buyer is personally liable to the bank for the deciency. As a general rule, the priority of a
mortgage is determined by the time it was placed on the property. When a mortgage is foreclosed,
the purchaser at the sale will take title as it existed when the mortgage was placed on the property.
Thus, foreclosure will terminate interests junior to the mortgage being foreclosed but will not
affect senior interests. The proceeds of the foreclosure sale are used rst (after expenses and
fees) to pay the principal and accrued interest on the loan that was foreclosed, and then to pay
off any junior interests in the order of priority. Where the proceeds of the sale are insufcient to
satisfy a mortgage debt, the mortgagee can bring a personal action against the mortgagor/debtor
for the deciency. Here, foreclosure by the credit union leaves the sellers senior purchase money
mortgage interest intact on the property; the purchaser at the foreclosure sale takes the property
subject to that mortgage. On the other hand, the banks mortgage interest, because it was junior
to the credit unions interest, was extinguished by the credit unions foreclosure action. After the
credit unions loan is paid off, the $20,000 that remains is used to reduce the amount of the debt
owed to the bank. The bank can recover the balance against the buyer personally in a deciency
action. (A) is wrong because the sellers mortgage and the banks mortgage are treated differently because of their priority in relation to the credit unions mortgage. (B) states the opposite
of the actual resultthe sellers mortgage (the senior interest) remains on the property and the
banks mortgage (the junior interest) is extinguished. (D) is wrong because, as discussed above,
the sellers mortgage remains on the land; thus, all of the remaining proceeds from the foreclosure
sale after the credit unions mortgage debt is satised go towards reducing the debt owed to the
bank.
APPROACH TO EXAMS
REAL PROPERTY
IN A NUTSHELL: The law of real property centers on a persons interest in land, which may be as
great as full ownership or as small as a right to enter. It governs how the land and those interests are
acquired and granted; bought and sold; rented and leased; and used as security for debts. Interests
in land arise through express creation (e.g., by a deed, will, or mortgage) and operation of law (e.g.,
through adverse possession). Realty may be owned by one individual or several, and an interest may
become possessory at once or in the future. However, when multiple parties claim conicting interests
in land, recording statutes dictate who will prevail. Real property law also governs items so afxed to
land that they are considered realty (i.e., xtures) and sets forth rights and responsibilities regarding the
use of water.
I. WHAT INTEREST IS INVOLVED?
A.
B.
a.
b.
c.
d.
e.
f.
g.
Any future interest that is not certain to vest or fail within a life in being plus 21 years
is void
Applies to contingent remainders, executory interests, class gifts (even if vested remainders), options and rights of rst refusal, and powers of appointment
Does not apply to vested interests, grantors reversionary interests, or gifts between
charities
Only the interest that violates the Rule is stricken (severed from the disposition)
Cases that always violate the common law Rule:
1) Executory interest following a defeasible feeexecutory interest is stricken
2) Gift to an open class conditioned on members surviving to an age beyond 21
entire class gift is stricken (bad as to one, bad as to all)
3) Remainder to As children living at his widows death (unborn widow
problem)contingent remainder is stricken
4) Gift conditioned on an administrative contingency is stricken
5) Options that might be exercised (not created) later than the Rules period are
stricken
At common law, a woman is conclusively presumed capable of bearing children (the
fertile octogenarian)
Departures from common law Rule:
1) Wait and see statutesvalidity of interest determined by actual future events
2) Uniform Statutory Rule Against Perpetuities90-year vesting period, wait and
see approach
3) Cy pres approachinvalid interests reformed to match grantors intent
C.
D.
Nonpossessory Interests
1. Easements
a. Afrmative easementright to use someone elses land
2.
3.
4.
5.
A.
1)
3.
B.
General warranty deedcovenants against any title defects created by the grantor
or prior titleholders
Special warranty deedcovenants against title defects created by the grantor
Quitclaim deedno covenants; transfers whatever interest grantor has
2)
3)
Wills
a. Effective on the testators death
b. If, at the testators death, she no longer owns property that was specically devised, that
gift fails (i.e., is adeemed)
c. If, at the testators death, the beneciary has already died, his gift fails (i.e., lapses) or
might pass to the beneciarys descendants under an anti-lapse statute if he and the
testator were related
Adverse Possession
1. Possessor must show: (i) actual entry giving rise to exclusive possession that is (ii) open and
notorious, (iii) adverse/hostile (i.e., lacking the owners permission), and (iv) continuous
throughout the statutory period for an ejectment action (e.g., 20 years)
2. The statute does not begin to run if the owner is under a disability to sue (e.g., incapacity)
when the possession begins
III. WHO WILL HOLD THE INTEREST?
A.
Concurrent Interests
1. All co-tenants share the right to possession and enjoyment of the property
2. Joint tenantstwo or more co-tenants with rights of survivorship (i.e., the dead co-tenants
share passes to the remaining co-tenants)
a. Created expressly, severed by a tenants sale or suit for partition
3. Tenants by the entiretytwo spouses with rights of survivorship
a. Created expressly or presumed in some states by a grant to spouses, severed by divorce
4. Tenants in commontwo or more co-tenants, no right of survivorship
a. Created by the severance of the above tenancies
b. Default co-tenancy created if nothing else was specied
B.
A.
2.
3.
4.
B.
A.
Fixtures
1. Fixtures are items so afxed to land that they become part of the realty
a. Constructive annexationitems not physically attached to land are xtures if they are
so uniquely adapted to the real estate that it makes no sense to separate them (e.g. keys
to doors)
2. Common ownership caseslandowner brings chattel onto land
a. Annexors objective intent determines whether items are xtures
3. Divided ownership caseslandowner does not bring chattel onto land
a. Items owner can remove it only if this would not leave unrepaired damage to the
premises
B.
Water
1. Rules vary by state and by source of water
a. Watercoursesrivers, streams, lakes
b. Groundwaterpercolating water from wells
c. Surface watersrainfall, melting snow, seepage
C.
Zoning
1. Governmental regulations that restrict the use of land
a. Existing zoning violations render title to land unmarketable
2. Variancepermission to depart from zoning restriction
Pauls Claim: Paul also has two possible arguments. His rst would be that Olivias conveyance
to X County created neither a fee simple determinable nor a fee simple subject to a condition subsequent, but rather a fee simple absolute with an afrmative covenant to use the two acres for a weighing
station. As a consequence, he would argue that X Countys transfer to Paul would not result in forfeiture of the land, but only in an action for damages against Paul for breach of covenant. While courts
will, in cases of substantial ambiguity, nd a covenant rather than a forfeitable interest, Paul will
probably lose on this argument because the condition and null and void language indicates that
forfeiture was clearly intended.
Alternatively, Paul would argue that Olivias conveyance created a fee simple subject to condition
subsequent, but failed to reserve a right of entry, resulting in a fee simple absolute in X County and,
consequently, a valid fee simple absolute in Paul. If, however, the court chooses to imply a right of
entry, Paul could then argue that the court should apply the common law rule, now in force in a small
number of states, that any attempt to transfer a right of entry inter vivos destroys the interest. As a
result, X Countys interest would be enlarged to a fee simple absolute because of the removal of the
condition subsequent, and Paul would now be the owner of a fee simple absolute.
Conclusions
David is entitled to the parcel if rights of entry and/or possibilities of reverter are transferable inter
vivos in the jurisdiction.
Henry wins if Olivias deed to David is construed not to transfer the reversionary interest (which is a
doubtful interpretation), or if the retained interest was a right of entry which, although descendible, is
not transferable inter vivos in a majority of states.
Paul wins if the condition in Olivias conveyance to X County is held to be a covenant (unlikely) or
if the retained interest was a right of entry and the common law rule barring transfer destroyed it (not
likely), thereby enlarging X Countys fee.
met the requirement of giving Landlord timely notice of damage. Unless the lease provides otherwise,
Landlords only liability to Tenant will be for damages, and Tenant will not be excused from paying
rent because the covenant to repair is independent of the covenant to pay rent.
Tenant will fail in an argument that the broken window constitutes constructive eviction, because
Landlord was not responsible for the damage. Tenant will also fail in an argument that the covenant of
habitability has been breached, because the broken window did not represent a substantial threat to her
health or safety.
Rehearsals: Landlord will not be liable to Tenant for Charlies rehearsals, and Tenant will have no
defense to an action for rent during this period. At issue is whether a landlord breaches the implied
covenant of quiet enjoyment or warranty of habitability by permitting another tenant to conduct daily
rehearsals in his apartment.
(1) Constructive Eviction: If the landlord does an act or fails to provide some service that he
has a legal duty to provide, and thereby makes the premises uninhabitable, the tenant may terminate
the lease and seek damages if she gives the landlord notice and a reasonable time to repair and then
vacates within a reasonable time. While the rehearsals probably constituted a substantial interference
with Tenants use and enjoyment of her apartment, they were not caused by Landlord. Although a small
handful of courts have taken the position that, by permitting one tenant to interfere with anothers
enjoyment, the landlord is himself responsible for that interference, this is not the majority rule. And,
even if the requirement of landlord conduct were met, Tenant did not vacate the premises quickly
enough to take advantage of the constructive eviction defense.
(2) Warranty of Habitability: If the premises become unsuitable for human residence, the tenant
may: (i) move out and terminate the lease, (ii) make repairs and offset the cost against future rent,
(iii) abate rent, or (iv) seek damages. The rehearsals probably did not represent a sufcient threat to
Tenants health for a breach of this warranty to be found. If, however, a court nds that loss of sleep
constitutes a sufcient injury to health, Tenant may collect damages from Landlord, or possibly have a
defense to an action for nonpayment of rent.
Narcotics Arrests: Landlord will not be liable to Tenant for Charlies friends possession of
narcotics on the premises. At issue is whether a landlord has a duty to prevent unlawful conduct
on the premises. As with the rehearsals, Tenant will encounter difculties in pursuing constructive
eviction and warranty of habitability claims. She might, in arguing constructive eviction, claim that the
unlawful conduct in Charlies apartment gave Landlord the right to terminate Charlies lease and thus
causally connected Landlord to the narcotics arrests. The two problems with this argument are that a
landlord cannot terminate a lease when the unlawful conduct is only occasional, as it was here, and the
conduct was not that of his tenant, Charlie, but rather of Charlies friends.
August 31 (When Tenant Moved Out) to May 31 (When Tenants Lease Expires)
Landlord has a right to recover rent from Tenant. At issue are a landlords rights and liabilities when
a tenant abandons the premises. Because no defenses are available to Tenant, Landlord can recover rent
for two months, July 1 to August 31. For the remainder of the lease term, Landlord can:
(1) Relet the premises on Tenants account, holding Tenant liable for any difference between the
rental payment under her lease and the rental paid by the new tenant; or
(2) If the jurisdiction follows the traditional rule, let the premises remain vacant, recovering rent
from Tenant as it becomes due.
whether the burden runs against Down as a successor to the promisor. The requirements for benet and
burden to run will be met for purposes of enforcing the agreement as an equitable servitude but not as
a negative covenant.
Intent that Lot 2 Enjoy the Benet of an Unimpaired View: The Adams-Baker agreement raises
a threshold issue of who is to enjoy the benet of the view. The recitals statement that Baker has
derived much pleasure from the view might suggest that the benet was to be held by Baker personally, in which case Park would not succeed to it and would be unable to enforce it. Similarly, the
recitals statement that the parties intended to assure an unobstructed view from Bakers house might
suggest an intent that the benet attach to the house rather than to the land, with the consequence that,
because the house has been destroyed, the benet cannot be enforced by Park.
However, the general constructional preference is for appurtenant benets, rather than benets in
gross. Because either of the above two interpretations would create a benet in gross, they will give
way to the third possible interpretation, under which the benet would be appurtenant: that Adams and
Baker intended that the benet attach neither to Baker nor to his house, but to Lot 2. Under this interpretation, the successor in interest to Lot 2 (Park) is in a position to enforce the benet.
The Adams-Baker Agreement Is Enforceable as an Equitable Servitude: In order for the benet
and burden of an equitable servitude to run, the covenant must touch and concern the land. This
requirement is met because the agreement increases the value and enjoyment of Lot 2. The one other
requirement for the benet to runthat Adams and Baker intended that Bakers successor enjoy the
benet in connection with Lot 2is met on the basis of the assumption made above, that the parties
intended that the benet attach to Lot 2.
The two other requirements for the burden to run have also been met. Intent is meteven though
the traditional formula, heirs, successors, and assigns, is missingbecause successors to Adams
would have to be bound if the purpose of the agreement, assuring an unobstructed view, is to be met.
Further, Bakers subsequent statement to Down about the building restriction provides some evidence
of an original intent to bind Adamss successors. The notice requirement is met by Bakers statement
to Down, putting Down at least on inquiry notice, and possibly on actual notice, of the existence of the
restriction.
Three possible equitable defenses may be asserted by Downabandonment, acquiescence, and
estoppelall premised on Parks absence from the state at the time Down started building the
two-story residence. However, because Park was unaware of Downs investment of labor, none of these
defenses is likely to be upheld.
The Adams-Baker Agreement Is Not Enforceable as a Negative Covenant: All requirements are
met for the benet of a negative covenant to run: intent (as in the equitable servitude); vertical privity
(because Park holds the complete interest in land held by Baker at the time the covenant was made);
and touch and concern (because the covenant increases the value and enjoyment of Lot 2). However,
the burden of the covenant does not run against Down. Although intent, notice (as in the equitable
servitude), vertical privity, and touch and concern are all met, the horizontal privity requirement is
not met. At the time Adams and Baker entered into the agreement, no independent interest in the land
passed between theme.g., there was no grantor-grantee or landlord-tenant relationship.
This Agreement Would Not Be Enforced as a Negative Easement: Under the analysis already
pursued, the easement would be appurtenant and, because the intent and notice requirements are met,
Park would be entitled to enforce the easement against Down. However, a court is not likely to characterize this as a negative easement. First, promissory language was used in the agreement (Adams
covenanted)suggesting a covenant rather than an easement, which would have been created by
grant or reservation. Second, while negative easements historically were limited to only four types of
arrangements (for light, air, subjacent or lateral support, or ow of an articial stream), none of which
is exactly like the one in dispute, today a negative easement is simply a restrictive covenant.
Thus, Park can probably obtain an injunction against Down on an equitable servitude theory. She is
less likely to recover damages, on either a negative covenant or negative easement theory, because there
appear to be no provable damages as of yet and, more important, the burden of the negative covenant
will be held not to run against Down, and the arrangement will not be construed as creating a negative
easement.
statute, because Bills recorded instrument appeared outside the chain of title and thus would not have
shown up in the course of a reasonable title search by Elk. Thus, under the majority chain-of-title
doctrine, Elk would prevail over Bill.
Thus, Elk has rst rights to Goldacre, Bill is second in priority, Christy is third, and Art has no
interest at all.